4120 1399
NOTEPAD
Results
of 259 questions answered correctly

You have reached of 259 points, ( %)

Your time

Question 1 of 259

1. A 28-year-old male patient complains of regurgitation, cough and heartburn that occurs every day after a meal, when bendi- ng forward or lying down. These problems have been observed for 4 years. Objective status and laboratory values are normal. FEGDS revealed endoesophagitis. What is the leading factor in the development of this disease?

Explanation

This patient presented with classic symptoms of Gastroesophageal Reflux Disease (GERD). Gastro-oesophageal reflux disease is a condition in which the reflux of gastric contents into the oesophagus provokes symptoms or complications and impairs quality of life. Typical symptoms of gastro-oesophageal reflux disease are heartburn and regurgitation. The pathogenesis of gastro-oesophageal reflux disease is multifactorial, involving transient lower esophageal sphincter relaxations (i.e failure of the inferior esophageal sphincter) and other lower esophageal sphincter pressure abnormalities. As a result, reflux of acid, bile, pepsin and pancreatic enzymes occurs, leading to esophageal mucosal injury as in this case Endoesophagitis. And the Esophagogastroduodenoscopy done confirms Endoesopahgitis as well.

H. pylori infection, Hypergastrinemia and Hypersecretion of hydrochloric acid can cause gastritis, gastric ulcer primarily but can also cause reflux disease if the lower or inferior esophageal sphincter is weak or relaxed. 

Duodeno-gastric reflux - Duodenum to Stomach.

 
2. A 70-year-old patient consulted a doctor about arrhythmic cardiac activity, dyspnea. Objectively: AP- 150/90 mm Hg, extrasystole arrhythmia (10-12 beats per minute), left ventricular systolic dysfuncti- on (ejection fraction at the rate of 42%). Which of antiarrhythmic drugs should be administered as initial therapy in this case?

Explanation

Amiodarone is a class III antiarrhythmic medication - potassium channel blockers. Amiodarone is a potent antiarrhythmic agent that is used to treat ventricular arrhythmias and atrial fibrillation. The drug prevents the recurrence of life-threatening ventricular arrhythmias and produces a modest reduction of sudden deaths in high-risk patients. Amiodarone is more effective than sotalol or propafenone in preventing recurrent atrial fibrillation in patients for whom a rhythm-control strategy is chosen.

Flecainide, Encainide, Moracizine are Class IC antiarrhythmic medication - Sodium channel blocker, commonly used only as a last resort in refractory Ventricular tachycardia. Digoxin is a cardiac glycoside that blocks Na+/K+ ATPase activity and improves contractility of the myocardium in heart failure. However, it can also be used in conditions such as Atrial Fibrillation.

 
3. A 57-year-old patient had an attack of retrosternal pain that lasted more than 1,5 hours. Objectively: the patient is inert, adynamic, has pale skin, cold extremities, poor volume pulse, heart rate - 120/min, AP- 70/40 mm Hg. ECG shows ST elevation in leads II, III, aVF. What condition are these changes typical for?

Explanation

This patient with ST segment elevation in leads II, III, aVF on ECG that also had an attack of retrosternal pain is definitely a case of STEMI - ST elevation Myocardial Infarction. With a blood pressure of 70/40mmHg (severe hypotension), the condition has deteriorated to a case of shock and in this case, it has a cardiac origin, therefore, it is a cardiogenic shock.

There is no arrhythmia recorded on ECG (no Atrial flutter or fibrillation, ventricular tachycardia or fibrillation), so it will be wrong to tag this an Arrhythmogenic shock. Perforated gastric ulcers can present with excessive bleeding and even result in shock but the patient will also present with hematemesis or melena and epigastric pain. Acute pericarditis can present with retrosternal pain but not likely to have shock. Acute pancreatitis will present with diarrhea, indigestion, epigastric belt-like pain radiating to the back, and it is unlikely for a patient with pancreatitis to have retrosternal pain and shock with the listed ECG findings.

 
4. A 63-year-old patient with persistent atrial fibrillation complains of moderate dyspnea. Objectively: peripheral edemata are absent, vesicular respiration is present, heart rate - 72/min, AP- 140/90 mm Hg. What combination of drugs will be most useful in the secondary prevention of heart failure?

Explanation

Beta blockers (drugs ending with the suffix ‘lol’ e.g. Metoprolol, propranolol, labetalol) can be used as antiarrhythmic drugs (treat arrhythmias like Atrial fibrillation). They belong to class II of the antiarrhythmic medications. They can also be used in hypertensive conditions. Beta 1 receptors are present in the heart and when a Beta blocker is deplored, it can reduce the heart rate thereby reducing the cardiac output and contractility. They can also reduce the transmission of impulses through the AV node which makes them useful as antiarrhythmic agents.

Angiotensin Converting Enzyme Inhibitors (ACEI) inhibit the conversion of Angiotensin I to Angiotensin II thereby inhibiting the vasoconstrictive properties of Angiotensin II. This makes them very useful as antihypertensives. 

In this case, the question is interested in how to prevent heart failure and a combination of Beta blockers and ACE inhibitors are the most effective as they both reduce blood pressure and workload on the heart. They reduce both the preload and afterload of the heart which will prevent ventricular remodelling in the long run.

Cardiac glycosides tend to increase heart contractility and in the long run, this increase in workload can lead to hypertrophy. Diuretics can reduce the preload but not effectively reduce the afterload.

 
5. A 19-year-old patient complains of dyspnea on exertion. He often has bronchitis and pneumonia. Since childhood, the patient presents with cardiac murmur. Auscultation revealed splitting of the II sound above the pulmonary artery, systolic murmur in 3 intercostal space at the left sternal border. ECG showed right bundle branch block. What is the provisional diagnosis?

Explanation

First heart sound - Mitral and Tricuspid valve

Second heart sound - Aortic and Pulmonic valve

Closure of these valves during cardiac cycle produces the sound. 

The second heart sound can have a physiologic splitting i.e. the aortic valve closes just before the pulmonic valve. This can occur when there is more blood flow through the pulmonic valve for example during inspiration there is increased blood return to the right side of the heart via the Superior vena cava and Inferior vena cava. This increased blood volume takes a longer time to pass through the pulmonic valve and this causes a slight delay - the aortic valve closes just before the pulmonic valve causing a physiologic splitting of S2.

Also, splitting of the second heart sound is commonly seen in Atrial Septal Defect, Pulmonic valve stenosis or right bundle branch block. In Atrial Septal Defect, there is a left to right shunt. Blood from the left atrium moves to the right atrium when they contract and this increases the blood volume in the right side of the heart. This increased blood volume takes longer to pass through the pulmonic valve. Thus, producing a split in the second heart sound.

Open Ductus Arteriosus will produce a continuous machine-like murmur throughout systole (heart contraction) and diastole (heart relaxation). Mitral insufficiency will produce a systolic murmur (holosystolic - throughout systole). Aortic stenosis will produce a bruit or murmur beyond the point of the stenosis. None of these other conditions produces a splitting of the second heart sound.

 
6. An emergency doctor has diagnosed a 32-year-old woman with generalized convulsive status epilepticus. The deterioration in the patient’s condition is caused by a sudden gap in the epilepsy treatment. Specify the doctor’s further tactics:

Explanation

Status epilepticus is said to occur when a seizure lasts too long or when seizures occur close together and the person doesn't recover between seizures. Over the last several decades, the length of seizure that is considered as status epilepticus has shortened. Years ago, a seizure needed to last longer than 30 minutes to be considered status epilepticus. In the last few years, it is now defined as any seizure greater than 5 minutes. This makes sense because most seizures do not last longer than 2 minutes. Very long seizures (i.e., status epilepticus) are dangerous and even increase the chance of death.

Status epilepticus requires emergency treatment by trained medical personnel in a hospital setting. This situation can be life-threatening and getting treatment started fast is vital.

 
  • Medical treatment needs to be started as soon as possible. Oxygen and other support for breathing, intravenous fluids (fluid given into a blood vessel), and emergency medications are needed.

  • Continuous EEG (electroencephalogram) monitoring may be needed to monitor the seizures and how a person responds to treatment.

  • At times, medicines called anesthetics are used in the hospital to put a person into a coma to stop the seizures. - This will definitely be done in the Intensive Care Unit (ICU).

7. A 26-year-old patient with affective bipolar disorder has developed a condition manifested by mood improvement, behavioural and sexual hyperactivity, verbosity, active body language, reduced need for sleep. Which of the following drugs are most effective in this case?

Explanation

Neuroleptics, also known as antipsychotic medications, are used to treat and manage symptoms of many psychiatric disorders. They fall into two classes: first-generation or "typical" antipsychotics and second-generation or "atypical" antipsychotics." They are Dopamine D2 receptor antagonists. Antipsychotics are drugs that have a specific sedative effect, and which improve the attitude and calm the behavior of psychotic patients.

Typical antipsychotics: Haloperidol, Chlorpromazine

Atypical antipsychotics: Risperidone, Clozapine, Aripiprazole

Neuroleptic drugs are beneficial for the management of behavioral disorders. They are also associated with sedative effects, weight gain, or anticholinergic activity.

Bipolar disorder: a manic episode and depressed mood. For treatment of Bipolar disorders: Lithium, Valproic acid, Lamotrigine, Carbamazepine, and Atypical antipsychotics. This patient is currently having a manic episode and it's best to use a Neuroleptic (antipsychotic) with sedative effect to calm the patient down and to sleep.

A manic episode requires 3 or more of the following:

  • Decreased need for sleep

  • Distractibility

  • Impulsivity/Indiscretion: seeks pleasure without regard to consequences including sexual pleasure

  • Talkativeness or pressured speech

  • Grandiosity: inflated self esteem

  • Flight of ideas: racing thoughts

  • Increased goal oriented activity/psychomotor agitation

 

Antidepressants are used for Major Depressive Disorders; Tranquilizers are used for General Anxiety Disorder or Panic attacks.

8. A 48-year-old patient complains of weakness, subfebrile temperature, aching pain in the kidney region. These presentations turned up three months ago after hypothermia. Objectively: kidneys are painful on palpation, there is bi- laterally positive Pasternatsky’s symptom. Urine test res: acid reaction, pronounced leukocyturia, microhematuria, minor proteinuria - 0,165-0,33 g/l. After the urine sample had been inoculated on conventional media, bacteriuria were not found. What research is most required in this case?

Explanation

There are some characteristics in a urine examination that suggest a diagnosis of renal Tuberculosis, such as acid pH, leukocyturia and/or hematuria, associated with negative urine culture for the usual bacteria that causes urinary tract infection (bacteria were not found on conventional media).

Points worthy of note from the Urine test result: acid reaction, leukocyturia and after inoculation on conventional media, bacteriuria were not found.

Now, Mycobacterium tuberculosis is acid fast due to the presence of Mycolic acid, so they will produce an acid reaction. There is increased lymphocytes (leukocyturia) with M.tuberculosis and viral infection (Neutrophils will increase with bacteria - Neutrophilia). Also, M. tuberculosis will not grow on conventional media and they stain poorly with Gram staining because their cell wall has high lipid content. To detect M. tuberculosis, Ziehl-Neelsen stain (Carbol fuchsin) is used instead.

Nechiporenko: This method helps to determine the amount of cellular elements (WBC, RBC and casts) in 1 ml of urine.

Zimnitsky's test characterize the concentrating and excretory ability of the kidney.

Daily proteinuria monitoring will give information about the extent of nephrotic syndrome or any other nephropathy like hypertensive nephropathy.

Isotope renography is an imaging technique that’s used to check kidney function.

 
9. A 22-year-old vegetarian patient with signs of malnutrition consulted a doctor about smell and taste distortion, angular stomatitis. Objectively: expressively blue sclerae. The patient was diagnosed with iron deficiency anemia. What is the dominating clinical syndrome?

Explanation

Vegetarian diets are dietary patterns that are devoid of all flesh foods but may include egg or dairy products. Vegetarians have lower incidences of several health conditions including ischemic heart disease, total incidence of cancer, and type 2 diabetes. Despite these advantages, vegetarians also are at a high risk of a deficiency of some nutrients, such as vitamin B12 and iron. 

This patient with malnutrition also complained about smell and taste distortion. Loss of smell and/or taste has been linked to inadequate nutritional intake, reduced social pleasure, and decreased psychological well-being.

Anemia is defined by reduced hemoglobin level or the erythrocytes volume in the circulation. Sideropenic anemia is a hypochromic, microcytic anemia caused by insufficient iron level in the body. This is the most common anemia. 

Patients with Iron deficiency often manifest Pica. Pica is an unusual craving for and ingestion of either edible or inedible substances. It seems to be strongly associated with iron deficiency anemia, and in the majority of cases the unusual eating and chewing behavior disappears upon iron supplementation.

 

Patients with the diagnosis of sideropenic anemia are recommended to take iron supplements. By changing eating habits we can maximize iron input into our organism and prevent sideropenic anemia.

10. A week ago a 65-year-old patient suffered an acute myocardial infarction, his general condition deteriorated: he complains of dyspnea at rest, pronounced weakness. Objectively: edema of the lower extremities, ascites is present. Heart borders are extended, paradoxical pulse is 2 cm displaced from the apex beat to the left. What is the most likely diagnosis?

Explanation

One of the most common complications occurring post-infarction is a ventricular aneurysm. Almost 85% to 90% of the ventricular aneurysms occur in the setting of acute anterior wall myocardial infarction. Left ventricular aneurysm is defined as a localized area of myocardium with abnormal outward bulging and deformation during systole and diastole. The natural course leading to the formation of a ventricular aneurysm involves a full-thickness (transmural) infarct that has been replaced by fibrous tissue. This inert portion cannot take part in the contraction and herniates outward during systole.

A week before presenting to the hospital, he had an acute myocardial infarction. A true left ventricular aneurysm following acute myocardial infarction can occur as early as within 48 hours or two weeks post-infarction. 

Common symptoms associated with the ventricular aneurysm include: Fatigue; Shortness of breath (dyspnea); Chest pain; Palpitations; Syncope; Fluid retention causing swelling of ankles, feet, or abdomen (ascites); Stroke; Limb or visceral ischemia.

 

A recurrent myocardial infarction will present with the classic substernal chest pain or tightness. Acute pericarditis will produce chest pain as well and friction rub between the pericardial sheets. Pulmonary embolism is usually a complication of deep vein thrombosis (DVT).

11. In a cold weather, the emergency room admitted a patient pulled out of the open water. There was no respiratory contact with the water. The patient is excited, pale, complains of pain, numbness of hands and feet, cold shiver. Breathing rate is 22/min, AP- 120/90 mm Hg, Ps- 110/min, rectal temperature is 34, 5oC. What kind of warming is indicated for this patient?

Explanation

Hypothermia is a severe condition in which the body temperature drops to an abnormally low level. It occurs when the body is unable to produce enough heat to counter the heat that it is losing. Under healthy conditions, the body maintains a relatively stable temperature of around 98.6˚F or 37˚C. 

The following techniques can help treat hypothermia:

Passive external rewarming: This uses the individual’s heat-generating ability. It involves removing their cold, wet clothing, ideally replacing it with adequately insulated, dry clothing, and moving them to a warm environment. 

Active external rewarming: This involves applying warming devices, such as hot-water bottles or warmed forced air, externally to truncal areas of the body. For example, the individual could hold a hot-water bottle under each arm.

Active core rewarming: This uses warmed, intravenous fluids to irrigate body cavities, including the thorax, peritoneum, stomach, and bladder. Other options include getting the individual to inhale warm, humidified air, or applying extracorporeal rewarming by using a heart-lung machine.

Do not give a person alcohol if they have signs of hypothermia, and avoid giving any drinks to an unconscious person.

 

From the vitals given, this patient is fairly stable except for the tachycardia (110bpm) and low body temperature, therefore passive warming will suffice. The patient was pulled out of the open water in a cold weather so removing the wet clothing, and replacing it with adequately insulated, dry clothing, and moving them to a warm environment will keep the patient warm.

12. Six months ago, a 5-year-old child was operated for CHD. For the last 3 weeks he has complained of fever, heart pain, aching muscles and bones. Examination results: \"white-coffee\" skin colour, auscultation revealed systolic murmur in the region of heart along with a noise in the III-IV intercostal space. Examination of fingertips revealed Janeway lesions. What is your provisional diagnosis?

Explanation

Infective endocarditis (IE) is caused by damage to the endocardium of the heart followed by microbial, usually bacterial, colonization. It is an infection in the heart valves or endocardium. The endocardium is the lining of the interior surfaces of the chambers of the heart. Infective endocarditis disproportionately affects those with underlying structural heart disease and is increasingly associated with healthcare contact (the child was operated for congestive heart disease - CHD), particularly in patients who have intravascular prosthetic material. 

 

Symptoms include: fever (most common); Janeway lesions (small, painless, erythematous lesions on palm or sole); new cardiac murmur; Roth spots (round white spots on retina surrounded by hemorrhage); Osler nodes (raised lesions on finger or toe pads due to immune complex deposition); and splinter hemorrhages on nail bed.

13. A 58-year-old patient complains of a headache in the occipital region, nausea, choking, opplotentes. The presentations appeared after a physical exertion. Objectively: the patient is excited. Face is hyperemic. Skin is pale. Heart sounds are regular, the 2nd aortic sound is accentuated. AP- 240/120 mm Hg, HR- 92/min. Auscultation reveals some fine moist rales in the lower parts of the lungs. Liver is not enlarged. ECG shows signs of hypertrophy and left ventricular overload. What is the most likely diagnosis?

Explanation

The patient presented with a blood pressure of 240/120mmHg which is markedly elevated. This increased blood pressure already led to the development of hypertrophy and left ventricular overload as stated in the question stem which are common complications associated with extremely high blood pressure. Initially, cardiac hypertrophies are usually compensatory mechanisms but if the blood pressure is not promptly and properly managed, it can become a full blown cardiac failure.

It is clear that this is a case of Complicated Hypertensive crisis. It is complicated with hypertrophy and pulmonary edema. The pulmonary edema is evidenced by the objective findings on auscultation of the lungs that revealed fine moist rales in the lower parts of the lungs.

 

This is not a case of acute myocardial infarction. Myocardial infarctions are often associated with severe substernal chest pain or tightness. There was no respiratory distress of any sort, so we can rule out both bronchial asthma and pneumonia.

14. A 58-year-old patient complains of general weakness, loss of 10 kg of weight within 1,5 months, progressive pain in the lumbar region, increased blood pressure up to 220/160 mm Hg, subfebrile temperature. Objectively: in the right hypochondrium palpation reveals a formation with uneven surface and low mobility; veins of the spermatic cord and scrotum are dilated. Blood test results: Hb- 86 g/l, ESR- 44 mm/h. Urine test results: specific gravity - 1020, protein - 0,99 g/l, RBCs - cover the whole field of vision, WBCs - 4-6 in the field of vision. What is the provisional diagnosis?

Explanation

The fact that the patient lost a lot of weight in a short period of time in the absence of any other pathology should signal a carcinogenic process. Key diagnostic features of this pathology include; blood in urine, presence of flank mass and pain accompanied with weight loss and high blood pressure. Note, most renal pathologies present with elevated blood pressure due to the Renin-Angiotensin-Aldosterone System (RAAS). The dilated veins of the spermatic cord and scrotum (varices) is due to disrupted venous blood flow caused by increased pressure from the obstruction caused by the tumour.

A patient with urolithiasis will also experience colicky (more like an attack with relieve period in between) flank pain, change in quantity of urine and can be differentiated by results from ultrasound which shows an obstruction in the ureter. 

 

Glomerulonephritis often follows an upper respiratory tract infection 2-3weeks prior to presentation and can also present with a nephrotic syndrome (increased proteinuria accompanied by presence of edema). Pyelonephritis is inflammation of the kidney parenchyma which can be as a result of infection via an ascending or hematogenous pathway.

15. After a contact with chemicals a plant worker has suddenly developed stridor, voice hoarseness, barking cough, progressing dyspnea. Objective examination reveals acrocyanosis. What is your provisional diagnosis?

Explanation

Laryngeal edema is a common complication in patients that have allergic predisposition. Symptoms of stridor, hoarseness, and tachypnea developed within 24 hours after contact with the offending agent which could be a bee sting, chemicals, animal dander, dust, pollen etc and subside within another 24 hours after inhalation therapy with heated aerosol mist. 

 

The contact with chemicals led to the edematic reaction in the worker’s larynx. This is a possible irritation caused by chemical agents. Recall that the larynx is the voicebox; this is why the edematic reaction presents with symptoms such as stridor, voice hoarseness, barking cough etc.

16. A 40-year-old female patient complains of having a bulge on the anterior surface of neck for 5 years. Objectively: Ps- 72 bpm, arterial pressure - 110/70 mm Hg, in the right lobe of thyroid gland palpation reveals a mobile 4x2 cm node, the left lobe is not palpable, the basal metabolic rate is 6%. What is the most likely diagnosis?

Explanation

 

Notice that the Blood pressure and heart rate are normal which indicates no case of hyperthyroidism or hypothyroidism. Recall that a goiter occurring in a hyperthyroid state is a Hyperthyroid goiter; one that occurs in a hypothyroid state is a hypothyroid goiter while a goiter that occurs in a normal physiologic state as it is in this patient is an Euthyroid goiter. This is a nodular form because on palpation, a mobile node was palpated.

17. A 40-year-old female patient has been hospitalized for attacks of asphyxia, cough with phlegm. She has a 4-year history of the disease. The first attack of asphyxia occurred during her stay in the countryside. Further attacks occurred while cleaning the room. After 3 days of inpatient treatment the patient’s condition has significantly improved. What is the most likely etiological factor?

Explanation

 

The patient was admitted as a result of an allergic attack. From anamnesis, we find that these attacks mostly occurred while cleaning the room indicating that the causative factor is a household allergen. - Type I Hypersensitivity Reaction.

18. 5 weeks after hypothermia a 22-year-old patient developed fever, weakness, muscle pain, inability to move independently. Objectively: tenderness, induration of shoulder and shin muscles, restricted acti- ve movements, erythema on the anteri- or surface of chest. There is a periorbi- tal edema with heliotropic erythema. Gottron’s sign is present. What study is required to verify the diagnosis?

Explanation

Gottron’s sign refers to erythematous macules/papules present on the elbows ( sometimes also on the knees) and are specific signs in diagnosis of dermatomyositis.

Dermatomyositis is a muscle disease, a long term inflammatory myopathy (muscle disorder) characterised by progressive proximal muscle weakness, Gottron papules and heliotrope rash. For diagnosis, a muscle biopsy is obtained and observed under the microscope; signs such as mononuclear antibodies, abnormal cell degeneration and regeneration  etc are observed.

 
19. A 24-year-old patient consulted a doctor about enlarged submandibular lymph nodes. Objectively: submandi- bular, axillary and inguinal lymph nodes are enlarged. Chest radiograph shows enlarged mediastinal lymph nodes. In blood: RBCs - 3,4 · 1012/l, Hb- 100 g/l, colour index - 0,88, thrombocytes - 190 · 109/l, WBCs - 7,5 · 109/l, eosinophils - 8%, stab neutrophiles - 2%, segmented neutrophiles - 67%, lymphocytes - 23%, ESR - 22 mm/h. What study is required to verify the cause of lymphadenopathy?

Explanation

20. A  patient operated for acute paraprocti- tis undergoes antibacterial and detoxi- fication therapy, the local course of the disease has the positive dynamics. Since the operation the patient has had chi- lls, pyrexia, tachycardia, euphoria for five days. The doctor suspected sepsis. What study will confirm the diagnosis?

Explanation

 

Sepsis refers to the body’s extreme response to infection. This patient most probably got infected during the surgical procedure; pyrexia, chills, tachycardia etc are key signs of the body’s reaction. To confirm the diagnosis and also find out the particular pathogen, a blood culture should be conducted.

21. A painter working at a motorcar plant has been diagnosed with moderately severe intoxication with ami- de compounds of benzene. The in-patient treatment resulted in a considerable health improvement. What expert deci- sion should be made in this case?

Explanation

22. An 18-year-old patient since childhood suffers from bleeding disorder after minor injuries. His younger brother also has bleeding disorders with occasional haemarthrosis. Which laboratory test will be informative for diagnosis verification?

Explanation

Clotting time is the time required for a sample of blood to coagulate in vitro under standard conditions. The time taken for blood to clot mainly reflects the time required for the generation of thrombin.  If the plasma concentration of prothrombin or of some of the other factors is low (or if the factor is absent, or functionally inactive), clotting time will be prolonged. The expected range for clotting time is 4-10 mins.

Clotting time involves the measurement of prothrombin time (PT) and activated partial thromboplastin time (aPTT). Prothrombin time assesses the extrinsic (factor VII) and common (factors X, V, II [prothrombin] and I [fibrinogen]) pathways, whereas aPTT assesses the intrinsic (prekallikrein, HMWK, and factors XII, XI, IX, and VIII) and common pathways.

 
23. A patient complains of fatigue, lack of appetite, pain and burning sensation in the tongue, numbness of the distal limbs, diarrhea. Objectively: pale skin with lemon-yellow tint, face puffiness, brown pigmentation in the form of a \"butterfly\", bright red areas on the tongue. The liver is 3 cm below the costal margin, soft. Blood count: RBCs - 1, 5 · 1012 /l, colour index - 1,2, WBCs - 3, 8 · 109 /l, thrombocytes - 180 · 109/l, eosinophils - 0%, stab neutrophils - 1%, segmented neutrophils - 58%, lymphocytes - 38% monocytes - 3%, RBC macrocytosis. ESR - 28 mm/h. What diagnosis are these presentation typical for?

Explanation

From the blood analysis given, Macrocytes are present- this refers to irregular large Red Blood cells. Also the color index is elevated. The erythrocyte level is also very low ( norm- Female: 3.5 − 5.5 · 1012/L) - this indicates an anemia. The major causes of macrocytic anemia include Vitamin B12 deficiency, Vitamin B9 (Folic Acid) deficiency or medications such as antiretroviral drugs. 

Symptoms include loss of appetite, brittle nails , pale skin, fatigue etc. To differentiate between B12 or Folic acid deficiency, Vit B12 deficiency is often associated with nervous system symptoms like paresthesias - subacute combined degeneration due to abnormal myelin synthesis. These nervous system symptoms are absent in Folic Acid Deficiencies.

With the symptoms listed in the question, and the presence of anemia and macrocytes - it is safe to say Vitamin B12 deficiency is the best choice.

Ascorbic acid or vitamin C deficiency will lead to scurvy, petechial bleeding or bruises are common in this case. Vitamin B1 deficiency will lead to the development of Beri-Beri.

 
24. Medical examination of a 19- year-old worker revealed generalized lymphadenopathy mainly affecting the posterior cervical, axillary and ulnar lymph nodes. There are multiple injection marks on the elbow bend skin. The man denies taking drugs, the presence of injection marks ascribes to influenza treatment. Blood count: RBCs- 3, 2·1012/l, Hb- 100 g/l, WBCs- 3, 1 · 109/l, moderate lymphopenia. What study is required in the first place?

Explanation

 

From physical examination, we observe the presence of multiple injection marks on the patient’s body. This should raise a suspicion about an intravenous drug abuser and an infection with HIV. HIV is an immunosuppressive disease and patients suffering from this disease are prone to having opportunistic infections. For diagnosis of HIV infection, HIV antibodies are found in serum or saliva via ELISA test ( Enzyme linked Immunosorbent assay) and is usually confirmed by a western blot test.

25. During the periodic medical examination an assembly fitter (works on soldering details) didn’t report any health problems. Closer examination revealed signs of asthenic-vegetative syndrome. Blood included red blood cells with basophilic aggregations and a somewhat higher number of reticulocytes, urine had a high concentration of delta- aminolevulinic acid. The complex of symptoms indicates the initial stage of chronic intoxication with:

Explanation

 

Delta-aminolevulinic acid dehydratase is an enzyme necessary for the synthesis of heme. In lead poisoning, this enzyme is inhibited leading to anemia and an increased delta aminolevulinic acid content in urine. N/B a high reticulocyte number means an increase in RBC production ( new cells) and is observed in cases such as bleeding, high altitude or anemia. The above patient presents with signs of asthenic vegetative syndrome coupled with the above stated reasons confirming a case of lead poisoning.

26. A  50-year-old patient complains of bursting pain in the left lower limb that is getting worse on exertion, swelling in the region of shin and foot. Objectively: left shin and foot are doughy, skin of the lower shin is indurated and has a bronze tint, subcutaneous veins are dilated, there is an ulcer with necrotic masses. What is the most likely diagnosis?

Explanation

Post-thrombotic syndrome refers to symptoms and signs of chronic venous insufficiency that develop following deep vein thrombosis (DVT) and is a common, burdensome, and costly complication . The term "post-thrombotic" replaces the prior terminology "postphlebitic" syndrome. A combination of reflux due to valvular incompetence and venous hypertension due to thrombotic obstruction is thought to underlie the pathophysiology of post-thrombotic syndrome . Symptoms and signs of post-thrombotic syndrome may include leg pain, leg heaviness, vein dilation, edema, skin pigmentation, and venous ulcers.

Deep vein thrombosis occurs when a blood clot  forms in a deep vein usually in the thigh and lower leg. It presents with pain, warmth and tenderness of the affected area. 

Acute arterial thrombosis occurs after an endovascular procedure results of the erosion or rupture of atherosclerotic plaque and/or distal embolization (DE). Both occurrences activate the coagulation and platelet systems, resulting in the occlusion of the artery by a thrombus.

 
27. A 48-year-old patient was found to have diffuse enlargement of the thyroid gland, exophthalmia, weight loss of 4 kg in 2 months, sweating. Objectively: HR- 105/min, AP- 140/70 mm Hg. Defecati- on act is normal. What kind of therapy is recommended in this case?

Explanation

 

The patient experiences an enlarged thyroid gland, tachycardia, loss of weight  and exophthalmos - these symptoms are consistent with Hyperthyroidism ( thyrotoxicosis). For treatment: Mercazolil (thiamazole) is administered. Propranolol is a beta blocker and used in heart related issues while thyroxine is indicated for hypothyroidism.

28. A 48-year-old male patient complains of constant pain in the upper abdomen, mostly on the left, that is getting worse after taking meals; diarrhea, weight loss. The patient is an alcohol abuser. 2 years ago he had acute pancreatitis. Blood amylase is 4 g/h·l. Coprogram shows steatorrhea, creatorrhea. Blood glucose is 6,0 mmol/l. What treatment is indicated for this patient?

Explanation

This patient has a case of acute Pancreatitis. Recall that the pancreas has both endocrine and exocrine functions; for the exocrine part, it produces enzymes that aid in digestion such as amylase, lipase, trypsin. Absence of these enzymes will lead to symptoms such as steatorrhea  (fats in feces), creatorrhea (undigested muscles in feces) etc. Panzinorm Forte is a combination of digestive enzymes. These enzymes are normally produced by the pancreas and are important for digesting fats, proteins, and sugars.

Panzinorm Forte is used to replace digestive enzymes when the body does not have enough of its own. Certain medical conditions can cause this lack of enzymes, such as cystic fibrosis, pancreatitis, pancreatic cancer, or pancreas surgery.

Panzinorm Forte may also be used to treat a condition called steatorrhea (loose, fatty stools).

Pirenzepine is an M1 selective antagonist used in treatment of peptic ulcer ( it reduces gastric acid secretion and muscle spasm). Contrykal is a protease inhibitor and is used in reducing blood loss in surgical procedures. Drotaverine is an antispasmodic drug used for cervical dilation in childbirth

 
29. A 20-year-old patient complains of severe headache, double vision, weakness, fever, irritability. Objectively: body temperature is at the rate of 38, 1oC, the patient is reluctant to contact, sensitive to stimuli. There is ptosis of the left eyelid, exotropia, anisocoria S>D, pronounced meningeal syndrome. On lumbar puncture the cerebrospinal fluid flowed out under a pressure of 300 mm Hg, the fluid is clear, slightly opalescent. 24 hours later there appeared the fibrinous film. Protein - 1,4 g/l, lymphocytes - 600/3 per mm3, sugar - 0,3 mmol/l. What is the provisional diagnosis?

Explanation

Meningitis is an acute infectious disease with involvement of the arachnoid and pia of the brain and spinal cord by pathogenic microorganisms. Etiologically, the various forms include; bacterial, viral. Fungal and tuberculous meningitis. The tuberculous form occurs as a secondary infection; The CSF is under increased pressure, is clear or slightly cloudy, and contains a predominance of mononuclear cells (usually >400/mm3), increased protein (100 to 400 mg/dL), and a decreased glucose content.
30. A 32-year-old patient has developed an acute condition after hypothermia: temperature - 40oC, cough with 200 ml of sputum per day. The sputum is purulent, foul-smelling. To the right of the lower lobe the mixed moist rales can be auscultated. Blood tst results: WBCs - 18, 0 · 109/l, ESR - 45 mm/h. Radiographi- cally: in the lower lobe of the right lung there is a thick-walled cavity up to 6 cm in diameter with a high horizontal level. What is the most likely diagnosis?

Explanation

31. A family consists of 5 persons. The husband is a stope miner. His spouse is a housewife. Their 20-year-old daughter works as a kindergarten teacher. Their 18-year-old son is a student. The grandmother is a pensioner, she has diabetes. Which member of this family can be primarily classed among the group of persons with a high risk of tuberculosis in the planning of preventive examinations for tuberculosis?

Explanation

Generally, persons at high risk for developing TB disease fall into two categories:

  • Persons who have been recently infected with TB bacteria

  • Persons with medical conditions that weaken the immune system

Persons in the second group include individuals with the following diseases; HIV infection (the virus that causes AIDS), Substance abuse, Silicosis, Diabetes mellitus, Severe kidney disease, Low body weight, Organ transplants, Head and neck cancer, Medical treatments such as corticosteroids or organ transplant, Specialized treatment for rheumatoid arthritis or Crohn’s disease.

32. A  patient with suspected pheochromocytoma has normal blood pressure in the periods between the attacks and a tendency to tachycardia. Urine test revealed no pathology. It was decided to use a provocative test with histamine. What medication should be prepared to provide emergency care in case of a positive test result?

Explanation

Pheochromocytoma is a tumor that affects the chromaffin cells of the adrenal medulla. It leads to the increased production of catecholamines ( epinephrine and norepinephrine) which have vasoconstrictive effects leading to hypertension. Amongst the listed drugs, Phentolamine which is a non-selective alpha adrenergic receptor blocker is used in the management of hypertensive emergencies majorly due to pheochromocytoma. This drug has a vasodilating  effect. 

Pipolphen ( promethazine) is an antihistamine and is used in the treatment of allergic reactions, motion sickness, anaphylaxis etc. Mesaton is a decongestant and a vasopressor.

 
33. A 54-year-old patient complains of frequent painful urination, chills, fever up to 38*C. Urine test results: protein - 0,33 g/L, WBCs - up to 50-60 in the field of vision, RBCs - 5-8 in the field of vision, gram-negative bacilli. Which of the listed antibiotics should be preferred in this case?

Explanation

 

From the results of physical examination and biochemical analysis, we can conclude that this patient is having a urinary tract infection ( UTI) most likely of bacterial origin. From the groups of antibiotics listed below, Ciprofloxacin is the best of choice because of its high bacteriological and clinical cure rates as well as low resistance rate from bacteria that cause UTIs.

34. Several hours before, a 28-year- old patient suddenly developed acute headache and repeated vomiting, then lost consciousness. Objectively: focal neurological symptoms were not found. Pronounced meningeal symptoms were revealed. AP - 120/80 mm Hg. According to clinical and liquorological findings the patient was diagnosed with subarachnoid haemorrhage. After administration of dehydrants the patient’s condition somewhat improved. What is the main component of further emergency care?

Explanation

 

This patient is experiencing a subarachnoid hemorrhage (bleeding into the subarachnoid space). From the list of drug groups, anticoagulants (against coagulation), antiaggregants (against aggregation) and fibrinolytics ( breakdown fibrin) will worsen the current situation if administered. A coagulant will help in the management of the hemorrhagic  situation.

35. Gastric juice analysis of a 42-year- old male patient revealed absence of free hydrochloric acid at all stages. Endoscopy revealed pallor, thinning of gastric mucosa, smoothed folds. Microscopically the atrophy of glands with intestinal metaplasia was found. What disease is this situation typical for?

Explanation

The following types of chronic gastritis include;

Type A is caused by your immune system destroying stomach cells. And it can increase your risk of vitamin deficiencies, anemia, and cancer.

Type B, the most common type, is caused by Helicobacter pylori bacteria, and can cause stomach ulcers, intestinal ulcers, and cancer.

Type C is caused by chemical irritants like nonsteroidal anti-inflammatory drugs (NSAIDs), alcohol, or bile. And it can also cause stomach lining erosion and bleeding.

Observe that on examination, free HCL is absent and results from endoscopy and microscopy indicate the gastric cells have been destroyed - this occurs in Type A gastritis.

In menetrier's disease, we see an overgrowth of gastric cells ( foveola) found in the gastric lining which leads to the presence of large gastric folds.

 
36. A 24-year-old female teacher complains of dizziness and heart pain irradiating to the left nipple. Pain is not associated with physical activity and cannot be relieved by nitroglycerin, it abates after taking Valocordin and lasts an hour or more. The patient has a nearly 2-year history of this disease. Objectively: Ps- 76 bpm. AP- 110/70 mm Hg. Heart borders are normal, heart sounds are clear. The ECG shows respiratory arrhythmia. Radiograph of the cervicothoracic spine shows no pathology. Lungs, abdomen are unremarkable. What changes in blood formula can be expected?

Explanation

 

From the description of the above patient, we can deduce that the issue is more psychological/mental than physical. This can be proven by the fact that all diagnostic procedures are without any pathological changes. Note that the problem was resolved when the patient was prescribed  Valocordin (corvalol) a tranquilizer indicated for neuroses. Since it is a mental issue, the blood formula will show no changes.

37. A 60-year-old male patient, who works as a construction worker, complains of pain in the right hip and knee joints, that is getting worse on exertion. These presentations have been observed for the last 5 years. Objectively: the patient is overnourished. Right knee joint is moderately deformed. Examination of other organs and systems revealed no pathology. Blood test results: WBCs - 8, 2 · 109/l, ESR - 15 mm/h. Uric acid - 0,35 mmol/l. What is the most likely diagnosis?

Explanation

Osteoarthritis is the inflammation of the joint that occurs due to continuous exertion of that area ( wear and tear); unlike Rheumatoid arthritis that is of autoimmune origin.  Observe that the presentation of this patient has been observed for the last 5 years, is one sided and worsens on exertion- these facts help differential from rheumatoid arthritis that has a rapid onset ( weeks- months), occurs symmetrically and improves upon usage of the affected area. 

Reactive arthritis is an autoimmune condition that develops in response to an infection in another part of the body. Coming into contact with bacteria and developing an infection can trigger reactive arthritis. It has symptoms similar to various other conditions collectively known as "arthritis,". It is caused by another infection and is thus "reactive". The symptoms of reactive arthritis very often include a combination of three seemingly unlinked symptoms—an inflammatory arthritis of large joints, inflammation of the eyes (conjunctivitis and uveitis), and urethritis. A useful mnemonic is "the patient can't see, can't pee and can't bend the knee" or "the patient can't see, can't pee and can't climb a tree". Also known as Reiter’s syndrome.

 
38. A  32-year-old female complains of dizziness, headache, palpitation, tremor. For the last several months she has been under outpatient observation for the increased arterial pressure. Since recently such attacks have become more frequent and severe. Objectively: skin is covered with clammy sweat, tremor of the extremities is present. HR- 110/min, AP- 220/140 mm Hg. Heart sounds are muffled. Blood test results: WBCs - 9, 8 · 109/l, ESR - 22 mm/h. Blood glucose - 9,8 millimole/l. What disease is the most likely cause of this crisis?

Explanation

 

The woman not only shows signs of hypertension but also presents with changes relating to body metabolic activities (tremors, tachycardia, dizziness, palpitations, sweating - these are signs relating to stress ( caused by the release of stress hormones i.e. catecholamines). Pheochromocytoma is a tumor of the adrenal gland, it is characterised by the increased production of catecholamines ( adrenaline, noradrenaline) coupled with stress symptoms. Essential hypertension is also called primary hypertension- a form of hypertension with no underlying disease as its cause. Preeclampsia is a pregnancy-associated hypertension coupled with other signs. Primary hyperaldosteronism  or Conn’s disease is also associated with the above stated signs , its other signs may include kidney related problems, excessive urination, low potassium content, flank pain etc.

39. A  55-year-old male has a 1,5-year history of viral cirrhosis with symptoms of portal hypertension. Over the last month the weakness has progrssed, there appeared coffee ground vomit. Fibrogastroduodenoscopy revealed variceal esophageal haemorrhage. What drug should be used to reduce the pressure in the portal vein?

Explanation

Note that vasopressin is the most potent vasoconstrictor of internal organs, it decreases blood flow to all abdominal organs, decreasing venous inflow into the portal veins and portal pressure. Vasopressin always should be accompanied by intravenous nitroglycerin  to maintain a systolic blood pressure of greater than 90 mm Hg.

Reserpine is an antihypertensive drug and acts by reducing the amount of norepinephrine. Calcium gluconate is used in the treatment of magnesium toxicity, hypocalcemia and hyperkalemia.

 
40. A 40-year-old patient complains of fever up to 39*C, cough with sputum and blood admixtures, dyspnea, weakness, herpetic rash on the lips. Objectively: respiration rate - 32/min. Under the shoulder blade on the right the increased vocal fremitus and dullness of percussion sound were revealed. Auscultation revealed bronchial respiration. Blood count: WBCs - 14 · 109/l, ESR - 35 mm/h. What is the provisional diagnosis?

Explanation

Vocal fremitus refers to vibrations transmitted through the body. It is usually increased when the lung tissue becomes dense and is mainly seen in diseases such as pneumonia. The presence of relative dullness on percussion coupled with results from auscultation all indicate Pneumonia.  Croupous pneumonia is also known as Lobar pneumonia, fibrinous pneumonia or lung fever.
41. In the morning a patient had nausea, abdominal discomfort, single vomiting, dry mouth. In the evening, the patient presented with the increasing general weakness, double vision, difficult swallowing of solid food. Objectively: ptosis, mydriasis, anisocoria, absence of gag and pharyngeal reflex, dry mucous membranes. The previous evening the patient had dinner with canned food and alcohol. What is the presumptive diagnosis?

Explanation

 

Botulism is an illness caused by Botulinum toxin , a toxin produced by Clostridium Botulinum ( a gram positive, spore forming, anaerobic bacteria). Foodborne botulism is gotten from eating infected food especially canned food. Symptoms include; disturbed vision (diplopia), flaccid paralysis, dysphagia, dyspnea etc. It is treated with Botulinum antitoxin.

42. A  28-year-old female patient with a six-year history of Raynaud’s syndrome has recently developed pain in the small joints of hands, difficult movement of food down the esophagus. What kind of disease can you think of in this case?

Explanation

Systemic scleroderma  is a chronic multisystem disorder of unknown etiology characterized clinically by thickening of the skin caused by accumulation of connective tissue and by involvement of visceral organs, including the gastrointestinal tract, lungs, heart, and kidneys. Common symptoms include Raynaud phenomenon, polyarthralgia, dysphagia, heartburn, and swelling and eventually skin tightening and contractures of the fingers. 

In Periarteritis nodosa (PAN), small and medium sized vessels are inflamed leading to disruption in major organs such as nerves, intestinal tract, heart, and joints.

Systemic lupus erythematosus is characterised by a malar/ butterfly rash.

 
43. A 47-year-old patient came to see a doctor on the 7th day of disease. The disease developed very fast: after the chill body temperature rose up to 40*C and lasted up to 7 hours, then it dropped abruptly, which caused profuse sweat. There were three such attacks occuring once in two days. Two days ago the patient arrived from Africa. Objectively: pale skin, subicteric sclera, significantly enlarged liver and spleen. What is the cause of fever attacks in this disease?

Explanation

From anamnesis, we understand that the patient has just returned from Africa , this should prompt the idea of a malarial infection. Malaria is caused by a plasmodium infection (P.vivax, P.falciparum, P.malariae, P.ovale. In its life cycle, various stages are involved ( see image).

The initial symptoms of malaria infection are nonspecific and can include headache, nausea, vomiting, photophobia and muscle aches. A malarial paroxysm is marked by onset of a sudden shaking chill which may last from 10 to 15 minutes or perhaps longer. Elevated temperature accompanies the paroxysm and may be sustained for typically 10 hours or more. This cycle repeats itself every 36 to 72 hours depending on which species the human host has been infected with. The phase of erythrocytic schizogony takes place within  day 6-15 of the disease ( patient arrives on Day 7); it is characterised by the production of merozoites which are released into the bloodstream.
44. On the 2nd day of illness a 27- year-old patient complains of unbearable headache, repeated vomiting. Objecti- vely: the patient is in a grave condition. He is conscious but adynamic. Lies in a forced position with his head thrown back. There is no skin rash. Nuchal muscles are evidently rigid, there are Kernig’s and Brudzinski’s signs. to- 39, 5oC, Ps- 120/min, AP- 130/80 mm Hg. The leading syndrome of this disease is caused by:

Explanation

45. A 42-year-old female patient suffers from micronodular cryptogenic cirrhosis. Over the last week her condition has deteriorated: she developed convulsions, mental confusion, progressing jaundice. What study may give reasons for such aggravation?

Explanation

Recall that ammonia is converted via the urea or ornithine cycle into urea; this conversion takes place in the mitochondria of liver cells. This patient has liver cirrhosis which means that liver cells are damaged; the presented symptom of convulsions and mental confusion are due to the fact that ammonia can cross the blood brain barrier thereby affecting the brain’s function. To confirm this, the level of ammonia in the patient’s blood should be checked.

Alpha fetoprotein is a tumor marker that indicates tumors related to the liver, ovaries, testes etc. In embryology, it is elevated in cases of neonatal defects such as down’s syndrome and spina bifida.

 
46. A 60-year-old patient complains of recurrent pain in the proximal interphalangeal and wrist joints, their periodic swelling and reddening that have been observed for 4 years. X-ray picture represents changes in form of osteoporosis, joint space narrowing and single usuras. What is the most likely diagnosis?

Explanation

So basically, we need to differentiate Rheumatoid Arthritis (RA) and Osteoarthritis (OA) so that we can arrive at the correct answer.

RA: autoimmune, which erodes articulated cartilage and bone. The inflammatory cells and cytokines then induce pannus (proliferative granulation tissue) formation, deep usurations.

Associated with pain, swelling and morning stiffness, lasting >1hr and the pain improves with use.

Joint findings: joint space narrowing; erosions; juxtaarticular osteopenia; soft tissue swelling; subchondral cysts

Involves Metacarpophalangeal joints; wrist; proximal interphalangeal joints. Does not affect distal interphalangeal joints on the hands

 

OA: mechanical wear and tear - it’s a degenerative joint disease.

Associated with Pain in weight bearing joints after use (i.e. at the end of the day), and the pain improves with rest.

Joint findings: joint space narrowing; osteophytes (bone spurs); subchondral sclerosis and cysts.

Involves distal interphalangeal joints on the hands (heberden nodes) and Proximal interphalangeal joints (Bouchard nodes); does not affect Metacarpophalangeal joints and wrist.

 

So let’s answer the question:

From the question stem - Radiocarpal (wrist joints); Proximal interphalangeal joints were affected with usurations.

These are clearly signs and symptoms of Rheumatoid Arthritis...

 
47. A 57-year-old male patient complai- ns of dyspnea on exertion, heaviness in the right hypochondrium and shin edemata towards evening. Objectively: temperature - 38, 1oC, HR- 20/min, HR=Ps=92/min, AP- 140/90 mm Hg. There is apparent kyphoscoliosis. In the lungs single dry rales can be auscultated. Heart sounds are muffled, rhythmic. ECG: Rv1+Sv5=15 mm. X-ray picture shows the bulging of pulmonary artery cone, right ventricle enlargement. What is the most likely cause of this condition?

Explanation

48. A 40-year-old woman with a history of combined mitral valve disease with predominant stenosis complains of dyspnea, asthma attacks at night, heart problems. At present, she is unable to do easy housework. What is the optimal tactics of the patient treatment?

Explanation

A commissurotomy is a surgical procedure used in people with narrowing/ stenosis of heart valves. It involves the removal of deposits on the valve leaflets such as calcium, scar etc.  An artificial valve replacement will be carried in situations of degenerative valve diseases, very low ejection fraction < 30%.
49. A 25-year-old patient complains of having dull heart pain for the last 10 days, dyspnea on mild exertion, palpi- tations. The diasease developed 2 weeks ago after a respiratory infection. Objecti- vely: acrocyanosis, AP- 90/75 mm Hg, Ps- 96/min. Cardiac borders appear to be shi- fted to the left and right. Heart sounds are weak and have triple rhythm, there is systolic murmur at the apex. ECG showed sinus rhythm, complete left bundle branch block. What is the most likely diagnosis?

Explanation

50. While staying in a stuffy room a 19- year-old emotionally labile girl developed severe weakness, dizziness, blackout, nausea and loss of consciousness without convulsions. Objectively: the patient is unconscious, the skin is pale, extremities are cold. AP- 90/60 mm Hg, Ps- 96/min, deficient, breathing is shallow. Pupillary and tendon reflexes are present. There are no pathological signs. What is the most likely diagnosis?

Explanation

Notice that the question states ‘loss of consciousness without convulsions’ ‘ this rules out the option of an epileptic attact which is characterised by seizures that sometimes lead to  loss of consciousness. Syncope is simply termed fainting; it occurs as a result of reduced blood flow to the brain; notice that the blood pressure is very low- this leads to a reduced blood flow to major organs of the body especially the brain. This is a leading mechanism for the development of syncope.

Hysterical neurosis is a neurologic disorder characterised by sensory and motor disturbances as well as emotional outbursts and breakdown.

 
51. A patient complains of frequent, bulky, frothy stools with greenish mucus, crampi- ng pain in the umbilical region, abdomi- nal murmur, body temperature at the rate of 39o C . The patient associates the disease with consumption of soft-boiled eggs. What is the most likely pathogen?

Explanation

52. X-ray picture of chest shows a density and an abrupt decrease in the upper lobe of the right lung. The middle and lower lobe of the right lung exhibit significant pneumatization. The right pulmonary hilum comes up to the dense lobe. In the upper and middle parts of the left pulmonary field there are multiple focal shadows. In the basal region of the left pulmonary field there are clear outlines of two annular shadows with quite thick and irregular walls. What disease is this X-ray pattern typical for?

Explanation

53. A patient complains of impaired far vision. Previously his eyes often turned red and hurt. Objectively: the eyes are not irritated, the cornea is transparent, anterior chambers are median deep, their liquid is transparent. The iris of the right eye has not changed in colour, its pattern is unchanged. The pupil is of irregular shape, scalloped. Biomicroscopy of the crystalline lens reveals the areas of opacity and vacuoles. Make a diagnosis:

Explanation

Cataract is the opacification (clouding) of the lens which leads to a decrease in vision (visual acuity). The visual acuity in adults is 60/60 or 6/6 which equals 1. Notice that on examination, the cornea, anterior chamber , iris showed no pathology but there are areas with opacities and vacuoles on the crystalline lens that indicates the presence of cataract. Also notice the irregularities in the pupil ( it is pathologic). A complicated cataract is one accompanied by or caused by an intraocular pathology - in this case the patient’s cataract is accompanied by the defect in the pupil.

Note that senile cataract is age-related ( progressive clouding and thickening of the lens as one gets older).

Tetanic cataract is mostly seen in individuals with hypoglycemia especially after a thyroidectomy. Also referred to as hypocalcemic cataract.

 
54. A 59-year-old male complains of heart pain, cough, fever up to 38*C . Three weeks ago he suffered a heart attack. Objectively: Ps- 86/min, rhythmic, blood pressure - 110/70 mm Hg. Auscultation reveals pericardial rub, rales beneath the shoulder blade. Radiography reveals no pathology. Blood count: WBCs - 10 · 109/l, ESR - 35 mm/h. ECG shows no dynamics. It would be most reasonable to administer the drugs of the following pharmaceutical group:

Explanation

 

From anamnesis, this patient presents with fever, pain, a sign of pericarditis- inflammation of the heart pericardium ( the presence of pericardial rub on auscultation) etc. From the listed options of medications to be given, fibrinolytics and coagulants should be given in the case of a clot formation, Nitrates and nitrites are vasodilators; they may relieve the pain associated with Ischemic heart disease but have no effect on fever and the inflammatory process. Antibiotics would have been highly considered but notice that from blood analysis, the WBC is just a little bit above normal while the ESR is within normal range. Recall that glucocorticoids have anti-inflammatory, immunosuppressive and antipyretic effects and in this case will be best suited amongst the available options.

55. A 40-year-old female patient complains of headache, dizziness, muscle weakness, occasional cramps in the extremities. She has been taking anti-hypertensive medications for 10 years. AP- 180/100 mm Hg. Blood potassium - 1,8 millimole/l, sodium - 4,8 millimole/l. In urine: alkaline reaction, the relative density - 1012, protein and sugar are not found, WBCs - 3-4 in the field of vision, RBCs - 1- 2 in the field of vision. Conn’s syndrome is suspected. Which drug should be chosen for the treatment of arterial hypertension?

Explanation

 

While considering the medication for hypertension, we should put into consideration that this patient is experiencing hypokalemia ( low level of potassium in the blood)(Potassium, plasma 3.5-5.0 mmol/L is the normal range) ; note that the alkaline urine reaction is as a result of an increased excretion of potassium in the urine. So while administering the antihypertensive medication, we should also try to reduce or stop the further excretion of potassium. In this case, the best possible option is spironolactone which is a Potassium sparing diuretic (causes urine excretion but prevents potassium removal).

Propranolol ( beta blocker) and enalapril ( sodium channel blocker) are anti hypertensive drugs too but they don't prevent the excretion of potassium. Hydrochlorothiazide is a thiazide diuretic, it reduces sodium reabsorption in the distal convoluted tubule and keeps calcium in the blood.

56. A 45-year-old patient, a sailor, was hospitalized on the 2nd day of the disease. A week ago he returned from India. Complains of body temperature of 41*C, severe headache, dyspnea, cough with frothy rusty sputum. Objectively: the patient is pale, mucous membranes are cyanotic, breathing rate is 24/min, tachycardia is present. In lungs: diminished breath sounds, moist rales over both lungs, crepitation. What is the most likely diagnosis?

Explanation

The result from auscultation indicates pneumonia in both lungs (diminished breath sounds, moist rales over both lungs). This patient has just returned from India and most likely got this disease during his travel. Pneumonic plague is a form of plaques ( others are septicemic, bubonic etc.) caused by Yersinia Pestis; usually transmitted via infected rats and rodents. This disease has higher frequencies in Asia, and Africa. Also note that RUSTY sputum is usually seen in patients with pneumonia caused by bacteria. In pneumonic plague the infection is in the lungs, in bubonic plague the lymph nodes, and in septicemic plague within the blood.
57. A 53-year-old female patient complains of cardiac pain and rhythm intermissions. She has experienced these presentations since childhood. The patient’s father had a history of cardiac arrhythmias. Objectively: the patient is in grave condition, Ps- 220 bpm, AP- 80/60 mm Hg. ECG results: heart rate - 215/min, extension and deformation of QRS complex accompanied by atrioventricular dissociation; positive P wave. Some time later heart rate reduced down to 45/min, there was a complete dissociation of P wave and QRST complex. Which of the following will be the most effective treatment?

Explanation

 

On a normal ECG, the P wave represents atrial depolarization (contraction), QRS complex- ventricular depolarization (contraction) and T wave - ventricular repolarization (relaxation). The sinus atrial node is the natural pacemaker of the heart; it produces electrical impulses at the rate of 60-70 beats per minute.

In this patient, at certain periods, she experiences fibrillations while at other times, she experiences bradycardia. Coupled with results from the ECG (complete dissociation of P wave and QRST complex), we can conclude that this patient has a defective pacemaker. An artificial pacemaker will be the most effective treatment for this patient. Calcium antagonists (blockers) will only be useful in periods of tachyarrhythmia but are contraindicated in bradycardia.

58. A  49-year-old patient complains of dyspnea, cough. There are no sputum discharges. He has repeatedly used salbutamol and intal but with no effect. Objectively: he is only able to sit while leaning on the table. Cyanosis of face, acrocyanosis are present. Breathing is shallow, laboured, in some parts it cannot be auscultated; there are diffuse rales, expiration is significantly prolonged. Heart sounds are muffled, tachycardia is present. Ps - 112/min, AP- 110/70 mm Hg. Liver is located near the costal arch. There are no peripheral edemata. What is your provisional diagnosis?

Explanation

This patient presents with difficulty in breathing, cough, no sputum production, and is cyanotic - these are signs that indicate an asthmatic attack. But also notice that salbutamol has no effect on his current situation (recall that salbutamol is beta-2 adrenergic receptor agonist usually in the form of inhalers; they cause relaxation - bronchodilation of airway smooth muscles). The current state of the patient coupled with unresponsiveness to the medication indicates a state known as Status Asthmaticus. It is defined as a state of severe asthmatic attack which does not respond to conventional treatment.

N/B the fact that this patient does not respond to salbutamol rules out the option of bronchial asthma (moderate gravity) and cardiac asthma is usually a complication of heart failure.

 
59. Blood typing resulted in positive isohemagglutination reaction with standard sera of А(II) and В(III) groups and negative reaction with sera of 0(I) and АВ(IV) groups. What is this result indicative of?

Explanation

Blood Group (ABO and Rh)

Different blood groups present with different antigens and antibodies.

Group I (O) presents with anti A and anti B antibodies but no antigen.

Group II (A) presents with anti B antibody and A-antigen.

Group III (B) presents with anti A antibody and B- antigen

Group IV (AV) presents with no antibodies and antigen A and antigen B. Note that the presented antigen determines the blood group.

 In terms of compatibility, Group IV (AB) is a universal recipient because it lacks antibodies, while Group I (O) is a universal donor because it lacks antigens; Group II (A) can receive from A and O but can donate to A and AB; Group III(B) can receive from B and O but can donate to B and AB.

From the question stem, a positive reaction with Group II (A) and III(B) shows that anti A and B antibodies are present in the sample blood (indicating group I (O)) - but also notice that when the blood sample was tested with group I (O) and IV (AB), a negative reaction occurs. This proves that the standard sera used in testing is faulty and is currently giving inaccurate results because there should be a positive reaction with Blood Group AB (IV) - it  has both A and B antigen.

60. A patient complains of jerking, throbbing pain in the III finger on the right hand. The patient associates these pain onsets with an injury by a nail. The finger skin is hyperemic and tense, palpation with a bulbous-end probe reveals the most painful area. What is the provisional diagnosis?

Explanation

 

A felon (a form of whitlow) is an acute and painful inflammation of the deeper tissues of the fingers or toes. It is characterised by the presence of an abscess in the affected area. Paronychia is a bacterial or fungal infection of the nail (where the nail meets the skin). Pandactilitis is an inflammation of the entire finger or toe joints, in this case, the inflamed digits look like sausages.

61. A 24-year-old female patient complains of pain in the right hypochondrium that is getting worse after taking meals; nausea, fever up to 37,7*C, icteric skin, pain in the large joints. These presentations have been observed for 8 months. Objectively: hepatosplenomegaly. Blood test results: ESR- 47 mm/h, total bilirubin - 86,1 mmol/l, direct bilirubin - 42,3 mmol/l. Total protein - 62 g/l, albumins - 40%, globulins - 60%, gamma globulins - 38%. Viral hepatitis markers were not detected. The antibodies to smooth muscle cells are present. On ultrasound the portal vein diameter was of 1 cm. What is the most likely diagnosis?

Explanation

 

Presence of pain in the right hypochondrium should indicate that the problem lies either in the liver, gallbladder or bile ducts. Coupled with an increased level of bilirubin, antibodies against smooth muscles were found from biochemical analysis. Note that Smooth muscles antibodies are  found in autoimmune type hepatitis. This is because certain liver diseases such as hepatitis and cirrhosis trigger the body to form antibodies against smooth muscle. Other antibodies that can be seen in autoimmune hepatitis include Antinuclear antibodies, Anti-actin antibodies, Anti-soluble liver antigen/liver pancreas (anti-SLA/LP) antibodies. Other antibodies may be present, even when the anti-smooth muscle antibodies are absent.

62. A patient is 60 years old, retired, worked as deputy director of a research institute. Behavioural changes appeared 2 years ago after the death of her husband: she stopped looking after herself and leaving the house; then she refused to clean the apartment and cook. Mental status: temporal disorientation. The patient does not understand many of the questions, is confused; does not know how to cook soup or fasten a button. Her speech is characterized by stumbling and logoclonia. She does not recognize doctors, fellow patients. She cries a lot but can not explain the reason for tears. What is the mechanism of this pathology?

Explanation

 

Recall that the cerebral cortex is made up of different lobes, sulci and gyri that act as centres for various functions for example the occipital lobe for vision. The cerebral cortex can further be divided into brodmann's area with each area having a specific function eg area 44 & 45 (broca’s area) - for motor activities in speech production etc. This patient presents with temporal disorientation, problems with understanding questions, repetition of syllables (logoclonia), long term memory defects- these signs are mostly due to a defect in the Temporal lobe ( responsible for long term memory, processing auditory and visual sensory input, language recognition and formation of new memories). She also has defects with the cognitive function - can't clean or cook, fasten her button, confused etc.

Note that from anamnesis, since this patient does not have any underlying disease that can lead to an atherosclerotic formation, that option is ruled out. The above stated symptoms are indicative of a global cortex dysfunction affecting a wide area of the cerebral cortex and most likely due to the atrophy of the cerebral cortex.

63. A 27-year-old sexually active female complains of numerous vesicles on the right sex lip, itch and burning. Eruptions regularly turn up before menstruation and disappear 8-10 days later. What is the most likely diagnosis?

Explanation

Herpes simplex viruses are enveloped double stranded linear viruses. HSV-1 is also known as oral herpes ( gingivostomatitis, keratoconjunctivitis etc) while HSV-2 is known as genital or neonatal herpes. The clinical presentation of genital herpes include  pain, itching, dysuria, vaginal and urethral discharge, tender lymphadenopathy, appearance of herpes vesicles on the  external genitalia, labia majora, labia minora, vaginal vestibule - for women and glans penis, the prepuce, the shaft of the penis, and sometimes on the scrotum, thighs, and buttocks- for men.

Primary syphilis usually presents with a localized painless hard chancre on the genitals.

Cytomegalovirus is also known as human herpes virus-5, it is usually seen in immunocompromised patients and infected cells have characteristic ‘ owl's eye’ intranuclear inclusions. 

 
64. A 26-year-old patient with left lower lobe pneumonia experiences an acute chest pain on the left during coughing. Objectively: diffuse cyanosis, extension of the left side of chest. Percussion reveals high tympanitis. Auscultation reveals no respiratory murmurs above the left side of chest. There is a deviation of the right cardiac border towards the midclavicular line. What examination will be the most informative?

Explanation

65. After myocardial infarction, a 50- year-old patient had an attack of asthma. Objectively: bubbling breathing with frequency of 32/min, cough with a lot of pink frothy sputum, acrocyanosis, swelling of the neck veins. Ps- 108/min, AP- 150/100 mm Hg. Heart sounds are muffled. Mixed moist rales can be auscultated above the entire lung surface. What drug would be most effective in this situation?

Explanation

66. Against the background of angina a patient has developed pain in tubular bones. Examination revealed generalized enlargement of lymph nodes, hepatolienal syndrome, sternalgia. In blood: RBCs - 3, 6 · 1012 /l, Hb- 87 g/l, thrombocytes - 45 · 109/l, WBCs - 13 · 109/l, blasts - 87%, stab neutrophils - 1%, segmented neutrophils - 7%, lymphocytes - 5%, ESR - 55 mm/h. What is the most likely diagnosis?

Explanation

Leukemias are malignant neoplasms of the hematopoietic stem cells characterized by diffuse replacement of the bone marrow by neoplastic cells.

From the question stem, using the full blood count, we can denote that there is Leukemia due to the presence of immature blast cells. To distinguish between Acute and  Chronic leukemia, the blast count is always very important. Blasts are immature white blood cells.

In chronic Leukemia, the blood and bone marrow contain less than 10% blasts (blast - 2%). In Acute leukemia, blasts are usually more than 20%. Using the lab values given in the question, we have 87% of blast cells, which makes it Acute Leukemia..

 
67. Examination of a 43-year-old man objectively revealed pallor of skin and mucous membranes, loss of tongue papillae, transverse striation of fingernails, cracks in the mouth corners, tachycardia. Blood test results: Hb- 90 g/l, anisocytosis, poikilocytosis. The most likely causative agent of this state is inadequate intake of:

Explanation

Iron deficiency anaemia is a condition where a lack of iron in the body leads to a reduction in the number of red blood cells. Iron is used to produce red blood cells, which help store and carry oxygen in the blood. If you have fewer red blood cells than is normal, your organs and tissues won't get as much oxygen as they usually would. Causes include blood loss ( bleeding, menorrhagia etc), poor diet, malabsorption and hookworms. Key findings include; koilonychia (spoon shaped, brittle nails), atrophic glossitis (loss of filiform papillae on the tongue), angular cheilitis (inflammation of mouth corners) etc. Note that Anisocytosis refers to the presence of RBCs of various sizes while poikilocytosis refers to RBCs of various shapes - both findings are usually in Iron deficiency anemias. 

Copper deficiency will present with signs such as difficulty in learning and forming memories, weak and brittle bones, difficulty walking, vision loss (Kayser–Fleischer ring in eyes). 

 
68. A patient had 4 generalized convulsive seizures within a day. Between the seizures the patient did not maintain clear consciousness (was in a coma or stupor). Specify his state:

Explanation

Status epilepticus (SE) is a common, life-threatening neurologic disorder that is essentially an acute, prolonged epileptic crisis. Status epilepticus is defined as a continuous seizure lasting more than 30 min, or two or more seizures without full recovery of consciousness between any of them. Based on recent understanding of pathophysiology, it is now considered that any seizure that lasts more than 5 min probably needs to be treated as SE.

It can also be defined as a seizure that persists for a sufficient length of time or is repeated frequently enough that recovery between attacks does not occur. Therefore, the key phrase here is “recovery between attacks does not occur.” The question puts it this way - ‘between the seizures the patient did not come to waking consciousness.’

Prolonged Status epilepticus can lead to cardiac dysrhythmia, metabolic derangements, autonomic dysfunction, neurogenic pulmonary edema, hyperthermia, rhabdomyolysis, and pulmonary aspiration. Permanent neurologic damage can occur with prolonged SE.

 
69. A 45-year-old patient complains of fever up to 40o C , general weakness, headache and spasmodic contraction of muscles in the region of a shinwound. The patient got injured five days ago when ti- lling soil and didn’t seek medical attenti- on. What kind of wound infection can be suspected?

Explanation

70. A 60 year-old female has been suffering weakness, dizziness, fatigue over the last year. Recently she has also developed dyspnea, paresthesia. Objectively: skin and mucous membranes are pale and slightly icteric. The tongue is smooth due to the loss of lingual papillae. Liver and spleen are located at the costal margin. Blood count: Hb- 70 g/l, RBCs - 1, 7·1012/l, colour index - 1,2, macrocytes. Administer the patient a pathogenetically justified drug:

Explanation

From the blood analysis given, Macrocytes are present- this refers to irregular large Red Blood cells. Also the color index is elevated. The erythrocyte level is also very low (norm- Female: 3.5 − 5.5 · 1012/L) - this indicates an anemia . The major causes of macrocytic anemia include Vitamin B12 deficiency, Vitamin B9 (Folic Acid) deficiency or medications such as antiretroviral drugs. 

Symptoms include loss of appetite, brittle nails , pale skin, fatigue etc. To differentiate between B12 or Folic acid deficiency, Vitamin B12 deficiency is often associated with nervous system symptoms like paresthesias - subacute combined degeneration due to abnormal myelin synthesis. These nervous system symptoms are absent in Folic Acid Deficiencies.

With the symptoms listed in the question, and the presence of anemia and macrocytes - it safe to say Vitamin B12 deficiency is the best choice.

 

Ascorbic acid or vitamin C deficiency will lead to scurvy, petechial bleeding or bruises are common in this case. Vitamin B1 deficiency will lead to the development of Beri-Beri. Iron deficiency will result in Anemia but macrocytes will be absent on the blood film, instead we will see microcytes.

71. A 57-year-old female complains of having a sensation of esophageal compresion, palpitation, difficult breathing during eating solid food, occasional vomiting with a full mouth, \"wet pillow\" sign at night for the last 6 months. Objectively: body temperature - 39*C, height - 168 cm, weight - 72 kg, Ps- 76/min, АP- 120/80 mm Hg. X-ray revealed a considerable dilation of esophagus and its constriction in the cardial part. What pathology is most likely to have caused dysphagia in this patient?

Explanation

Esophageal achalasia is an esophageal motility disorder involving the smooth muscle layer of the esophagus and the lower esophageal sphincter (LES). It is characterised by an increased LES tone, incomplete LES relaxation and lack of esophageal peristalsis. The sign of “wet pillowresults from increased salivation and nocturnal discharge of saliva and mucus from the mouth and is observed in esophageal pathologies. Diagnosis is confirmed by x-ray examination and from the above patient, we see the dilation of esophagus and its constriction in the cardial part which confirms our diagnosis.
72. A 49-year-old patient consulted a doctor about difficult swallowing, voice hoarseness, weight loss. These symptoms have been gradually progressing for the last 3 months. Objectively: the patient is exhausted, there are enlarged supraclavicular lymph nodes. Esophagoscopy revealed no oesophageal pathology. Which of the following studies is most appropriate in this case?

Explanation

A CT scan of the chest can help find problems such as infection, lung cancer, blocked blood flow in the lung (pulmonary embolism), and other lung problems. It also can be used to see if cancer has spread into the chest from another area of the body. Note that this patient most likely has cancer since there has been loss of weight over 3 months, progressing symptoms coupled with enlarged lymph nodes around this area. A lung x-ray gives a pictorial view of the heart, lungs, airways, blood vessels and the bones of the spine and chest but a CT scan is more detailed. Because it is able to detect very small nodules in the lung, chest CT is especially effective for diagnosing lung cancer at its earliest, most curable stage. Imaging of the esophagus is not needed as the question clearly stated that the esophagus revealed no pathology.
73. A 45-year-old female patient complaining of general weakness, nausea and vomiting has been delivered to a hospital by the ambulance. Recently there has been a lack of appetite, weight loss. Objectively: hyperpigmentation of skin, blood pressure at the rate of 70/45 mm Hg, bradycardia. Additional studies revealed the reduced concentration of aldosterone and cortisol in blood, decreased excretion of 17-ketosteroids and 17-oxyketosteroids in the urine, hyponatremia, chloropenia, hypokalemia. What therapeutic measures are required?

Explanation

This patient presents with decreased aldosterone (mineralocorticoid); cortisol (glucocorticoid); hyponatremia; hypotension - these are signs of adrenal insufficiency because aldosterone and cortisol are primarily produced in the Adrenal gland.

Adrenal crisis, also termed acute adrenal insufficiency is an acute life-threatening condition. It is considered one of the endocrine emergencies precipitated by an internal or external process in the setting of known or unknown lack of production of the adrenal hormone cortisol, the major glucocorticoid.

The adrenal gland\'s main function is to produce both mineralocorticoids and glucocorticoids. There are other hormones secreted by the adrenal glands including catecholamines and precursors of sex hormones. The adrenocorticotropic hormone (ACTH) is produced by the pituitary gland and its main function is to stimulate the cortisol release from the adrenal glands.

The main function of aldosterone is sodium retention and potassium secretion. The cortisol promotes gluconeogenesis, increases sensitivity to catecholamines, and regulates the immune system. If a patient with primary insufficiency has an adrenal crisis, they may be found to be hyponatremic in addition to having hypoglycemia and hypotension due to both aldosterone and cortisol deficiency, respectively. They also present with hyperpigmentation as a result of increased ACTH levels.

It is now clear that the best treatment is to administer hormones that have low levels - glucocorticoids (cortisol); mineralocorticoids (aldosterone) and a diet with high content of cooking salt to correct hyponatremia.

74. A 23-year-old female patient has a mental disease since the age of 18, the course of disease has no remission periods. At a hospital the patient mostly presents with non-purposeful foolish excitation: she makes stereotypic grimaces, exposed, masturbating in front of a loud laugh, repeating the stereotypical abusive shouts. The patient should be assigned:

Explanation

Neuroleptics, also known as antipsychotic medications, are used to treat and manage symptoms of many psychiatric disorders. They fall into two classes: first-generation or "typical" antipsychotics and second-generation or "atypical antipsychotics." They are Dopamine D2 receptor antagonists. Antipsychotics are drugs that have a specific sedative effect, and which improve the attitude and calm the behavior of psychotic patients.

Typical antipsychotics: Haloperidol, Chlorpromazine

Atypical antipsychotics: Risperidone, Clozapine, Aripiprazole

This patient is currently having a manic episode and it\'s best to use a Neuroleptic (antipsychotic) with sedative effect to calm the patient down.

A manic episode requires 3 or more of the following:

  • Decreased need for sleep

  • Distractibility

  • Impulsivity/Indiscretion: seeks pleasure without regard to consequences including sexual pleasure

  • Talkativeness or pressured speech

  • Grandiosity: inflated self esteem

  • Flight of ideas: racing thoughts

  • Increased goal oriented activity/psychomotor agitation

Antidepressants are used for Major Depressive Disorders; Tranquilizers are used for General Anxiety Disorder or Panic attacks. Mood stabilizers like Lithium, Valproic acid, Lamotrigine, Carbamazepine are commonly used in Bipolar disorders. Nootropics are used to enhance mental function not mania.

75. A young woman with seborrhea adiposa has numerous non-itchy light brown and white spots with clear outlines and defurfuration on the torso and shoulder skin. What is the provisional diagnosis?

Explanation

The above description is typical for Tinea (pityriasis) versicolor which is caused by malassezia spp. A yeast like fungus. It is characterised by areas of hypopigmentation ‘white spots’. Hyperpigmentation can also occur due to inflammatory response - ‘areas of brown spots .’ It differs from dermatophytes because it is less pruritic (itchy).

Basically, all the other options listed here are itchy, while Pityriasis versicolor is non-itchy.

 

Pityriasis rosea is a viral rash that resembles small oval red patches and are very itchy. Seborrheic dermatitis is a common skin disease that causes an itchy rash with flaky scales and mainly affects the scalp, face and trunk. Vitiligo is a condition that is associated with pathological destruction of melanocytes.

76. An emergency physician arrived to provide medical care for a hangman taken out of the loop by his relatives. The doctor revealed no pulse in the carotid arteries, absence of consciousness, spontaneous breathing and corneal reflexes; cadaver spots on the back and posterior parts of extremities. A person can be declared dead if the following sign is present:

Explanation

Forensic pathologists are essentially required to estimate the time since death (TSD) to assist in death investigation. The early post-mortem phase is most frequently estimated using the classical triad of post-mortem changes – rigor mortis, algor mortis and livor mortis. 

Algor Mortis: Humans are warm-blooded organisms, which means that we maintain a constant internal temperature, regardless of the outside environment. The brain is our thermostat and the circulatory system is the main heat dissipator. However, within seconds of death, the brain cells begin to die and the heart stops pumping blood. Without the brain and and the blood distributing heat, the corpse eventually starts to match the outside temperature.

Rigor mortis is the post-mortem stiffening of muscles, caused by the depletion of adenosine triphosphate (ATP) from the muscles.

Livor Mortis: This is the final stage of death. When the heart stops beating, the blood is now at the mercy of gravity. It tends to collect at certain parts of the body. Depending on the position of the body, these parts would vary. For instance, if the person was flat on their back when they died, the blood would collect in the parts that are touching the ground (just as in this patient - cadaver spots were on the back and posterior parts of the extremities). If the person was hanging, it would collect in their fingertips, toes, and earlobes. Lividity starts with the skin where the blood has settled developing a bright red color. After a few hours, the color changes from red to bluish-purple. The bluish coloring of the skin is called livor mortis or cadaver spots.

 

Absence of spontaneous breathing, pulselessness or unconsciousness can be as a result of collapse (syncope) and not outright death. Absence of corneal reflexes is very possible and can be diagnosed in patients that are alive but with cranial nerve disorders (especially cranial nerve V and VII).

77. A 40-year-old patient is registered in a narcological dispensary. Somatically: skin is dramatically hyperemic, sclera are injected, hyperhidrosis is present. AP- 140/100 mm Hg, heart rate - 100/min. Mental state: autopsychic orientation is intact, allopsychic orientation is distorted. The patient presents with motor anxiety. There is a look of fear on his face. He refuses to talk about his problems and asks to release him immediately, because he \"may be killed.\" This state developed a day after a regular drinking bout. What is your provisional diagnosis?

Explanation

Delirium tremens also known as Alcohol withdrawal delirium is the most severe form of alcohol withdrawal symptoms. It is characterised by an altered mental status and hyperactivity of the autonomic nervous system. Remember from the question stem, this started a day after one of his drinking bouts. 

Delirium tremens occurs in chronic alcohol abusers who abruptly discontinue alcohol use, often as early as 24 - 48 hours. Alcohol acts as a central nervous system depressant. It enhances the effect of inhibitory neurotransmitters while down-regulating excitatory neurotransmitters. Abrupt cessation of alcohol causes a decrease in the inhibitory actions of GABA neurotransmitter resulting in overactivity of the central nervous system. 

 

The overactivity seen with alcohol withdrawal manifests as altered mental status, hallucinations (“may be killed”), anxiety, seizures, tremors, psychomotor agitation, insomnia etc.

78. A 35-year-old female patient has gained 20 kg weight within a year with the normal diet. She complains of chill, sleepiness, shortness of breath. The patient’s mother and sister are corpulent. Objectively: height - 160 cm, weight - 92 kg, BMI - 35,9. Obesity is uniform, there are no striae. The face is amimic. The skin is dry. The tongue is thickened. Heart sounds are muffled. HR- 56/min, AP- 140/100 mm Hg. The patient has constipations, amenorrhea for 5 months. TSH- 28 mkME/l (normal rate - 0,32-5). Craniogram shows no pathology. What is the etiology of obesity?

Explanation

Your thyroid produces thyroid hormone, which controls many activities in your body, including how fast you burn calories and how fast your heart beats. Diseases of the thyroid cause it to make either too much or too little of the hormone. Depending on how much or how little hormone your thyroid makes, you may often feel restless or tired, or you may lose or gain weight. Women are more likely than men to have thyroid diseases, especially right after pregnancy and after menopause.

Hypothyroidism is when your thyroid does not make enough thyroid hormones. It is also called underactive thyroid. This slows down many of your body's functions, like your metabolism. Signs and symptoms include the following: Feeling cold when other people do not; Constipation; weight gain, even though you are not eating more food; Feeling sad or depressed; Feeling very tired; Pale, dry skin; Dry, thinning hair; Slow heart rate (bradycardia); puffy face;  hoarse voice; abnormal menstrual bleeding etc.

Note that the craniogram shows no pathology so we can rule out Hypothalamic-pituitary. Hypo-ovarian can present with amenorrhea, vaginal dryness, monophasic cycles etc but not with bradycardia or weight gain. Absence of abdominal striaes or skin hyperpigmentation and sometimes diabetes - rules out the option of cushing’s disease (hypercorticoidism).

The increased TSH levels seen in this patient implies an activation of the feedback mechanism signalling  the body to produce more thyroid hormones; T3 and T4.

79. What juice should be included in a complex drug and dietary therapy for patients with gastric ulcer or duodenal ulcer and increased gastric juice acidity in order to accelerate the ulcer healing?

Explanation

 

Potato juice  helps eliminate  gastric acidity thereby, relieving ulcer pain.  Apple juice is rich in vitamin A and is recommended in individuals with poor sight, it also helps in reducing the risk of dementia in old individuals.

80. A 43-year-old alcohol abuser had not consumed alcohol for the last two days. In the evening he claimed to see rats and feel like they bite his feet. The patient is disoriented, agitated, all the time attempts to run somewhere. Specify the psychopathological syndrome:

Explanation

Alcohol Withdrawal

Delirium tremens also known as Alcohol withdrawal delirium is the most severe form of alcohol withdrawal, it is characterised by an altered mental status and hyperactivity of the autonomic nervous system. From the question stem, we observe that this patient is a chronic alcoholic and has been without alcohol for 2 days leading to the described symptoms.

Amentia: this is a rough obscured consciousness with incoherent thought, complete unavailability for contact, isolated deception of perception and signs of sudden physical exhaustion. Patients in amentia generally lie in spite of having chaotic excitements. 

Oneiroid: it is also called dreamlike state and is defined by fantastic psychological suffering. It is characterized by ambivalent/contradictory speech, criminal action, senses global changing, catastrophe, and celebration simultaneously.

 
81. A 67-year-old male patient complains of rash, severe pain in the subscapular region on the right. Objectively: skin in the right subscapular region is covered with linearly arranged pink-red edematous lesions that are somewhat infiltrated, and have clear boundaries. On the lesion surface there are vesicles with transparent exudate. What is the most likely diagnosis?

Explanation

Herpes zoster is a viral disease that is caused by the Varicella-Zoster virus, it is characterized by inflammation of intervertebral or cranial nerves nodes and is shown up by a vesicular rash (with transparent exudate)on a skin along the nerves and symptoms of intoxication. The rash elements are macule - papule - vesicle - crust polymorphism which is often accompanied by hyperthermia. 

Dermatitis herpetiformis (also known as ‘DH’ or Duhring disease) is a rare but persistent blistering skin condition related to coeliac disease. Dermatitis herpetiformis is characterised by extremely itchy bumps (prurigo papules) and blisters (vesicles), which arise on normal or reddened skin. They tend to be distributed symmetrically and are most often found on the scalp, shoulders, buttocks, elbows and knees.

As the blisters are so itchy, they are often immediately scratched, resulting in erosions and crusting. Older lesions may leave pale or dark marks (hypopigmentation and hyperpigmentation).

 
82. A 64-year-old male patient has a 35- year history of chronic pancreatitis. In the last 5 years, he claims to observe the pain abatement, bloating, frequent bowel movements up to 3-4 times a day, grayish, glossy stool with undigested food rests, the progressive loss of body weight. Change of symptoms in the patient is due to overlay of:

Explanation

 

The pancreas plays a major role in digestion ( its exocrine function). It produces digestive enzymes such as; trypsin and chymotrypsinogen ( for protein digestion), Amylase (for carbohydrate digestion), lipase ( for breakdown of fats). In pathologies relating to the pancreas such as pancreatitis, this function is affected leading to a digestion and absorption issue. For example, due to a disruption in the release of lipase, fat breakdown is negatively affected leading to not enough absorption and its appearance in stool ( steatorrhea). Likewise a disruption in chymotrypsin and trypsin secretion will lead to an abnormal excretion of muscle fibres in faeces (creatorrhea).

83. During the doctor’s round, a 56-year-old male patient with decompensated cirrhosis complains of dizziness, palpitations, moving black specks seen before the eyes, general weakness. The patient is pale, Ps- 110/min, AP- 90/50 mm Hg. What complication is most likely to have occurred in the patient?

Explanation

 

Usually seen in individuals with serious liver diseases, Esophageal varices refers to enlarged or swollen veins on the lining of the esophagus. Most likely complications are severe internal bleeding, hypovolemic shock and cardiac arrest. Notice that this patient has  decompensated cirrhosis of the liver ( the cause of these varices); an absence of neurological signs rules out the option of hepatic encephalopathy.

84. A  57-year-old female patient complains of intense pain in the right hypochondrium irradiating to the right supraclavicular region. Skin and sclerae are icteric. There is tension and tenderness in the right hypochondrium on palpation. Body temperature is 38,8oC. Blood test results: WBC- 11, 2 · 109 /l, total bilirubin - 112 mmol/l (conjugated - 86 mmol/l, unconjugated - 26 mmol/l). What is the most likely diagnosis?

Explanation

 

Intense pain in the right hypochondrium should signal the presence of a pathology in relation to the liver or gallbladder. This can be further proven by the presence of  icteric skin and sclera ( due to the deposition of bilirubin on the skin and sclera).  The results from palpation ( tenderness in right hypochondrium) and biochemical analysis ( increased bilirubin levels) helps confirm this. Cholangitis is an inflammation of the biliary system mostly as a result of bacterial infection. Of all the listed options, only cholangitis is in relation to gallbladder or liver pathology.

85. Examination of a 35-year-old patient with rheumatism revealed that the right heart border was 1 cm displaced outwards from the right parasternal line, the upper border was on the level with inferior margin of the 1st rib, the left border was 1 cm in from the left midclavicular line. Auscultation revealed atrial fibrillation, loud apical first sound, diastolic shock above the pulmonary artery. Echocardioscopy revealed abnormal pattern of the mitral valve motion. What heart disease is characterized by these symptoms?

Explanation

Acute rheumatic fever may be complicated by progressive, permanent damage to the heart valves (especially the mitral valve), resulting in chronic rheumatic heart disease. Note that Diastolic murmurs are usually heard in either mitral/tricuspid valve stenosis or Aortic/Pulmonary regurgitation. This patient presents with a diastolic murmur and this helps narrow down our differentials which can be confirmed  by the results from echocardioscopy - ‘ abnormal pattern of mitral valve motion’ Mitral/Pulmonary Regurgitation or insufficiency and Aortic/Pulmonary Stenosis will produce a systolic murmur. Mitral valve prolapse also produces a systolic murmur.
86. A 54-year-old male patient works as an engineer. At the age of 35, he got infected with syphilis and treated it with \"traditional remedies\". About 5 years ago, he became forgetful, unable to cope with work, told cynical jokes, bought useless things, collected cigarette butts in the street. Objectively: the patient is indifferent, has slow speech, dysarthria, can make only primitive judgments, is unable to perform simple arithmetic operations or explain simple metaphors. The patient is untidy, takes no interest in anything, passive. Considers himself to be completely healthy. Qualify mental condition of the patient:

Explanation

This is a classical case of Neurosyphilis. Neurosyphilis is a bacterial ( syphilitic) infection of the brain and spinal cord; it occurs on the background of a chronic syphilitic infection (notice that this patient has a long history of syphilis infection not properly treated). 

Dementia is a brain related disorder characterized by progressive impairment of memory, thinking and behaviour that affects one’s everyday activities ( a decline in one’s mental and cognitive function that does not correspond to the age of the patient).

Lacunar dementia is also known as Binswanger disorder or Subcortical arteriosclerotic encephalopathy. Typically presents with Gait, urinary incontinence, parkinsonian features, pseudobulbar palsy etc.

Korsakoff’s ( amnestic ) syndrome is characterised by thiamine deficiency ( beri-beri) on the background of chronic alcoholism.

In Somnolentia, the individual experiences incomplete sleep/ drowsiness which resembles a state of drunkenness. 

 
87. 5 days before, a 26-year-old female patient developed an acute condition. Objectively: marked headache, vomiting, weakness, poor appetite, temperature up to 39oC. Objectively: the patient is in a moderately grave condition, excited. The face is hyperemic, sclerae are injected. The tongue is coated with brown fur. The trunk and limbs are covered with plentiful roseolous and petechial rash. Hepatosplenomegaly is present. Complement binding reaction with Rickettsia prowazekii is positive with the titer of 1:640. What drug should be administered?

Explanation

 

The results from the complement binding reaction is positive for Rickettsia Prowazekii which is the causative agent for Epidemic typhus. Epidemic typhus is an infectious disease transmitted via Body lice ( note that other variants of the typhus disease can be transmitted by fleas). It is characterised by fever,  petechial rashes  which begins on the trunk and spreads to the extremities, and flu-like symptoms. Note that for treatment, Doxycycline is the drug of choice.

88. A 39-year-old female patient complains of dyspnea when walking, palpitation, edemata in the evening. The patient’s height is 164 cm, weight - 104 kg. Objectively: overnutrition. Heart sounds are weak, and tachycardia is present. The menstrual cycle is not broken. Blood sugar is 5,6 mmol/l, ACTH-response tests revealed no alterations. X-ray of the Turkish saddle revealed no pathology. What disease is it?

Explanation

Notice that on X Ray, no pathology was found in the Turkish saddle - sella turcica (the location of the Pituitary gland on the sphenoid bone) - this finding rules out the option of Pituitary obesity. The blood glucose level is within normal range  and the adrenocorticotropic hormone revealed no alteration- these rule out the options of diabetes mellitus and cushing’s syndrome. Alimentary obesity occurs as a result of an increased intake of food (overnutrition). Overnutrition is the major criteria in putting the diagnosis of this  patient.
89. A 26-year-old male patient complains of a rash on the upper lip skin, which arose on a background of influenza with high-grade fever and is accompanied by pain and burning. The rash has been present for 3 days. Objectively: the skin of the upper lip is edematic and erythematous, grouped vesicles are filled with serous fluid and have a rough surface. What is the most likely diagnosis?

Explanation

 Herpetic vesicular dermatitis (herpes simplex dermatitis)  occurs as a result of an infection by HSV 1 ( human simplex virus 1) and is characterised by the presence of grouped vesicles ( blisters), erythematous in nature. These blisters are usually very painful and itchy. Notice that; From objective examination of this patient, erythematous grouped vesicles are found.

Dermatitis herpetiformis (duhring’s disease) is an autoimmune skin disease that targets those with gluten intolerance or celiac disease. People with celiac disease are unable to digest the gluten protein that is present in grains such as wheat, rye and barley. Itchy blisters and hives will form commonly on a person’s back or buttocks; this disorder is handled by removing all gluten containing products from a person\'s lifestyle. People with hypothyroidism, Sjogren’s disease or rheumatoid arthritis are also known to get this skin condition.

90. A 36-year-old female has a 7-year history of pollen allergy. Over the last 2 years in August and September (during ragweed flowering), the patient has had 2-3 asthma attacks that could be treated with one dose of salbutamol. Objectively: body temperature - 36, 5o C , respiratory rate - 18/min, Ps- 78/min, AP- 115/70 mm Hg. There is vesicular breathing above the lungs. Cardiac sounds are sonorous, of regular rhythm. What drug would be most effective to prevent asthma attacks during the critical season for the patient?

Explanation

This patient has a history of allergy (asthmatic attacks); recall that an allergy is a type 1 hypersensitivity disorder - B cells are stimulated by  CD4+TH2 cells which leads to the production of IgE antibodies against the trigger antigen. Recall that these IgE antibodies bind to specific receptors on mast cells and basophils. On further exposure to the same trigger ( allergen), these mast cells release mediators ( histamine, prostaglandins, leukotrienes etc). These mediators are responsible for the changes observed in allergic attacks.

Intalum inhalation (Cromolyn sodium) is a mast cell stabilizer. They block the early and late responses in asthmatic attacks, control inflammation in asthma and are used for prophylaxis in allergic asthma.

Berotec Inhalation (fenoterol) -β2 agonist; Atrovent Inhalation - Ipratropium (muscarinic antagonist).

91. A 25-year-old female patient complains of marked weakness, sleepiness, blackouts, di- zziness, taste disorder. The patient has a hi- story of menorrhagia. Objectively: the pati- ent has marked weakness, pale skin, cracks in the corners of mouth, peeling nails, systolic apical murmur. Blood test results: RBC - 3, 4 · 1012 /l, Hb- 70 g/l, color index - 0,75, platelets - 140 · 109 /l, WBC- 6, 2 · 109 /l. What is the most likely diagnosis?

Explanation

From anamnesis, we see that the patient has a history of menorrhagia; the current state of the patient (pale skin, fatigue weakness etc) is as a result of the amount of blood already lost. A post hemorrhagic anemia is a condition in which a person loses lots of hemoglobin and RBCs due to blood loss. It is usually seen in cases of trauma, severe injuries etc. A chronic form is seen in patients with moderate or prolonged occurrence of bleeding eg, gastric ulcers, menorrhagia, hemophilia etc while an acute form is seen in short term events such as trauma.
92. A 49-year-old male patient who had been scheduled for a surgery for gastric cancer underwent preoperative infusion therapy. Up to 3,0 liters of liquid was introduced into the right cubital vein. The following day, he felt a dragging pain in the right shoulder. Objectively: on the inner surface of the shoulder there is a longitudinal hyperemic zone, edema of skin, a tender cord. What complication occurred in the patient?

Explanation

Acute thrombophlebitis refers to the inflammation of a vein due to the formation of a blood clot (thrombus). Key findings include; pain, hyperemia , edema, tenderness and Veins being hard and cord-like

Paravenous tissue necrosis is ruled out due to the absence of necrosis on examination; a phlegmon refers to the inflammation of soft tissue due to an infection and is characterised by the production of pus ( this helps rule out tissue phlegmon). 

 
93. A 36-year-old female patient complains of intense pain in the knee joints and neck. In the morning she experiences pain in the interscapular region and leg joints; pain subsides after warm-up gymnastics. The patient is overnourished, there is a clicking sound in the knees when squatting, the knees are somewhat disfigured, painful on palpation. Blood test results: ESR- 18 mm/h, WBC- 8,0 · 109 /l. Radiography reveals subchondral sclerosis in the left knee. What is the basis of this pathology?

Explanation

Subchondral sclerosis is a condition common in people with osteoarthritis. It refers to the hardening/ stiffening of the bone just below the cartilage. Note that “Chondra” is another word for cartilage, so subchondral means “beneath the cartilage.” “Sclerosis” means hardening. Subchondral sclerosis appears in the later stages of osteoarthritis, when there’s a degeneration of cartilage.

Deposition of urates (tophi)in the articular tissues will be seen in Gout while Hemarthroses refers to bleeding into the joint cavity.

 
94. A 35-year-old female reports heart pain (aching and drilling) occurring mainly in the morning in autumn and spring and irradiating to the neck, back and abdomen; rapid heartbeat; low vitality. Occurrence of this condition is not associated with physical activity. In the evening, the patient’s condition improves. Study of somatic and neurological status, and ECG reveal no pathology. What pathology is most likely to have caused these clinical presentations?

Explanation

Somatoform disorders - multiple, recurrent and frequent somatic complaints requiring medical attention without association with any physical disorder. The above patient’s heart ache is not related to any physical activity and occurs at a particular period of the year.

Hypochondriacal Disorder is characterized by a persistent preoccupation and a fear of developing or having one or more serious and progressive physical disorders. Physician physical examination does not reveal any disorder, but the fear and convictions persist despite the reassurance.

 
95. A 33-year-old female complains of escalating spastic pain in the abdomen after the psycho-emotional stress. The patient has intermittent bowel movements, that is 2-3 bowel movements after waking up alternate with constipation lasting for 1-2 days. Objectively: body weight is unchanged, there is moderate pain on palpation of the sigmoid colon. Hb- 130 g/l, WBC- 5,2 · 109/l, ESR- 9 mm/h. Proctosigmoidoscopy causes pain due to spastic bowel condition, intestinal mucosa is not changed. In the lumen there is a lot of mucus. What is the most likely diagnosis?

Explanation

One major difference between Irritable bowel syndrome  and inflammatory bowel disease is that intestinal mucosa is not affected in IBS while IBDs (Crohn’s disease and ulcerative colitis) have their mucosa affected.

Irritable bowel syndrome is an inflammatory bowel disease characterised by recurrent abdominal cramps/spasms (pain), change in form (consistency) and frequency of stool, constipation etc. . This condition is common in middle aged women and is associated with underlying conditions such as stress, anxiety, depression or a previous case of intestinal infection. 

Notice that in this patient, on endoscopic examination, painful spasm occurred while intestinal mucosa is not changed- this is indicative of Irritable bowel syndrome. 

In malabsorption syndrome, the small intestine is unable to absorb certain nutrients eg carbohydrates, proteins etc. can be caused by certain diseases such as celiac disease, lactose intolerance etc. 

 
96. After a holiday in the Crimea, a 36-year-old female patient presents with severe pain in the elbow joints, dyspnea and weakness. The body temperature is of 37,6*C, the skin is pale, there is erythema of cheeks and nose, lower lip ulceration. Visual inspection reveals no changes in the joints, the right elbow movement is limited. There is murmur and pleural friction in the lungs below the right angle of the scapula. Cardiac sounds are muffled, there is tachycardia, gallop rhythm, Ps- 114/min. AP- 100/60. What is the most likely diagnosis?

Explanation

The key finding here is the erythema on the cheeks and bridge of nose which describes a malar/ butterfly rash classically seen in patients with Systemic lupus erythematosus.

SLE is a chronic inflammatory disease of unknown cause which can affect the skin, joints, kidneys, lungs, nervous system, serous membranes and/or other organs of the body. Distinct immunologic abnormalities, especially the production of a number of antinuclear antibodies, other signs include, arthralgia, myalgia, nephritis, systolic murmurs in about 30% of patients, dry pleurisy in the lungs etc. The dry pleurisy is characterised by an abnormal sound on auscultation called a pleural friction rub

Rheumatic carditis and arthritis will be observe in a case of rheumatic fever ( prior infection by group A beta hemolytic streptococci infection)

 
97. A 63-year-old male patient complains of cough with expectoration of mucous blood- streaked sputum, asthma, low-grade fever, general weakness. These presentations have been observed for 3 months. The patient has been a smoker since childhood. Objecti- vely: to- 37,4oC, respiratory rate is 26/min, Ps- 82/min, rhythmic. AP- 130/85 mm Hg. There is limited breathing movement in the right side of chest cavity, as well as percussive dullness and diminished breath sounds. Radi- ograph shows a homogeneous opacity of the pulmonary field on the right with the mediasti- num displacement to the affected side. What is the most likely diagnosis?

Explanation

98. A 33-year-old male patient developed a condition that had a stormy clinical course: chills, fever up to 39oC, vomiting, epigastric pain, diarrhea with watery smelly feces. 6 hours before, he ate a raw egg, fried potatoes with stewed meat, drank some juice. What pathogen is likely to have caused this conditi- on?

Explanation

99. A 76-year-old male consulted a therapist about slow discharge of urine with a small jet. The patient reported no cardiac problems. Examination revealed atrial fibrillation with a heart rate of 72/min and without pulse deficit. There are no signs of heart failure. ECG confirms the presence of atrial fibrillation. From history we know that the arrhythmia was detected three years ago. What tactics for the treatment of atrial fibrillation in the patient should be chosen?

Explanation

 

Note that from anamnesis, the patient reported no cardiac problems. The heart rate revealed 72 beats per minute without pulse deficits - this finding debunks the claim of a clinically significant atrial fibrillation. Also take note that there are no signs of Heart failure therefore, this patient does not require any form of treatment.

100. A 53-year-old male has been admitted to a hospital for an attack of renal colic which has repeatedly occurred throughout the year. Objectively: in the region of auricles and the right elbow some nodules can be seen that are covered with thin shiny skin. Ps- 88/min, AP- 170/100 mm Hg. There is bilateral costovertebral angle tenderness (positive Pasternatsky’s symptom). The patient has been scheduled for examination. What laboratory value would be most helpful for making a diagnosis?

Explanation

This patient has a history of repeated renal colic over the past one year. Renal colic refers to a form of abdominal pain that occurs as a result of urinary stones blocking the urinary tract ( bladders, ureters, urethra). This kidney pathology can be supported by a positive pasternetsky’s sign ( costovertebral angle tenderness). One basic component excreted by the kidneys via urine is uric acid. Due to the blockage of the urinary tract, these products are not excreted and remain accumulated in the blood system. Amongst the listed options,  An elevated uric acid level is the most helpful in putting a diagnosis. 

Elevated rheumatoid factor is helpful in the diagnosis of autoimmune conditions such as rheumatoid arthritis, Sjogren's syndrome etc.

 
101. A 58-year-old female patient complains of spontaneous bruises, weakness, bleeding gums, dizziness. Objectively: the mucous membranes and skin are pale with numerous hemorrhages of various time of origin. Lymph nodes are not enlarged. Ps- 100/min, AP- 110/70 mm Hg. There are no changes of internal organs. Blood test results: RBC - 3,0·1012/l, Нb - 92 g/l, colour index - 0,9, anisocytosis, poikilocytosis, WBC - 10·109/l, eosinophils - 2%, stab neutrophils - 12%, segmented neutrophils - 68%, lymphocytes - 11%, monocytes - 7%, ESR - 12 mm/h. What laboratory test is to be determined next for making a diagnosis?

Explanation

 

Since the patient complains about spontaneous bruises, bleeding gums, numerous hemorrhages of various time of origin, the blood platelet level should be checked, as it was not given in the blood count. Platelets/ thrombocytes help in blood clot formation.

102. A 48-year-old male in-patient undergoes treatment for essential hypertension of II- B stage. It is known from history that he works in a design engineering office. His job involves neuro-emotional stress. Which of these foodstuffs do not stimulate the central nervous system and can be recommended for the patient?

Explanation

103. A 48-year-old male patient complains of pain in the lower extremities, especially when walking, intermittent claudication, numbness in the fingers, cold extremities, inability to walk more than 100 meters. Sleeps with his leg lowered. The patient has been a smoker since he was 16, abuses alcohol, has a history of CHD. The left lower extremity is colder than the right one, the skin of extremities is dry, pedal pulse cannot be palpated, femoral pulse is preserved. What is the most likely diagnosis?

Explanation

Obliterating Endarteritis is also known as Buerger’s disease and vasculitis. The condition is thought to occur as a result of tobacco use and more common in males. The disease is characterized by inflammation and thrombosis of small and medium arteries of the legs and feet which recurs and progresses in stages. During the ischemic period, the veins of the lower leg are inflamed, skin has ulceration, and there is gangrene with intermittent claudication. During the nutritional disorder period there is persistent pain and muscle atrophy. During the necrosis period, the pain becomes worse as infection leads to the development of ulcers and gangrene. Pains usually persist; In addition, the patient usually complains of coldness and numbness. Burning sensations is another disturbing symptom.

Treatment usually has to do with smoking cessation.

Raynaud disease is a condition in which some areas of the body feel numb and cool in certain circumstances. In Raynaud's phenomenon, smaller arteries that supply blood to the skin constrict excessively in response to cold, limiting blood supply to the affected area. The fingers, toes, ears and tip of the nose are commonly involved and feel numb and cool in response to cold temperatures or stress.

Leriche Syndrome, also commonly referred to as aortoiliac occlusive disease, is a product of atherosclerosis affecting the distal abdominal aorta, iliac arteries, and femoral popliteal vessels. Its triad include; claudication, erectile dysfunction and decreased distal pulse.

 
104. A 38-year-old male complains of tonic tension of the masticatory muscles, so that he cannot open his mouth. 12 days before, he was bitten by an unknown dog. Objectively: there is pronounced tension and twitching of the masticatory muscles. What is the most likely diagnosis?

Explanation

Notice that this patient has tonic tension of the masticatory muscles and cannot open the mouth (TRISMUS); tension and twitching of the muscles (Tetany). These two findings occurred in the background of an unknown dog bite which helps indicate a Tetanus infection (lockjaw).

Tetanus (lockjaw) is an acute disease caused by neurotoxins (tetanospasmin) from the bacterium Clostridium tetani. C. tetani enters the body through broken skin (e.g., deep puncture wounds).

Rabies is a neurotropic virus contracted from the bite of an infected animal. The virus enters the patient's skin from the saliva of the animal and migrates along the peripheral nerves to the central nervous system (CNS). An incubation period of 4–12 weeks typically precedes the clinical appearance of the disease, which manifests with a prodrome of nonspecific flu-like symptoms, followed by acute rabies encephalitis. Clinical findings include fever, hydrophobia, hypersalivation, and stupor alternating with mania. 

 
105. A 72-year-old male had had a moderate headache. Two days later, he developed the progressing speech disorders and weakness in the right extremities. The patient has a history of myocardial infarction, arrhythmia. Study of the neurologic status revealed elements of motor aphasia, central paresis of the VII and XII cranial nerves on the right, central hemiparesis on the same side and hyperaesthesia. What is the most likely diagnosis?

Explanation

Ischemic stroke: cerebral infarction due to insufficient cerebral blood flow (hypoperfusion), which results in ischemia and neuronal injury. Commonly associated with Atherosclerosis (patient has a history of Myocardial Infarction).

Transient ischemic attack: temporary, focal cerebral ischemia; that results in neurologic deficits without acute infarction or permanent loss of function (previously defined as lasting < 24 hours).

Hemorrhagic stroke: cerebral infarction due to hemorrhage. Commonly associated with elevated blood pressure and a severe headache ("worst headache of my life").

 
106. 2 weeks after having quinsy, a 26-year-old male patient got facial edemata, moderate pain in the sacrum. Objectively: body temperature is 37.5*C, AP- 100/80 mm Hg. Urinalysis results: RBC- up to 100 fresh cells in per HPF, protein - 2,2 g/l, hyaline cylinders - up to 10 per HPF, relative density - 1002. What is the most likely diagnosis?

Explanation

Quinsy: Peritonsillar abscess which occurs due to  Bacterial infection (mostly streptococcal).

Glomerulonephritis are a group of disorders where the damage to the glomerular filtration apparatus causes a leak of protein +/- blood into the urine, depending on the form. Patients may be asymptomatic or present with hematuria ( red cell casts), proteinuria, edema or hypertension.  Acute glomerulonephritis has a sudden onset while the chronic form has a gradual onset. Note that these changes occurred after the infection two weeks ago. 

Pyelonephritis refers to the bacterial infection of the renal pelvis. Symptoms include fever, oliguria, vomiting, pain. 

Urolithiasis refers to the lodging of stone in the urinary tract.

 
107. A 27-year-old patient has a severe headache, nausea and vomiting. Objectively: body temperature is 38.9*C, there is a haemorrhagic stellate rash on the legs. The patient takes meningeal pose in bed. Meningeal symptoms are strongly positive. Deep reflexes are brisk, uniform. Pathological reflexes are absent. It has been suspected that the patient has epidemic cerebrospinal meningitis. Which of additional tests should be performed in the first place to verify the diagnosis?

Explanation

It is well noted that on examination, the patient presents with Meningeal Symptoms (stiff neck, sensitivity to light etc) with the suspicion of cerebrospinal meningitis. To understand the cause of these meningeal signs and prove the diagnosis, a lumbar puncture is carried out in order to obtain the cerebrospinal fluid for analysis.
108. 3 hours before, a 68-year-old male pati- ent got a searing chest pain radiating to the neck and left forearm, escalating dyspnea. Ni- troglycerin failed to relieve pain but somewhat reduced dyspnea. Objectively: there is cri- mson cyanosis of face. Respiratory rate is 28/min. The patient has vesicular breathing with isolated sibilant rales. Heart sounds are muffled, with a gallop rhythm. Ps- 100/min, AP- 100/65 mm Hg. ECG shows negative T- wave in V 2 − V 6 leads. What drug can reduce the heart’s need for oxygen without aggravati- ng the disease?

Explanation

109. Two years ago, a 46-year-old patient was diagnosed with stage I silicosis. Currently the patient complains of escalating dyspnea, pain in the infrascapular regions. Radiograph shows a diffuse enhancement and distorti- on of lung markings, as well as multiple nodular shadows 2-4 mm in diameter. There is interlobar pleural density on the right. Dense shadows are found in the hilar regions. Specify the form of radiographic pulmonary fibrosis in this case:

Explanation

110. A 63-year-old female complains of general weakness, a feeling of heaviness, compression in the epigastrium, postprandial fullness, nausea, belching after meals. These symptoms have been observed for about 15 years. Objectively: body temperature is 36,4*C , respiratory rate - 20/min, Ps - 88/min, blood pressure - 115/75 mm Hg. Skin and mucous membranes are pale. Blood test results: RBC - 2,0·1012/l,Hb-100g/l. Tests revealed parietal- cell antibodies. What is the most likely reason for the development of anemia in this patient?

Explanation

 

Histologically, the stomach is made up of parietal and chief cells. The parietal cells secrete Hydrochloric acid responsible for the stomach’s acidity (intrinsic factor) which helps in digestion and Vitamin B12 absorption. Gastric chief cells secrete Pepsinogen which is converted into pepsin and helps in protein digestion. Notice that  the RBC level is low and the test result revealed parietal cell antibodies which helps suggest that this patient has Vitamin B12 anemia (pernicious anemia). As earlier stated, Gastric acidity (intrinsic factor) is necessary for the absorption of vitamin B12, its absence due to parietal cells destruction by antibodies will lead to vitamin B12 deficiency. 

111. During dressing of a poorly-granulating wound Pseudomonas aeruginosa infection was revealed. What medication would be optimal for the wound d-bridement?

Explanation

Boric acid Medical uses include; 

 
  • antiseptic in burns, cuts and dressing.

  • Vaginal douche in bacterial vaginosis and candidiasis.

  • Prevention of athlete’s foot, otitis externa.

112. A 30-year-old female patient has been delivered to a hospital for sudden dyspnea progessing to asthma, sensation of having a \"lump in the throat\", hand tremor, fear of death. The attack has developed for the first time and is associated with a strong emotion. There is no previous history. Objectvely: respiratory rate - 28/min, Ps- 104/min, rhythmic, AP- 150/85 mm Hg. The patient has rapid superficial vesicular breathing with extended expiration. Percussion findings: heart borders are not changed. Cardiac sounds are loud, rhythmic. What is the most likely diagnosis?

Explanation

Neurocirculatory asthenia is a circulatory and a form of anxiety disorder that is characterised by breathing difficulties, heart palpitations,shortness of breath, dizziness and insomnia on the bases of neurological or psychiatric disturbance. In this case, let's try to rule out all other options. Bronchial Asthma will present earlier, most likely during childhood. It's not going to present for the first time at 30 years of age. Cardiac asthma often result as a consequence of left heart failure leading to pulmonary congestion and the patient will present with other symptoms like cough, inability to lie flat on bed at night etc. Thyrotoxic crisis will present with other symptoms of hyperthyroidism like heat intolerance, weight loss, etc. Hypertensive crisis will likely not present with a sensation of lump in the throat and we might expect a higher blood pressure up to like 180/120mmHg.  
113. A 42-year-old male patient wth essential hypertension presents with headache, palpitations, unexplained fear. Objectively: Ps- 100/min, AP- 200/100 mm Hg, the left border of cardiac dullness is displaced by 1,5 cm to the left, vesicular breathing is present. ECG shows sinus tachycardia, signs of left ventricular hypertrophy. What drug should be administered as an emergency?

Explanation

Obzidan is also known as Propranolol, inderal is a drug indicated for the treatment of high blood pressure, a number of types of irregular heart rate, thyrotoxicosis, capillary hemangiomas, performance anxiety, and essential tremors. It is classified as a beta blocker (non- selective);  weak indirect α1-adrenoceptor agonist in addition to potent β-adrenoceptor antagonist.

Reserpine is an adrenergic blocking agent used to treat mild to moderate hypertension via the disruption of norepinephrine vesicular storage. The antihypertensive actions of Reserpine are a result of its ability to deplete catecholamines from peripheral sympathetic nerve endings. These substances are normally involved in controlling heart rate, force of cardiac contraction and peripheral resistance. 

114. A 37-year-old male patient has a history of diabetes of moderate severity. On the left side of face the patient has a carbuncle. What severe complication might have occurred in the patient?

Explanation

 

Cavernous sinus thrombosis is usually a late complication of an infection of the central face or paranasal sinuses. In cavernous sinus thrombosis, a blood clot blocks a vein that runs through a hollow space underneath the brain and behind the eye sockets ( the cavernous sinus). Notice that this patient has a carbuncle located on the side of the face; this infection is most likely to spread via the sinuses. Cavernous sinus thrombosis can be a complication of this infectious process.

Cavernous Sinus Facial Vein Anastomosis

115. A 42-year-old female patient complains of a dull pain in her left side, low-grade fever, accelerated painful urination in small portions. These presentations have been observed for three years. For a long time, the patient has had cystitis with frequent exacerbations, there is pulmonary tuberculosis in the past history. Urinalysis results: microscopic hematuria, leukocyturia. What is the most likely provisional diagnosis?

Explanation

Notice that this patient has a history of pulmonary tuberculosis. Renal tuberculosis occurs due to hematogenous spread of Mycobacterium Tuberculosis to the kidneys. It is the  second most frequent kind of extrapulmonary tuberculosis. Key findings include increased frequency, dysuria ( painful urination), back pain, hematuria ( blood in urine) etc. 

Urolithiasis refers to stones in the urinary tract. Dysuria, hematuria are also found in this condition. 

Glomerulonephritis are a group of disorders where the damage to the glomerular filtration apparatus causes a leak of protein +/- blood into the urine, depending on the form. Patients may be asymptomatic or present with hematuria ( red cell casts), proteinuria, edema or hypertension.  The chronic form has a gradual onset.

In Chronic cystitis ( inflammation of the bladder), there will be a burning sensation in the pelvic region with the frequent urge to urinate.

 
116. A 65-year-old male patient complains of dyspnea that is getting worse with exertion, morning cough with expectoration of mucous sputum. For about 15 years, he has been subject to regular medical check-up for chronic bronchitis. The patient takes berodual (16 inhaled doses per day). Objectively: body temperature is 36, 8o C , RR- 24/min, Ps- 110/min, AP- 145/90 mm Hg. Auscultation reveals a lot of dry rales above the lungs. FEV1- 65%. What is the optimal tactics of further management of the patient?

Explanation

 

From the complaints of this patient ( dyspnea on exertion, cough accompanied by mucus production etc) we understand that this patient has been having exacerbations of COPD ( chronic obstructive pulmonary disease) - Chronic bronchitis. Berodual ( Ipratropium Bromide) is a Short acting Muscarinic agent ( anticholinergic drug) with a bronchodilatory effect and is one of the drugs indicated for the management of COPDs. Note that for The management of acute exacerbation of COPDs, an Inhaled Corticosteroid plus a Long acting Beta 2 Agonist should be administered. 

117. A 49-year-old countryman got an itching papule on the dorsum of his right hand. In the centre there is a vesicle with serosanginous exudate. Within the next 2 days the patient developed a painless edema of hand and forearm. On the 4th day the temperature rose to 38,5*C, in the right axillary region a large painful lymph node was found. One day before the onset of the disease the patient had examined a dead calf. What is the most likely diagnosis?

Explanation

Bacillus anthracis is an aerobic, non-motile, spore forming, large non-hemolytic Gram-positive rod that grows well on blood agar. The clinical manifestations of human anthrax - Cutaneous and Inhalation. 

Cutaneous anthrax, the most common form of naturally occurring disease, begins as a small, painless, pruritic papule that within 2 days enlarges, develops vesicles, and ulcerates to form an eschar. Inhalational anthrax follows inhalation of infectious doses of anthrax spores. 

When anthrax spores get into the skin, usually through a cut or scrape, a person can develop cutaneous anthrax. Human cases may follow contact with contaminated carcasses (dead calf) or animal products. This can happen when a person handles infected animals or contaminated animal products like wool, hides, or hair. Cutaneous anthrax is most common on the head, neck, forearms, and hands. It affects the skin and tissue around the site of infection.

Bubonic plague is caused by bacteria called Yersinia pestis. It’s usually spread by fleas. These bugs pick up the germs when they bite infected animals like rats, mice, or squirrels.

 
118. A 38-year-old female suddenly developed acute inflammatory rash in form of roseolas, papules, vesicles that are scattered on the skin of trunk in irregular and predominantly focal manner. The rash appeared a few hours after visiting a restaurant. The patient complains of itching skin. What is the most likely diagnosis?

Explanation

Toxicodermatosis is a disease of the skin caused by a poison or by an allergen. The causative agents may be such chemical substances as antibiotics, sulfanilamides, and bromine, iodine, arsenic, and mercury compounds, or such food products as berries, cheese, or fish. As described in this scenario, the rash appeared after visiting a restaurant. Toxicoderma can also occur as a metabolic disorder or an occupational disease.

The causative agent enters the body through the respiratory organs or the digestive tract, during injections, or by direct contact with the skin. The primary manifestation of toxicoderma is a rash, which is generally symmetrical; it can take the form of inflamed or hemorrhagic maculae, papule, or vesicles (blisters). In severe cases, widespread areas of the skin are affected (erythroderma) and the patient manifests fever, cardiovascular disorders, or symptoms of anaphylactic shock.

With the removal of the allergen the symptoms of toxicoderma rapidly disappear, although relapses (usually severe) may occur with subsequent repeated exposure to the causative agent. Treatment consists in avoiding contact with the suspected allergen or toxic agent, as well as in the use of desensitizing agents and of expectant treatment.

Atopic dermatitis is a chronic condition that requires symptom management. It’s characterized by an itchy, red rash that usually appears at joints in your body, such as knees or elbows, and even around the neck. This condition occurs in flare-ups or bouts, meaning that it gets worse and improves in irregular cycles.

Many doctors use the terms “atopic dermatitis” and “eczema” interchangeably. They are both generic terms for “inflammation of the skin.” Eczema usually starts in early infancy, but also affects a substantial number of adults. It is commonly associated with elevated levels of immunoglobulin E (IgE) and a series of allergic diseases—including food allergy, asthma, and allergic rhinitis.

Contact Dermatitis occurs when your skin has a reaction to something it’s come in contact with. This can include bleach, soap, poison ivy, certain metals, or other irritants. The rash is typically red and may itch or burn.

 
119. After lifting a load, a 36-year-old male patient has experienced a severe pain in the lumbar region, which spread to the right leg and was getting worse when he moved his foot or coughed. Objectively: the long back muscles on the right are strained. Achilles jerk is reduced on the right. There is a pronounced tenderness of paravertebral points in the lumbar region. The straight leg raise (Lasegue’s sign) is positive on the right. What additional tests should be performed in the first place?

Explanation

 

A positive Lasegue’s sign - Straight leg raise test refers to the sensation of pain by applying pressure to the piriformis muscle and its tendons , particularly when the hip is flexed at the angle of 90° and the knee extended. A positive straight leg raise test usually indicates S1 or L5 root irritation. To support this diagnosis, a radiography of the spinal column should be carried out in the first place.

120. A 26-year-old female patient has an 11-year history of rheumatism. Four years ago she suffered 2 rheumatic attacks. Over the last 6 months there have been paroxysms of atrial fibrillation every 2-3 months. What option of antiarrhythmic therapy or tactics should be proposed?

Explanation

The question is asking for the best antiarrhythmic medication or tactic to be employed in managing this patient’s atrial fibrillation.

Cordarone is in the same class as Amiodarone, an antiarrhythmic drug Class III - Potassium channel blocker. Other medications in this class include Ibutilide, Sotalol, Dofetilide. Indications - Atrial fibrillation, atrial flutter, ventricular tachycardia.

Lidocaine is an antiarrhythmic drug Class IB - Sodium channel blocker used to treat Acute ventricular arrhythmias especially Post Myocardial Infarction.

Defibrillation is commonly used in cases of ventricular fibrillation or pulseless ventricular tachycardia.

Heparin is an anticoagulant. It is not an antiarrhythmic medication.

 
121. On the second day of the disease a 22- year-old male patient complains of high-grade fever, headache in the region of forehead and superciliary arches, and during eye movement; aching muscles and joints. Objectively: body temperature is 39o C . Face is hyperemic, sclerae are injected. The mucous membrane of the soft palate and posterior pharyngeal wall is bright hyperemic and has petechial hemorrhages. What changes in the hemogram are typical for this disease?

Explanation

122. A 44-year-old male patient complains of severe non-localized abdominal pain, pain in the right shoulder girdle, repeated vomiting, red urine. The onset of the disease is associated with alcohol consumption. The face is hyperemic. AP- 70/40 mm Hg. Abdominal radiography reveals no pathological shadows. Hemodiastase is 54 mg/h/l. Prothrombin is 46%. What is the provisional diagnosis?

Explanation

Acute Pancreatitis refers to the inflammation of the pancreas usually caused by alcohol consumption or biliary tract disorders. Acute pancreatitis usually presents with epigastric pain radiating to the back, nausea and vomiting, and epigastric tenderness on palpation. These signs are accompanied by the release of proteolytic enzymes from the pancreas that cause autodigestion. Notice that the patient’s diastase level is high, diastase refers to groups of enzymes that catalyze the breakdown of starch to maltose; these enzymes include α,β and γ amylase (especially α). The diastase level is useful in the diagnosing of diseases such as pancreatitis, jaundice, gall stones etc.

In Acute myocardial infarction Biomarkers such as Troponin, Creatinine phosphokinase (MB isoform) and myoglobins will be elevated on biochemical analysis.

 
123. A 41-year-old male patient was delivered to a hospital unconscious. During the previous 7 days he had been taking large doses of biseptolum for a cold. The night before, he began complaining of dyspnea, especially when lying down, swollen legs, 2-day urinary retention. In the morning he had seizures and lost consciousness. Objectively: noisy breathing at the rate of 30/min, edematous legs and lumbar region, Ps- 50/min. Plasma creatinine is 0,586mmol/l,plasmapotassium-7,2mmol/l. What treatment is necessary for this patient?

Explanation

 

This Patient presents with Acute Kidney Failure which is defined as a significant reduction in renal function occurring over hours or days clinically manifesting as an abrupt and sustained rise in serum urea and creatinine. It might be asymptomatic but oliguria is common. Life threatening consequences include volume overload ( presents as edema), increased potassium levels and metabolic acidosis. Note that plasma potassium levels > 7 mmol/l is an indication for Dialysis. Hemodialysis is a procedure used for removal of waste materials from the blood through filtration.

124. A 45-year-old male patient with acute abscess of the left lung has suddenly developed acute chest pain and dyspnea while coughing, tachycardia has increased. The control Ro-gram shows left lung collapse, the air in the left pleural cavity and a horizontal fluid level. What is the mechanism of this complication?

Explanation

Lung abscess is a type of liquefactive necrosis of the lung tissue and formation of cavities (more than 2 cm) containing necrotic debris or fluid caused by microbial infection. It can also be caused by aspiration, which may occur during altered consciousness and it usually causes a pus-filled cavity.

Recall that An abscess is a cavity filled with Purulent content found within body tissue. This patient presents with an acute abscess of the left lung and can be easily identified on X-ray fluid as a cavity with air-fluid level. 

One major complication of an abscess is the rupture and release of its content into surrounding tissues. This is a serious medical concern. Potential complications following rupture of an abscess could lead to Empyema - this is a large collection of infected fluid around the lung that occurs where the abscess is. As seen in this patient with air and horizontal fluid level in the pleural cavity.

 
125. A 24-year-old male patient had been diagnosed with class III diffuse toxic goiter. There is moderate hyperthyroidism. A surgery was suggested, and the patient agreed to it. What preoperative measures should be taken for prevention of thyrotoxic crisis in the postoperative period?

Explanation

 

The patient in question already has diffuse toxic goiter with clinical manifestation of hyperthyroidism. Thyroid storm, also referred to as thyrotoxic crisis, is an acute, life-threatening, hypermetabolic state induced by excessive release of thyroid hormones in individuals with thyrotoxicosis. During a thyroid storm, an individual’s heart rate, blood pressure, and body temperature can soar to dangerously high levels. Without prompt, aggressive treatment, thyroid storm is often fatal. Thyroid storm may be precipitated by the stress of surgery, anesthesia, or thyroid manipulation and may be prevented by pretreatment with antithyroid drugs.

126. A 26-year-old male patient complains of pain in the right knee, which is getting worse in the morning. Two weeks before, he consulted an urologist about prostatitis. Objectively: conjunctivitis is present. There is also peri-articular edema of the knee joint, redness of the overlying skin. Rheumatoid factor was not detected. Until further diagnosis is specified, it would be reasonable to start treatment with the following antibiotic:

Explanation

A 26 year old male patient that presented with arthritis, prostatitis and conjunctivitis raised strong suspicion for a sexually transmitted infection with Chlamydia or Reactive Arthritis in which Chlamydia is also a culprit. 

Chlamydia is a common sexually transmitted infection caused by bacteria. In men, it can manifest with urethritis which can spread to the prostate causing Prostatitis; conjunctivitis; Arthritis and commonly cause pelvic inflammatory disease (PID) in women. Chlamydia is an obligate intracellular organism and often treated with macrolides (Azithromycin) or Tetracyclines. Drugs’ ability to accumulate intracellularly makes them very effective against Chlamydia. 

Penicillin is the drug of choice in the case of Syphilis; Cephalosporin (Ceftriaxone) in the case of Gonorrhea.

 
127. A 66-year-old female patient has been admitted to a hospital for massive gross hematuria with release of shapeless blood clots, frequent painful urination. The patient also reports a moderate weight loss within 3-4 months. Gross hematuria that was not accompanied by pain and dysuria first occurred three months ago for no apparent reason, and after a few days the bleeding subsided independently. What is the most likely diagnosis?

Explanation

People with bladder cancer may experience the following symptoms or signs: Blood or blood clots in the urine (Hematuria) which is usually painless; Pain or burning sensation during urination; Frequent urination; Feeling the need to urinate many times throughout the night (Nocturia); Feeling the need to urinate, but not being able to pass urine etc.

Most often, bladder cancer is diagnosed after a person tells his or her doctor about blood in the urine, also called hematuria. "Gross hematuria" means that enough blood is present in the urine that the patient can see it. It is also possible that there are small amounts of blood in the urine that cannot be seen. This is called "microscopic hematuria," and it can only be found with a urine test.

Renal tumor will present with hematuria, flank pain and palpable mass which is absent in this patient. Urolithiasis will also present with colicky flank pain (lumboabdominal pain), hematuria, vomiting, which are also not present in this patient. Hematuria is not usually associated with Acute or Chronic Cystitis, suprapubic pain is more common with these conditions.

 
128. A 47-year-old female patient has an 8-year history of ulcerative colitis, has been treated with glucocorticoids. She complains of cramping pain in the umbilical region and left iliac region which has significantly increased during the past 2 weeks, diarrhea with mucus and blood 4-6 times a day, elevated body temperature up to 38 − 39*C, headache and pain in the knee joints. Objectively: the patient is in moderate condition, Ps- 108/min, AP- 90/60 mm Hg; heart and lungs are unremarkable; the tongue is moist; abdominal muscle tone is significantly decreased; peristaltic noises are absent. What complication developed in the patient?

Explanation

The term inflammatory bowel disease describes a group of disorders in which the intestines become inflamed. Two major types of inflammatory bowel disease are Ulcerative colitis and Crohn’s disease. Ulcerative colitis is limited to the colon or large intestine. Crohn’s disease, on the other hand, can involve any part of the gastrointestinal tract from the mouth to the anus. 

The patient in question suffers from Ulcerative colitis. Common complications include; Toxic megacolon (toxic dilatation of the colon), or fulminant colitis. If Ulcerative colitis attack is severe, the colon might burst, or infection could spread through your body. Your intestines stop moving waste, and your belly swells. However, if a toxic megacolon is not urgently attended to, it could further lead to perforation.

Fistulas, abscess, and strictures are commonly associated with Crohn’s disease.

 
129. For 3 days, a 28-year-old female patient had had the body temperature increase up to 38*C, weakness, poor appetite, nausea, a single vomiting. On the 4th day the temperature was normal, the condition improved, but the jaundice developed. Objectively: moderate ictericity of skin, +3 cm enlarged liver of elastic consistency. Ortner’s, Kehr’s and Voznesensky’s symptoms are negative. What test will verify the diagnosis?

Explanation

This patient’s illness of an acute infection that lasted for 3 days with fever, hepatomegaly, jaundice and resolved on its own is consistent with Acute Hepatitis. Hepatitis B, C and D commonly manifest as chronic illnesses, while Hepatitis E is commonly associated with pregnant women. Hepatitis A on the other hand is transmitted fecal-orally and can produce an acute hepatitis infection in which the patient can recover completely with or without treatment.

Hepatitis A is a highly contagious liver infection caused by the hepatitis A virus. You're most likely to get hepatitis A from contaminated food or water or from close contact with a person or object that's infected. Mild cases of hepatitis A don't require treatment. Most people who are infected recover completely with no permanent liver damage.

IgM indicates an Acute Infectious process (IgM Anti-HAV - IgM Antibody to hepatitis A virus). IgG indicates a Chronic Infectious Process and it can be seen in immunized patients.

Of all the options listed, only IgM Anti-HAV will verify the diagnosis. Complete Blood Count will show increased Lymphocytes (shows an active viral infection) but will not verify which particular virus. Other options will tell us there is a pathology somewhere in the liver but none will verify the particular virus.

 
130. A 40-year-old male patient has had heaviness in the epigastric region for the last 6 months. He has not undergone any examinations. The night before, he abused vodka. In the morning there was vomiting, and 30 minutes after physical activity the patient experienced dizziness and profuse hematemesis. What pathology should be suspected in the first place?

Explanation

The patient abused Vodka the night before he presented to the hospital with dizziness, profuse hematemesis (bloody vomit). Heavy alcohol ingestion is considered to be one of the most important predisposing factors as about 50% to 70% of the patients diagnosed with Mallory-Weiss syndrome have a history of the same.

Mallory-Weiss syndrome (MWS) is one of the common causes of acute upper gastrointestinal (GI) bleeding, characterized by the presence of longitudinal superficial mucosal lacerations (Mallory-Weiss tears). These tears occur primarily at the gastroesophageal junction; they may extend proximally to involve the lower or even mid esophagus and at times extend distally to involve the proximal portion of the stomach.

It is a lower esophageal lacerations (not ulcerations) happening to patients with repetitive forceful retching and vomiting following excessive alcohol intake.

 
131. A 55-year-old patient complains of severe itching, burning and pain in the eyes, skin redness in the outer corners of the palpebral fissure. Objectively: skin around the outer corners of the palpebral fissure is macerated, eczematous, there are single moist cracks. Palpebral conjunctiva is hyperemic, quaggy. There are minor discharges in form of stringing mucus. What is the most likely diagnosis?

Explanation

The conjunctiva is a thin, translucent, relatively elastic tissue layer with both bulbar and palpebral portions. The bulbar portion of the conjunctiva lines the outer aspect of the globe, while the palpebral portion covers the inside of the eyelids. 

Conjunctivitis refers to any inflammatory condition of the membrane that lines the eyelids and covers the exposed surface of the sclera. It is the most common cause of “red eye.”

Chronic conjunctivitis is most commonly caused by Staphylococcus species, although other bacteria are occasionally involved. This type of conjunctivitis often develops in association with blepharitis - bacterial colonization of the eyelid margins (palpebral conjunctiva is hyperemic and quaggy). The symptoms of chronic conjunctivitis include itching, burning, a foreign-body sensation and morning eyelash crusting, flaky debris, erythema and warmth along the lid margins, as well as eyelash loss and bulbar conjunctival injection (red eye).

Acute bacterial conjunctivitis typically presents with burning, irritation, tearing and, usually, a mucopurulent or purulent discharge and usually self-limited.

 
132. An 80-year-old patient complains of constantly urinating small amounts, a feeling of pressure in the lower abdomen. Objectively: there is a suprapubic spherical bulging with percussion dullness over it. What syndrome occurred in the patient?

Explanation

Paradoxical Ischuria is also known as urinary retention which refers to the inability to completely empty the bladder. It results from either a blockage that partially or fully prevents the flow of urine, or your bladder not being able to maintain a strong enough force to expel all the urine. Note that Urinary Incontinence and Urinary retention are completely opposite terms. Urinary Incontinence refers to the inability to hold urine and occurs when the control of the urinary sphincter is lost or weakened. 

Notice that there is a suprapubic spherical bulging with percussion dullness - this is indicative of a full bladder which implies that this possess is unable to empty the bladder ( paradoxical ischuria/ urinary retention).

 
133. A 49-year-old female patient with schizophrenia is all the time listening to something, insists that \"there is a phone in her head\" as she hears the voice of her brother who tells her to go home. The patient is anxious, suspicious, looks around all the time. Specify the psychopathological syndrome:

Explanation

This patient clearly shows signs of auditory hallucination which is very often seen in patients with Schizophrenia. Auditory hallucinations, or hearing voices, is a common symptom in people living with Schizophrenia. These voices can call your name, argue with you, threaten you, come from inside your head or via outside sources, and can begin suddenly and grow stronger over time.

Hallucinations: perceptions in the absence of external stimuli e.g. seeing a light that is not actually present; hearing or talking to someone when no one is actually there. Types: Auditory (common in Psychiatry illnesses like Schizophrenia); Visual (common with delirium, drug intoxication); Olfactory; Tactile (e.g. ‘Cocaine crawlies’) etc.

 
134. A 30-year-old male patient complains of inertness, low-grade fever, bleeding gums, frequent quinsies, aching bones. Objectively: the patient has pale skin and mucous membranes, sternalgia, +2 cm liver, +5 cm painless spleen. Blood test results: RBC- 2,7 · 1012/l, Нb- 80 g/l, WBC- 3 · 109/l, eosinophils - 4%, basophils - 5%, blasts - 4segmented neutrophils - 17%, lymphocytes - 29%, myelocytes - 25%, promyelocytes - 12%, monocytes - 2%, platelets - 80 · 109/l, ESR - 57 mm/h. What test should be performed to verify the diagnosis?

Explanation

Normal Rbc level in the blood ranges from about Male: 4.3 − 5.9 · 1012/L Female: 3.5 − 5.5 · 1012/L and hemoglobin level: Male: 135-175 g/L; Female: 120-160 g/L. A reduction in these values in the blood test indicates Anemia. Red blood cells are produced in the bone marrow of large bones and a bone marrow biopsy is needed for diagnosis. Platelets and White Blood Cells are also markedly decreased in this case. We are actually looking at a case of Pancytopenia.  

 

Pancytopenia occurs when a person has a decrease in all three blood cell types. This happens when something is wrong with the bone marrow, where blood cells are formed. Pancytopenia has many possible causes: Diseases such as cancer, lupus or bone marrow disorders. Sternal puncture which is a type of bone marrow biopsy is indicated to verify the diagnosis.

Complete Blood Count
135. A 46-year-old male patient complains of periodic epigastric pain that occurs at night. Objectively: HR- 70/min, AP- 125/75 mm Hg, tenderness in the epigastric region is present. EGD confirms duodenal ulcer of 0,6 cm in diameter. Test for H. Pylori is positive. Which of the given antisecretory drugs will be a compulsory element of the treatment regimen?

Explanation

Infection with H pylori is associated with the development of peptic ulcer disease, gastric cancer, and mucosa-associated lymphoid tissue lymphoma. Treatment Regimens for Eradication of H. pylori are omeprazole, amoxicillin, and clarithromycin (OAC) for 10 days; bismuth subsalicylate, metronidazole, and tetracycline (BMT) for 14 days; and lansoprazole, amoxicillin, and clarithromycin (LAC), which has been approved for either 10 days or 14 days of treatment. 

Proton Pump Inhibitors (PPI) are the first line antisecretory agents used in treating H. Pylori associated Peptic Ulcer Disease. Examples of PPI’s include: Omeprazole; Lansoprazole; Esomeprazole; Pantoprazole. They irreversibly inhibit H+/K+ ATPase in stomach parietal cells.

 

Famotidine, Cimetidine are examples of H2-Histamine receptor blockers. This class of antisecretory agents are older than Proton Pump Inhibitors and no longer first line antisecretory agents due to their adverse effect profile such as multiple drug interactions, antiandrogenic effects etc.

136. A 47-year-old male patient has been lately complaining of compressing chest pain that occurs when he walks a distane of 700-800 m. Once a week, he drinks 2 liters of beer. Rise in arterial pressure has been observed for the last 7 years. Objectively: Ps- 74/min, AP- 120/80 mm Hg. The bicycle ergometry performed at workload of 75 watts shows 2 mm ST-segment depression in V 4 − V 6 leads. What is the most likely diagnosis?

Explanation

An angina refers to chest pain due to an ischemic myocardium secondary to coronary artery narrowing or spasms. Canadian Cardiovascular Society grading of angina pectoris is a classification used to grade the severity of Angina.

Class 0: Asymptomatic Angina- Mild myocardial ischemia with no symptoms.

Class I: Angina only with strenuous exertion- Presence of angina during strenuous, rapid, or prolonged ordinary activity (walking or climbing the stairs).

Class II:Angina with moderate exertion- Slight limitation of ordinary activities when they are performed rapidly, after meals, in cold, in wind, under emotional stress, during the first few hours after waking up, but also walking uphill, climbing more than one flight of ordinary stairs at a normal pace and in normal conditions.

Class III: Angina with mild exertion- Having difficulties walking one or two blocks or climbing one flight of stairs at normal pace and conditions e.g walking short distances (20 - 100m)

Class IV: Angina at rest- No exertion needed to trigger angina.

 

The patient in perspective experiences anginal symptoms after he walks about 700 - 800m (a short distance and normal activity).

 

The New York Heart Association (NYHA) also classifies the extent of heart failure. It classifies patients in one of the four categories based on their limitations during physical activity.

CLASS I: No symptoms and no limitation in ordinary physical activity

CLASS II: Mild symptoms (mild shortness of breath and/or angina) and slight limitation during ordinary activity.

CLASS III: Marked limitation in activity due to symptoms, even during less than ordinary activity e.g. walking short distances (20 - 100m). Comfortable only at rest.

 

CLASS IV: Severe limitations. Experiences symptoms even while at rest.

 
137. A patient had four generalized convulsive seizures within a day. Between the seizures the patient did not come to waking consciousness (was in a coma or stupor). Specify his state:

Explanation

Status epilepticus (SE) is a common, life-threatening neurologic disorder that is essentially an acute, prolonged epileptic crisis. Status epilepticus is defined as a continuous seizure lasting more than 30 min, or two or more seizures without full recovery of consciousness between any of them. Based on recent understanding of pathophysiology, it is now considered that any seizure that lasts more than 5 min probably needs to be treated as SE.

It can also be defined as a seizure that persists for a sufficient length of time or is repeated frequently enough that recovery between attacks does not occur. Therefore, the key phrase here is “recovery between attacks does not occur.” The question puts it this way - ‘between the seizures the patient did not come to waking consciousness.’

Prolonged Status epilepticus can lead to cardiac dysrhythmia, metabolic derangements, autonomic dysfunction, neurogenic pulmonary edema, hyperthermia, rhabdomyolysis, and pulmonary aspiration. Permanent neurologic damage can occur with prolonged SE.
138. A  34-year-old male visited Tajikistan. After return, he complains of fever up to 40*C which occurs every second day and is accompanied by chills, sweating. Hepatosplenomegaly is present. Blood test results: RBC- 3 · 1012/l, Нb- 80 g/l, WBC- 4 · 109/l, eosinophils - 1%, stab neutrophils - 5%, segmented neutrophils - 60%, lymphocytes - 24%, monocytes - 10%, ESR - 25 mm/h. What is the provisional diagnosis?

Explanation

From anamnesis, we understand that the patient has just returned from Tajikistan (Asia) , this should prompt the idea of a malarial infection. Malaria is caused by a plasmodium infection (P.vivax, P.falciparum, P.malariae, P.ovale. In its life cycle, various stages are involved.

The initial symptoms of malaria infection are nonspecific and can include headache, nausea, vomiting, photophobia and muscle aches. A malarial paroxysm is marked by onset of a sudden shaking chill which may last from 10 to 15 minutes or perhaps longer. Elevated temperature accompanies the paroxysm and may be sustained for typically 10 hours or more. This cycle repeats itself every 36 to 72 hours depending on which species the human host has been infected with. The phase of erythrocytic schizogony takes place within  day 6-15 of the disease ; it is characterised by the production of merozoites which are released into the bloodstream.
139. A 23-year-old male patient consulted a doctor about pain occurring in the lower third of the thigh with weight bearing activities and unloaded. The patient denies any injuries to the region. Objectively: the skin is of normal color, deep palpation reveals pastosity and tenderness, movements of the knee joint are limited. Radiograph of the distal femoral metaepiphyseal region shows a zone of degradation and spicules. In blood: immature cells are present, there are no signs of inflammation. What is the most likely diagnosis?

Explanation

Osteosarcoma is a type of cancer that produces immature bone. It is the most common type of cancer that arises in bones, and it is usually found at the end of long bones, often around the knee. Presents as a painful enlarging mass or pathologic fractures. A codman Triangle ( from elevation of the periosteum)  or sunburst pattern are classical findings on radiography. Note that in the blood immature cells were found. The absence of signs of inflammation rules out the option of Chronic osteomyelitis ( which is an infection of the bone tissue) . 

Multiple myeloma is also known as B-cell myeloma. It is a form of cancer that affects the plasma cells. Bence Jones proteins are a key finding in this pathology. 

 

Marble bone disease is also called Osteopetrosis or Albers-Schonberg disease. It is a rare inherited disorder in which the bone hardens and becomes denser ( stone bone). It is the opposite of osteoporosis and osteomalacia.

140. A 47-year-old female patient complains of having pain on swallowing and difficult passing of solid food for two months. The patient has taken to the liquid and semi-liquid food. During the last week the liquid food has barely passed through. General condition is satisfactory, the patient is undernourished, the appetite is preserved, there is a fear of eating. What is the provisional diagnosis?

Explanation

Esophageal cancer refers to a malignant tumor of the esophagus. The esophagus is the tube that connects the throat with the stomach. The two main subtypes of esophageal cancer are:

  • Squamous cell carcinoma: This arises from the cells lining the upper part of the esophagus.

  • Adenocarcinoma of the esophagus: This occurs due to changes in the glandular cells that exist at the junction of the esophagus and the stomach.

The junction between the stomach and esophagus (Gastroesophageal junction) is already narrower than other parts of the esophagus. So, having a cancer develop at this junction or close to it just exacerbates the condition and prevents free passage of food starting with solids.

People often experience no symptoms during the initial stages of this cancer. The majority of esophageal cancer diagnoses occur when the cancer has advanced.

 

When symptoms are present, they may include and usually the first noticeable symptom: Dysphagia (difficulty in swallowing). As the tumor narrows the passage in the esophagus, pushing food through becomes harder. And eventually, if the growth goes on unchecked, it begins to become narrower and affects semi-solids and then at the terminal stage, liquids.

141. A 36-year-old male patient complains of having headache, obstructed nasal breathing, purulent nasal discharge for 2 weeks. A month before, he had his right maxillary premolar filled. Radiolography revealed an intense opacity of the right maxillary sinus. Diagnostic puncture revealed a large amount of thick malodorous crumbly pus. What is the most likely diagnosis?

Explanation

The first point to take note of here is that the patient had his right maxillary premolar filled a month ago.

Sinusitis is the inflammation of one or multiple sinuses. Your sinuses are hollow air-filled cavities (like pouches) behind your forehead, cheeks and eyes. Odontogenic sinusitis is any inflammation of a dental origin and of your sinuses. It’s almost always one of the maxillary sinuses that are affected. These can be found to the right and left of your nose, behind the cheeks.

This condition occurs when a dental infection caused by lack of oral hygiene or dental surgery, such as tooth implants, a tooth removal or a sinus lift, reaches the maxillary sinus. Your teeth project up towards the maxillary sinus, and sometimes directly into it. There is a membrane called the Schneiderian membrane that protects your sinuses from teeth and teeth infections, but if surgery or a tooth’s root disrupts the membrane, the infection can spread.

 

Odontogenic sinusitis accounts for the majority of cases of chronic maxillary sinusitis. It is the most common cause of unilateral maxillary sinusitis

142. A patient with autoimmune thyroiditis accompanied by multinodular goiter underwent the right lobe ectomy and subtotal resection of the left lobe. What drug should be administered to prevent postoperative hypothyroidism?

Explanation

Lobectomy has been generally recommended as an optimal surgical approach for symptomatic unilateral goiter, toxic adenoma, and low-risk differentiated thyroid cancer. Although patients are expected to maintain normal thyroid function after lobectomy or subtotal resection since not all the thyroid tissue was resected, some patients develop hypothyroidism that requires thyroid-hormone replacement. Routine levothyroxine (L-Thyroxine) supplementation is commonly prescribed for prophylaxis of postoperative hypothyroidism. 

 

Merkazolil and Iodomarin can be used in case of Hyperthyroidism. They are anti-thyroid medications. Lithium is commonly used in Psychiatry cases for Bipolar conditions but it is commonly associated with Hypothyroidism and will not be useful in this case.

143. A 49-year-old female patient has type 1 diabetes of moderate severity. The disease is complicated by retinopathy and polyneuropathy. Besides that, repeated analyses of the daily urinary excretion of albumin revealed microalbuminuria (200-300 mg/day). Glomerular filtration rate is 105 ml/min. Blood pressure is within normal range. Normalization of the following indicator should be the first-priority task in the secondary prevention of diabetic nephropathy:

Explanation

The glycated hemoglobin (HbA1c) test is an important blood test that determines how well you are managing your diabetes. The hemoglobin A1c test tells you your average level of blood sugar over the past 2 to 3 months.

Diabetes mellitus (DM) is the leading cause of chronic kidney disease (CKD) worldwide. Measuring glycated hemoglobin (HbA1c) has been suggested as a means of assessing glycemic control in patients with diabetes. Current guidelines recommend a target HbA1c of approximately 7% for preventing or delaying microvascular complications, including diabetic kidney disease (Diabetic Nephropathy).
144. An emergency physician arrived to provide medical care for a hangman taken out of the loop by his relatives. The doctor revealed no pulse in the carotid arteries, lack of consciousness, spontaneous breathing and corneal reflexes; cadaver spots on the back and posterior parts of extremities. A person can be declared dead if the following sign is present:

Explanation

Forensic pathologists are essentially required to estimate the time since death (TSD) to assist in death investigation. The early post-mortem phase is most frequently estimated using the classical triad of post-mortem changes – rigor mortis, algor mortis and livor mortis. 

Algor Mortis: Humans are warm-blooded organisms, which means that we maintain a constant internal temperature, regardless of the outside environment. The brain is our thermostat and the circulatory system is the main heat dissipator. However, within seconds of death, the brain cells begin to die and the heart stops pumping blood. Without the brain and and the blood distributing heat, the corpse eventually starts to match the outside temperature.

Rigor mortis is the post-mortem stiffening of muscles, caused by the depletion of adenosine triphosphate (ATP) from the muscles.

 

Livor Mortis: This is the final stage of death. When the heart stops beating, the blood is now at the mercy of gravity. It tends to collect at certain parts of the body. Depending on the position of the body, these parts would vary. For instance, if the person was flat on their back when they died, the blood would collect in the parts that are touching the ground (just as in this patient - cadaver spots were on the back and posterior parts of the extremities). If the person was hanging, it would collect in their fingertips, toes, and earlobes. Lividity starts with the skin where the blood has settled developing a bright red color. After a few hours, the color changes from red to bluish-purple. The bluish coloring of the skin is called livor mortis or cadaver spots.

145. A 69-year-old male patient has been hospitalized with hypothermia. Objectively: the patient is pale, has shallow breathing. AP- 100/60 mm Hg, Ps- 60/min. Palpation of the abdomen and chest reveals no pathological signs. The body temperature is of 34,8*C. The patient’s breath smells of alcohol. Give treatment recommendations:

Explanation

Hypothermia is a severe condition in which the body temperature drops to an abnormally low level. It occurs when the body is unable to produce enough heat to counter the heat that it is losing. Under healthy conditions, the body maintains a relatively stable temperature of around 98.6˚F or 37˚C. 

The following techniques can help treat hypothermia:

Passive external rewarming: This uses the individual’s heat-generating ability. It involves removing their cold, wet clothing, ideally replacing it with adequately insulated, dry clothing, and moving them to a warm environment. 

Active external rewarming: This involves applying warming devices, such as hot-water bottles or warmed forced air, externally to truncal areas of the body. For example, the individual could hold a hot-water bottle under each arm.

Active core rewarming: This uses warmed, intravenous fluids to irrigate body cavities, including the thorax, peritoneum, stomach, and bladder. Other options include getting the individual to inhale warm, humidified air, or applying extracorporeal rewarming by using a heart-lung machine.

Do not give a person alcohol if they have signs of hypothermia, and avoid giving any drinks to an unconscious person.

 

Considering the critical condition of this patient, active core warming by using warmed intravenous fluids which gets into the system immediately and will bring the most desired result at the shortest time possible is by far the best treatment recommendation.

146. A 26-year-old male patient consulted a doctor about sore throat, fever up to 38,2*C. A week before, the patient had quinsy, didn’t follow medical recommendations. On examination, the patient had forced position of his head, trismus of chewing muscles. Left peritonsillar region is markedly hyperemic, swollen. What is the provisional diagnosis?

Explanation

Extension of a tonsil infection can take place in the surrounding tissue and is referred to as  a Peritonsillar abscess or quinsy. The 26 year old man had previously had quinsy but didn’t follow medical advice. This complication of tonsillitis is usually caused by a B-hemolytic streptococcal infection and are characterised by an extremely sore throat and high fever ( as seen in the patient) . 

A peritonsillar abscess forms in the tissues of the throat next to one of the tonsils. An abscess is a collection of pus that forms near an area of infected skin or other soft tissue. The abscess can cause pain, swelling, and, if severe, blockage of the throat. If the throat is blocked, swallowing, speaking, and even breathing become difficult.

 

The key sign that distinguishes quinsy/peritonsillar abscess from ordinary tonsillitis is the presentation of trismus. Trismus is usually absent in any form of tonsillitis but would be seen in a Peritonsillar abscess.

147. A 77-year-old male patient complains of inability to urinate, bursting pain above the pubis. The patient developed acute condition 12 hours before. Objectively: full urinary bladder is palpable above the pubis. Rectal prostate is enlarged, dense and elastic, well-defined, with no nodes. Interlobular sulcus is distinct. Ultrasonography results: prostate volume is 120 cm3, it projects into the bladder cavity, has homogeneous parenchyma. Prostate-specific antigen rate is of 5 ng/ml. What is the most likely disease that caused acute urinary retention?

Explanation

 

Benign prostatic Hyperplasia is a non carcinogenic pathology that occurs mostly in men above 40. It presents with symptoms similar to Prostate cancer. These include frequency of urination, hesitancy, dribbling, and frequent nighttime urination. To differentiate between these two pathologies, an objective examination is carried out (  a digital rectal examination ). In this case, the prostate is dense, enlarged, elastic  and well defined with no nodes - these findings support the initial diagnosis of BPH because on digital rectal examination, prostatic carcinoma will not be well defined and will present with nodes. To further prove this diagnosis, the Prostate specific antigen is measured ( normal <4). A marginal increase is going to support the diagnosis of BPH while PSA of 10 and above will indicate prostate cancer. 

148. After having the flu, a 39-year-old male patient with a history of Addison’s disease developed a condition manifested by weakness, depression, nausea, vomiting, diarrhea, hypoglycemia. AP- 75/50 mm Hg. Blood test results: low corticosterone and cortisol, 13-oxycorticosteroids, 17-oxycorticosteroids levels. What condition developed in the patient?

Explanation

An Addisonian crisis most often occurs in a patient with known adrenal insufficiency (this patient has a history of Addison’s disease), either primary or secondary. Primary adrenal insufficiency (Addison Disease) is characterized by low cortisol and high ACTH from the destruction of the adrenal glands. Chronic, stable primary adrenal insufficiency is called Addison's disease, which is a  chronic condition controlled with long-term glucocorticoid therapy.

Addisonian crisis, also known as adrenal crisis or acute adrenal insufficiency is an endocrinologic emergency commonly seen in patients with known adrenal insufficiency. It is characterized as an acute change in physiologic status, quickly progression from nonspecific symptoms of fatigue, weakness, nausea, vomiting, abdominal pain, back pain, diarrhea, dizziness, hypotension, syncope, eventually to obtundation, metabolic encephalopathy, and shock. 

 

Most cases are patients with known adrenal insufficiency who experience an acute stress event (patient had flu), who do not adjust their usual dose of glucocorticoid therapy and their body's cortisol stores are quickly depleted leading to an Addisonian crisis as these patients cannot produce their own cortisol. Roughly half of the patients have undiagnosed Addison's disease and present to the emergency room in acute crisis and shock after an acute stress event.

149. In a cold weather, the emergency room admitted a patient pulled out of the open water. There was no respiratory contact with the water. The patient is excited, pale, complains of pain, numbness of hands and feet, cold shiver. Respiratory rate is 22/min, AP- 120/90 mm Hg, Ps- 110/min, rectal temperature is 34, 5*C . What kind of warming is indicated for this patient?

Explanation

Hypothermia is a severe condition in which the body temperature drops to an abnormally low level. It occurs when the body is unable to produce enough heat to counter the heat that it is losing. Under healthy conditions, the body maintains a relatively stable temperature of around 98.6˚F or 37˚C. 

The following techniques can help treat hypothermia:

Passive external rewarming: This uses the individual’s heat-generating ability. It involves removing their cold, wet clothing, ideally replacing it with adequately insulated, dry clothing, and moving them to a warm environment. 

Active external rewarming: This involves applying warming devices, such as hot-water bottles or warmed forced air, externally to truncal areas of the body. For example, the individual could hold a hot-water bottle under each arm.

Active core rewarming: This uses warmed, intravenous fluids to irrigate body cavities, including the thorax, peritoneum, stomach, and bladder. Other options include getting the individual to inhale warm, humidified air, or applying extracorporeal rewarming by using a heart-lung machine.

Do not give a person alcohol if they have signs of hypothermia, and avoid giving any drinks to an unconscious person.

 

From the vitals given, this patient is fairly stable except for the tachycardia (110bpm) and low body temperature, therefore passive warming will suffice. The patient was pulled out of the open water in a cold weather so removing the wet clothing, and replacing it with adequately insulated, dry clothing, and moving them to a warm environment will keep the patient warm.

150. A week before, a 65-year-old male patient suffered an acute myocardial infarction. His general condition has deteriorated: he complains of dyspnea at rest, pronounced weakness. Objectively: edema of the lower extremities, ascites is present. Heart borders are extended, paradoxical pulse is 2 cm displaced from the apex beat to the left. What is the most likely diagnosis?

Explanation

One of the most common complications occurring post-infarction is a ventricular aneurysm. Almost 85% to 90% of the ventricular aneurysms occur in the setting of acute anterior wall myocardial infarction. Left ventricular aneurysm is defined as a localized area of myocardium with abnormal outward bulging and deformation during systole and diastole. The natural course leading to the formation of a ventricular aneurysm involves a full-thickness infarct that has been replaced by fibrous tissue. This inert portion cannot take part in the contraction and herniates outward during systole.

A week before presenting to the hospital, he had an acute myocardial infarction. A true left ventricular aneurysm following acute myocardial infarction can occur as early as within 48 hours or two weeks post-infarction. 

 

Common symptoms associated with the ventricular aneurysm include: Fatigue; Shortness of breath (dyspnea); Chest pain; Palpitations; Syncope; Fluid retention causing swelling of ankles, feet, or abdomen (ascites); Stroke; Limb or visceral ischemia.

151. A 41-year-old patient cosulted a dermatologist about discoloration, thickening, brittleness of toenails. These symptoms have been present for about five years. Objectively: nail plates in all toes are thickened, of dirty yellow color, lustreless, crumble over the edge. Microscopy of the nail plate material treated with alkali revealed mycelial filaments. Material inoculation onto Sabouraud medium resulted in growth of the Trichophyton rubrum colony. What is the most likely diagnosis?

Explanation

Dermatophyte infections, also known as tinea, are the most common fungal infections of the skin, hair, and nails.The term "Dermatophyte" refers to fungal species that infect keratinized tissue, and includes members of the Trichophyton, Microsporum, and Epidermophyton genera. 

Trichophyton rubrum is a dermatophytic fungus. It colonizes the upper layers of dead skin, and is the most common cause of athlete's foot, fungal infection of nail, jock itch, and ringworm worldwide. T. rubrum colony shows Partial yellow-green fluorescence under wood-lamp.

T. rubrum is now the most common cause of invasive fungal nail disease (called onychomycosis or tinea unguium). Nail invasion by T. rubrum tends to be restricted to the underside of the nail plate and is characterized by the formation of white plaques on the lunula that can spread to the entire nail. The nail often thickens and becomes brittle, turns brown or black.

 

The question clearly states that inoculation onto Sabouraud medium resulted in growth of T. rubrum colony which makes Rubromycosis the correct answer and rules out Epidermophytosis and Psoriasis (patient must have psoriasis first before a diagnosis of Psoriasis of the nails can be made). 

152. A 25-year-old patient works as a tractor driver. Four days ago, he got pain in the left axillary region, general weakness, fever up to 38*C. He hadn’t sought medical help until a painful solid lump appeared in this region. Objectively: in the left axilla there is a very painful cone-shaped mass sized 3x2,5 cm, with a destruction in the center of the pointed vertex. The surrounding skin is hyperemic, there are purulent discharges. What is the most likely diagnosis?

Explanation

Hydradenitis refers to small painful lumps that form under the skin usually in areas of friction such as: axilla, groin, buttocks and breast.  

Carbuncle: clusters of boil; an area of subcutaneous necrosis which discharges itself unto the surface through multiple sinuses. 

 

Furuncle ( boil) - forms under the hair follicle and associated glands which contains pus. 

153. A 19-year-old male patient complains of intense pain in the left knee joint. Objectively: the left knee joint is enlarged, the overlying skin is hyperemic, the joint is painful on palpation. Blood test results: RBC- 3, 8 · 1012 /l, Hb- 122 g/l, lymphocytes - 7, 4 · 109 /l, platelets - 183 · 109/l. ESR- 10 mm/h. Duke bleeding time is 4 minutes, Lee-White clotting time - 24 minutes. A-PTT is 89 sec. Rheumatoid factor is negative. What is the most likely diagnosis?

Explanation

Partial Thromboplastin Time (PTT): <28seconds (normal)

PTT in this patient is 89seconds.

Defect in the Intrinsic coagulation pathway results in an Increased Partial Thromboplastin Time (PTT).

Intrinsic pathway coagulation defect:

Hemophilia A: deficiency of Factor VIII

Hemophilia B: deficiency of Factor IX

Hemophilia C: deficiency of Factor XI

Hemophilia is usually an inherited bleeding disorder in which the blood doesn't clot properly. This can lead to spontaneous bleeding as well as bleeding following injuries or surgery. Blood contains many proteins called clotting factors that can help to stop bleeding. People with hemophilia have low levels of any of factor VIII (8); factor IX (9) or factor XI (11).

 

Hemophilia commonly presents as hemarthrosis (bleeding into joints e.g. knee); easy bruising; bleeding after trauma or surgery (e.g. dental procedures).

154. A 29-year-old female patient complains of dyspnea and palpitations on exertion. According to her mother, as a child she had heart murmur, did not undergo any examinations. Objectively: the patient has pale skin, Ps- 94/min, rhythmic. AP- 120/60 mm Hg. In the II intercostal space on the left auscultation reveals a continuous rasping systolodiastolic murmur, diastolic shock above the pulmonary artery. Blood and urine are unremarkable. What is the most likely diagnosis?

Explanation

Patent ductus arteriosus is a congenital heart anomaly in which the ductus arteriosus fails to close at birth. PDA is normal in utero and normally closes only after birth. Due to the failure of the ductus arteriosus ( a connection between the aorta and pulmonary artery), oxygenated blood flows from the aorta into the pulmonary artery then back to the lungs for more oxygenation. Key signs include: dyspnea, tachycardia, cyanosis of the lower extremities etc. On objective examination, we observe increased systolic pressure, left subclavicular thrill, a continuous rasping systo diastolic murmur in the 2nd intercostal space on the left; this murmur can also be termed like “a machine gun” or “rolling thunder”. Echocardiogram and normal chest x-ray are used in the diagnosis. One major finding on chest x-ray is the cardiac Silhouette (loss of normal borders between thoracic structures). 

 Tetralogy of Fallot is caused by the anterosuperior displacement of the infundibular septum and is the most common cause of cyanosis in early childhood. It consists of 4 anomalies: Ventricular septal defect, Right ventricular hypertrophy, pulmonary valve stenosis and a misplaced (overriding) aorta. A systolic ejection murmur is common in this anomaly.
155. A 67-year-old female patient with hypertensive crisis has asthma, cough with expectoration of frothy pink sputum, moist rales in the lungs. The patient stays in sitti- ng position, respiratory rate is 40/min, AP- 214/136 mm Hg, heart rate - 102/min. What is the most rational tactics of this patient management?

Explanation

156. A 49-year-old male patient complains of retrosternal pain, heartburn, weight loss of 8 kg over the last year, constipation, weakness. The patient has been a smoker for 20 years, and has a 10-year history of gastroesophageal reflux disease. The patient is asthenic, has dry skin. EGD revealed an ulcer in the lower third of the esophagus and esophageal stricture accompanied by edema, hyperemia and multiple erosions of the mucosa. What study is required for more accurate diagnosis?

Explanation

 

From anamnesis, this patient has a history of Gastroesophageal reflux disease; a condition that occurs as a result of weakening of the lower esophageal sphincter. Due to the weakening of the LES, gastric contents flow back into the esophagus leading to symptoms such as heartburns, retrosternal pain, constipation etc. The result from Esophagoduodenoscopy shows the presence of ulcers, stricture and edema at the lower end of the esophagus. For more accurate diagnosis, during the EGD, a biopsy of the esophageal mucosa should be obtained. This biopsy is used to check for an abnormal growth, pre existing infection etc.

157. A 63-year-old male patient with persistent atrial fibrillation complains of moderate dyspnea. Objectively: peripheral edemata are absent, vesicular breathing is present, heart rate - 72/min, AP- 140/90 mm Hg. What combination of drugs will be most effective for the secondary prevention of heart failure?

Explanation

 

The absence of peripheral edema in this patient helps rule out the use of diuretics - this group of drugs will be in the first line of action if the patient presents with any forms of edema. The groups of Drugs indicated for the secondary prevention of heart failure include: Angiotensin Converting enzyme inhibitor ( ACE inhibitors) or Angiotensin Receptor blockers ( ARBs), Beta blockers, Calcium channel blockers, Diuretics etc. Amongst these, a combination between ACE inhibitors and Beta blockers has proved to be more effective than monotherapy. 

158. A 57-year-old male patient had an attack of retrosternal pain that lasted more than 1,5 hours. Objectively: the patient is inert, adynamic, has pale skin, cold extremities, poor volume pulse, heart rate - 120/min, AP- 70/40 mm Hg. ECG shows ST elevation in II, III, aVF leads. What condition are these changes typical for?

Explanation

 

The results from the ECG ( ST elevation in II, III and AVF) indicates an acute myocardial infarction of the inferior wall. Myocardial infarction is the leading cause of cardiogenic shock. Others include; arrhythmias, heart failure, cardiac trauma, mitral valve regurgitation etc. Cardiogenic shock is a medical emergency in which the heart is unable to pump adequate blood to vital organs.

159. During an exam, a 22-year-old female student fainted. She grew up in a family with many children, has a history of frequent acute respiratory infections. Objectively: the patient has pale skin and mucous membranes, split- end hair, brittle nails. Blood test results: RBC- 2, 7 · 1012 /l, Hb- 75 g/l, color index - 0,7, WBC- 3, 2 · 109 /l, platelets - 210 · 109 /l, ESR- 30 mm/h. Blood serum iron is 6 mmol/l. What is the most likely diagnosis?

Explanation

Anemia is a condition characterised by too ew RBCs . In iron deficiency anemia, we observe the presence of low hemoglobin levels coupled with a low color index; recall that iron is responsible for the characteristic red color in Rbc and a deficiency in Iron content in blood will lead to a decreased color index. In this patient, together with the RBC level, the color index and hemoglobin levels are also low ( norm Color index 0.85- 1.05,  Hemoglobin, blood Male: 135-175 g/L Female: 120-160 g/L); Characteristics of individuals with iron deficiency anemia include; brittle hair and nails, pale skin and mucous membrane  etc. 

 

B12 deficiency will lead to the presence of megaloblasts in blood smear with the patient experiencing some neurological disturbances.

160. During the ultrasound study of carotid and vertebral arteries a 74-year-old patient developed a condition manifested by dizziness, weakness, nausea, transient loss of consciousness. Objectively: pale skin, AP- 80/60 mm Hg, Ps- 96/min of poor volume. ECG shows sinus tachycardia, left ventricular hypertrophy. Focal neurological symptoms were not found. What is the provisional diagnosis?

Explanation

Anatomically, the carotid sinus is located at the proximal end of the internal carotid artery just  above the bifurcation of the common carotid artery into the internal and external carotid artery. The carotid sinus houses baroreceptors (stretch receptors); The carotid sinus baroreceptor functions as a sensor responding to the mechanical stretch that occurs to the carotid artery as the arterial blood pressure increases. Notice that the above stated changes occurred on the background of an ultrasound study of the carotid and vertebral bodies.

Carotid sinus syncope (hypersensitivity) refers to an  exaggerated response to carotid sinus baroreceptor stimulation. It is characterised by a drop in blood pressure with or without accompanying bradycardia. 

Orthostatic syncope refers to the loss of consciousness resulting from the postural decrease in blood pressure ( change in position).

Morganani Adams stokes attack refers to periodic loss of consciousness due to a significant loss of cardiac output. 

 

Absence of focal neurological signs rules out the options of acute cerebrovascular accident.

161. A 45-year-old male patient complains of acute pain in his right side irradiating to the right thigh and crotch. The patient claims also to have frequent urination with urine which resembles a meat slops. The patient has no previous history of this condition. There is costovertebral angle tenderness on the right (positive Pasternatsky’s symptom). What is the most likely diagnosis?

Explanation

 

A positive pasternestky’s sign indicates kidney related pathology ruling out the option of Acute appendicitis, cholecystitis and pancreatitis. Urolithiasis and pyelonephritis ( infection of the kidney tubules mostly by bacteria)  have many symptoms in common (eg, hematuria, flank pain, shaking chills, anorexia). Urolithiasis ( stones in the urinary tract) is usually not associated with fever, except in patients with concomitant pyelonephritis. The best way to differentiate these diseases is by conducting a renal ultrasonography.

162. After a holiday in the Crimea, a 49- year-old male patient with a history of lung tuberculosis felt increased weakness, periodic dizziness, easing bowel movements with abdominal pain, the need for additional salting his meals. The patient has noted that his condition improves after some sweet tea and validol taken sublingually. Objectively: there is an intense darkening of skin, AP- 70/50 mm Hg, glycemia is 3,0 mmol/l. What is the possible cause of health deterioration:

Explanation

 

Note that Tuberculosis is the most common cause of Addisson’s disease ( primary adrenal insufficiency). Adrenal insufficiency is the decreased production of adrenocortical hormones (glucocorticoids, mineralocorticoids, and adrenal androgens) and can be primary, secondary, or tertiary. Primary adrenal insufficiency (Addison disease) is caused by a disorder of the adrenal glands. Patients with long-standing adrenal insufficiency can present with postural hypotension, nausea, vomiting, weight loss, anorexia, lethargy, depression, and/or chronic hyponatremia. Patients can also present with loss of libido as a result of hypoandrogenism. Patients with primary adrenal insufficiency also tend to develop hyperpigmentation of the skin, mild hyperkalemia, and metabolic acidosis. The glycemic level of 3.0 mmol/l rules out the option of diabetes mellitus. 

163. A 42-year-old male patient has been delivered to a hospital in a grave condition with dyspnea, cough with expectoration of purulent sputum, fever up to 39, 5*C . The first symptoms appeared 3 weeks ago. Two weeks ago, a local therapist diagnosed him with acute right-sided pneumonia. Over the last 3 days, the patient’s condition deteriorated: there was a progress of dyspnea, weakness, lack of appetite. Chest radiography confirms a rounded shadow in the lower lobe of the right lung with a horizontal fluid level, the right sinus is not clearly visualized. What is the most likely diagnosis?

Explanation

Recall that an abscess refers to a pus filled cavity found within tissues. The result from the chest radiography confirms a rounded shadow ( a cavity) in the lower lobe of the right lung with a horizontal fluid level ( purulent content).  This patient presents with a primary abscess which occurs on the background of a pneumonia infection. 

In Pleuropneumonia, the inflammation of the lung is accompanied by the inflammation of the pleura ( pneumonia plus Pleurisy).

In pulmonary empyema and pleural effusion, purulent contents build up within the pleural space and also occurs as a complication of bacterial pneumonia. 

 

Atelectasis is a loss of lung volume that may be caused by a variety of ventilation disorders, for instance, bronchial injury or an obstructive mass such as a tumor.

164. A 28-year-old male patient complains of sour regurgitation, cough and heartburn that occurs every day after having meals, when bending forward or lying down. These problems have been observed for 4 years. Objective status and laboratory values are normal. FEGDS revealed endoesophagitis. What is the leading factor in the development of this disease?

Explanation

 

Sour regurgitation, cough, heartburns after meals are key findings in Gastroesophageal reflux disease. In this disorder, there occurs a backflow of acidic content of the stomach into the lower end of the esophagus leading to inflammation and formation of erosions. This disorder occurs due to the failure of the lower esophageal sphincter to function properly; its inability to close properly allows for the backflow of gastric content.  A Fibroesophagogastroduedenoscopy is usually carried to confirm diagnosis- in this case we observe an endo esophagitis which helps confirm this diagnosis. 

165. A 30-year-old male patient had been admitted to the TB hospital because of the following changes detected by fluorography: an ill-defined shadow of low intensity up to 1 cm in diameter in the S1 of the right lung. CT scan showed a destruction area in the center of the shadow. Sputum analysis revealed MTB. The patient was diagnosed with focal tuberculosis. What phases of tuberculosis are the identified changes typical for?

Explanation

166. A 43-year-old female complains of significant weakness, sore throat, occurrence of multiple unexplained bruises on her skin. These symptoms have been present for a week, the disease is associated with quinsy which she had some time before. Objectively: body temperature - 38, 9o C , respiratory rate - 24/min, Ps- 110/min, AP- 100/65 mm Hg. The patient has pale skin, petechial rash on the extremities, enlarged lymph nodes. Blood test results: Hb- 80 g/l, RBC- 2,2 · 1012/l; WBC- 3,5 · 109/l; blasts - 52%; eosinophils - 2%; stab neutrophils - 3%; segmented neutrophils - 19%; lymphocytes - 13%; monocytes - 1%; platelets - 35 · 109/l. ESR - 47 mm/h. What test is required to specify the diagnosis?

Explanation

Immunophenotyping is an analysis of heterogeneous ( various) populations of cells for the purpose of identifying the presence and proportions of the various populations ( identifying how much of each cell type is present). This technique works on the basis of identifying different cell markers or antigens peculiar for each cell type. 

This patient presents with low RBC, WBC and platelets coupled with an increased blast cell level; the result from the general blood analysis suggests a leukemia or a lymphoma. Immunophenotyping will help specify the diagnosis. 

 

Cytogenetic study is used to analyze the structure of chromosome material and the study of diseases caused by structural and numerical abnormalities of chromosomes.

167. A 47-year-old male patient complains of compressive chest pain that occurs both at rest and during light physical activity; irregular heartbeat. These problems arose 3 months ago. The patient’s brother died suddenly at the age of 30. Objectively: Ps- 84/min, arrhythmic, AP- 130/80 mm Hg. ECG confirms signs of left ventricular hypertrophy, abnormal Q- waves in V 4 − V 6 leads. EchoCG reveals that interventricular septum is 1,7 cm, left ventricular wall thickness is 1,2 cm. What is the most likely diagnosis?

Explanation

The results from ECG and echocardiogram helps to put the diagnosis of this patient. From these analysis, we can confirm signs of an increase in the thickness ( hypertrophy) of the left ventricle and interventricular septum. From the question stem, the left ventricle has undergone hypertrophy and the interventricular septum is increased in size ( norm 0.6-1.1cm). Hypertrophic cardiomyopathy is a condition in which the heart becomes enlarged without any underlying disease/pathology; this reduces the heart’s pumping ability leading to  cardiac irregularities, dizziness, dyspnea etc. In this case, it is most likely a genetic condition  - autosomal dominant due to mutations in myosin heavy chain. (Patient's brother died at a young age)  
168. A 45-year-old man with thrombophlebitis of the deep veins in his legs suddenly after physical exertion developed sharp pain in his thorax on the right, dyspnea, and hemoptysis. Objectively his condition is severe; he presents with acrocyanosis, shortening of pulmonary percussion sound on the right, and weakened respiration. Respiration is 30/min., blood pressure is 110/80 mm Hg. ECG shows sinus tachycardia, his heart rate is 120/min., the electrical axis of the heart deviates to the right, SI-QIII. What is the most likely diagnosis?

Explanation

 

Notice that this patient has thrombophlebitis- a condition in which blood clots form and block  an already inflamed vein(s). This condition can be superficial or deep ( deep vein thrombosis- as in this case). Pulmonary embolism is one of the various complications of DVT- this occurs when a clot or substance formed elsewhere travels and blocks an artery in the lungs. Sharp pain in the thorax and hemoptysis coupled with weakened respiration are clinical findings in this condition.

169. A 38-year-old woman has been suffering from glomerulonephritis for 20 years. For approximately 16 years she has been presenting with progressing renal parenchymal arterial hypertension that became refractory and accompanied by leg edemas. She receives a combination of 100 mg losartan and 20 mg lercanidipine with insufficient antihypertensive effect. What medicine can she be recommended for intensification of the antihypertensive effect of her therapy?

Explanation

Notice that this patient has Progressing renal parenchyma arterial hypertension accompanied by edemas of the leg; she is already on a combined medication that includes Losartan ( Angiotensin receptor blocker) and Lercanidipine ( a calcium channel blocker). On the basis that this patient has suffered from glomerulonephritis, currently having a renal induced hypertension, presents with edema and does not have a diuretic prescribed as part of the complex treatment, the preferred drug of choice to intensify this treatment will be Torasemide ( a loop diuretic).

This patient is already on an Angiotensin receptor blocker, there is no need to use an ACE inhibitor ( Lisinopril);  we need to try another class with a different mechanism of action.

 
170. A 45-year-old man has been suffering from ankylosing spondylitis for 15 years. For the last 3 years he has been noticing facial swelling and edemas of the limbs. Objectively he assumes a ’’beggar’s” position. X-ray shows,’bamboo spine” changes in the thoracic and lumbar segrilents. Heart ultrasound shows aortic regurgitation. Complete blood count: Hb- 106 g/L; leukocytes - 8.9 ■ 109/L; ESR- 40 mm/hour. Daily proteinuria - 9.6 grams per 24 hours. Blood creatinine - 230 mcmol/L. What is the cause of kidney failure in this case?

Explanation

These abnormal proteins are produced as a result of various diseases. Ankylosing spondylitis is a chronic inflammatory disease of the axial skeleton that leads to partial or even complete fusion and rigidity of the spine. The most common renal  complication of ankylosing spondylitis is Renal amyloidosis. Amyloidosis is a collective term for the extracellular deposition of abnormal proteins, either in a single organ (localized amyloidosis) or throughout the body (systemic amyloidosis). The different subtypes of amyloidosis are categorized according to the origin of the deposited proteins (e.g., AA, AL). Renal amyloidosis is characterised by nephrotic syndrome; in this patient we can see a high level of proteinuria coupled with facial swelling and edema ( due to decreased oncotic pressure).
171. A patient was brought into the pulmonology department with complaints of inspiratory dyspnea and dry cough at the highest point of inhalation. On examination the following is observed: pale skin, cyanotic lips, \"Hippocratic fingers.\" Auscultation detects Velcro-type crackles (like opening a Velcro fastener). X-ray shows a ”ground glass opacity” pattern. What is the most likely diagnosis?

Explanation

Hippocratic nails refer to a condition in which there is proliferation of the distal part of the fingers, especially the nail bed. Velcro Type crackles refer to bilateral crepitation heard especially during slow inspiration but can be associated with expiratory crackles. These types of crackles are seen in fibrotic interstitial lung diseases.

Hand-schuller-christain disease is characterised by a triad of single or multiple punched out bone lesions on the skull, uni/bilateral exophthalmos and diabetes insipidus. Other findings in this disorder include; infantilism, dwarfism and polyuria.

 
172. A 45-year-old man came to the hematologist with complaints of general weakness, elevated body temperature, excessive sweating, enlarged cervical lymph nodes. Objectively his body temperature is 37.5°C, the skin is pale and dry the posterior cervical lymph nodes are dense and elastic, up to 2 cm in diameter, mobile. There are no peculiarities in the patient’s heart and lungs. Hepatosplenomegaly was detected. What examination is necessary to determine the scope of the pathologic process?

Explanation

 

Of the above listed diagnostic procedures, the Computed tomography will be most appropriate in determining the scope of the pathologic process. Note that on examination, enlarged cervical lymph nodes were found; a CT scan is the best procedure ( among the listed) in examining these lymph nodes. An ultrasound will be appropriate for diagnosis of pathologies relating to internal organs.

173. A 35-year-old forestry officer was delivered to the hospital on the 7th day after the onset of the disease. He complains of chills, elevated body temperature up to 40.0°C, sharp headache, and myalgias. On examination his face is puffy and hyperemic, the tongue is dry, “chalk- dusted.” In the left inguinal area, a sharply painful conglomeration of enlarged lymph nodes can be palpated. The skin over the conglomeration is hyperemic and tense. What etiotropic therapy should be prescribed to this patient?

Explanation

 

Notice that from examination, the mouth is dry, chalk dusted describing a case of oral candidiasis (thrush). This condition is caused by the growth of Candida spp. ( a fungus ) in the oral cavity. An etiotropic therapy is a form of treatment targeted at the causative agent ( in this case the fungus- candida). Of the listed drugs, Ketoconazole is the only antifungal drug; other possible choices include Nystatin, Itraconazole. Ribavirin is an antiviral drug used in hepatitis C, RSV infections etc. Streptomycin is indicated in Tuberculosis and notable for its ototoxic side effect. 

174. A 65-year-old man with acute anterior myocardial infarction developed an asphyxia attack. Examination detected diffuse cyanosis. In the lungs there are numerous heterogeneous wet crackles. Heart rate is 100/min. Blood pressure is 120/100mmHg. What complication occurred in this patient?

Explanation

Note that the key finding here are the numerous wet crackles heard from the lungs; these crackles are heard when there is excess fluid in the airways. Pulmonary edema is usually seen in heart related issues ( in this case post MI); in this condition air sacs in the lungs are filled with fluid making it difficult to breathe. 

Cardiogenic shock can also be caused by Myocardial Infarction but its clinical features include; Tachycardia, Hypotension, Cold/clammy extremities, weak pulse, slow capillary refill etc. 

The blood pressure of this patient is 120/100 mmHg which rules out the option of hypertensive crisis, a condition in which the blood pressure is equal or greater than 180/120mmHg.

 
175. A 48-year-old woman has been hospitalized due to development of tachysystolic atrial fibrillation. She has lost 5 kg of body weight within 2 months. On palpation there is a node in the left lobe of the thyroid gland. What pathology resulted in the development of this condition?

Explanation

The presence of tachysystolic atrial fibrillation and a sudden loss of weight indicates the presence of hyperthyroidism. The presence of nodes on palpation indicates a nodular goiter ( butterfly shaped in the mixed form). A toxic goiter is a form of goiter that causes hyperthyroidism while a non toxic goiter is one that does not cause any hyper/hypothyroidism.
176. A 58-year-old man complains of weakness and tumor-like formations that appeared on the anterior surface of his neck and in the inguinal region. Palpation detects soft painless mobile cervical and inguinal lymph nodes up to 2 cm in diameter. The liver protrudes by 2 cm from the edge of the costal margin, the lower splenic pole is at the umbilical level. In blood: erythrocytes -3.5 • 1012/L, Hb- 88 g/L, leukocytes -86 ■ 109/L, band neutrophils - 1%, segmented neutrophils -10%, lymphocytes -85%, eosinophils - 2%, basophils - 0%, monocytes-2%, erythrocyte sedimentation rate - 15 mm/hour, Gumprecht shadows. What is the most likely diagnosis?

Explanation

Note that from the blood analysis, the key finding is the presence of Gumphrect shadow (Smudge cells). These cells are seen in Chronic lymphocytic leukemia (CLL).

Chronic Myeloid leukemia (CML) is defined by Philadelphia Chromosome ( a translocation between chromosome 9 and 22) coupled with a myeloid stem cell proliferation. 

 
177. A 28-year-old man, a teacher, after an emotional stress developed painful muscle spasms in his right hand that occur during writing; now he has to hold the pen between the second and third fingers. He has no problems with typing or writing on the blackboard; no other motor disturbances or neurological pathologies are detected. What is the most likely diagnosis?

Explanation

Writer’s cramp is a specific form of focal dystonia that affects the fingers, hand, or forearm. Focal dystonia of the hands is a neurological movement disorder.

Cortical agraphia is a neurological disorder in which an individual loses the ability to communicate either through writing or he/she forgets how to spell.

 
178. A 45-year-old woman was hospitalized with complaints of periodical severe headaches against the background of elevated blood pressure up to 180/90 mm Hg, muscle weakness, and frequent urination (at night as well). Her anamnesis states that despite combining various antihypertensive drugs and adjusting their dosage her arterial hypertension cannot be corrected with drugs. The patienfs blood serum potassium levels are 2.0 mmol/L, blood serum sodium levels are 160.0 mmol/L. Ultrasound imaging detects three- dimensional formations approximately 1.0 cm in diameter in the both adrenal glands. Selective endovascular blood sampling from the suprarenal veins was performed, which revealed significant increase of cortisol and aldosterone levels. Make the diagnosis:

Explanation

 

Notice that from ultrasound, this patient presents with a tumor of the adrenal gland. From the results of the blood and biochemical analysis, we see an elevated cortisol, aldosterone coupled with increase in sodium levels - this indicates that the tumor is present in the adrenal cortex ( ruling out the option of Pheochromocytoma which is a tumor of the adrenal medulla and is characterised by elevated catecholamines and blood pressure). This patient has an aldosteroma ( an aldosterone producing adenoma); recall that aldosterone is produced by the zona glomerulosa of the adrenal gland and is responsible for the resorption of sodium and water during formation of urine coupled with deposition of potassium.

179. A 45-year-old man underwent a cardiac surgery one week ago. His general state has been deteriorating since then: dyspnea at rest, retrosternal pain that irradiates to the neck, marked weakness. Objectively his body temperature is hectic. His cardiac borders are expanded, apical beat is weakened. Auscultation detects pericardial friction rub. What is the most likely diagnosis?

Explanation

 

A pericardial friction rub is highly specific for Acute pericarditis. Pericarditis is an inflammation of the pericardium. Acute pericarditis is most commonly caused by viral infection; however, a number of conditions can cause an inflammatory response in the pericardium. Acute inflammation typically manifests with fever, pleuritic chest pain, and a pericardial friction rub on auscultation. The diagnosis is established based on clinical findings, although diffuse ST segment elevations on ECG and imaging may support the diagnosis.

180. A 23-year-old woman without visible cause developed a conflicting behavior at her workplace. She accused the management of underestimating her, claimed that she can be a deputy director, because she speaks four languages, is very attractive, and can make useful connections for the company She has been dressing extravagantly flirting with her colleagues, and singing loudly in her office. In fact, she has only the training of a computer operator and speaks no foreign languages. What is the likely clinical diagnosis?

Explanation

A manic episode is characterized by a sustained period of abnormally elevated or irritable mood, intense energy, racing thoughts, and other extreme and exaggerated behaviors. People can also experience psychosis, including hallucinations and delusions, which indicate a separation from reality. The symptoms of mania can last for a week or more and manic episodes may be spaced within periods of depression during which you may experience fatigue, sadness, and hopelessness.

Schizophrenia is mostly characterised by hallucinations, delusions, catatonic ( disorganised) behaviour and speech.
181. A 45-year-old woman undergoes an inpatient treatment. She complains of elevated body temperature up to 39.0°C, pain in her right lumbar area, turbid urine with blood. CT scan shows an area of low density within the parenchyma, no difference between the cortical and medullary layers, and increased density of the perinephric fat due to edema. What is the diagnosis?

Explanation

 

An abscess is a painful pus filled cavity usually caused by a bacterial infection. Renal abscess are also referred to as Renal corticomedullary  abscesses; the key finding here is that there is no difference between the cortical and medullary layers of the kidney on CT scan which proves that the cortex and medulla are uniformly affected; also, we see edema and increased density of perinephric fat  which occurs due to the accumulation of fluid.

182. A 56-year-old woman was diagnosed with stage 2 hypertension of the 2nd degree. She belongs to the group of moderate risk and has bronchial asthma. What group of drugs is CONTRAINDICATED to this patient?

Explanation

 

The groups of drugs mostly used in the management of hypertension include; ACE inhibitors, ARBs, calcium channel blockers , Beta blockers, diuretics etc. Recall that Beta-blockers are drugs that bind to beta-adrenoceptors and thereby block the binding of norepinephrine and epinephrine to these receptors. This inhibits normal sympathetic effects that act through these receptors.  Beta receptors are not only found in the heart ( beta 2) but also in the lungs ( beta 1); blocking of these receptors will cause the heart to slow down but have increased force of contraction, in the lungs- Bronchoconstriction. Bronchoconstriction is very dangerous in a patient with asthma thus NON- SELECTIVE BETA BLOCKERS eg Atenolol are contraindicated. 

183. A 68-year-old woman with congestive heart failure and left ventricular ejection fraction of <40.% receives the following pharmacotherapy scheme: ramipril, torasemide, bisoprolol, clopidogrel, and digoxin. During one of her regular examinations, frequent polymorphic ventricular extrasystoles were detected in the patient. What medicine should be removed from her therapy scheme?

Explanation

 Although Digoxin helps in improving the ejection fraction in a patient with Heart failure, Notice that frequent polymorphic ventricular extrasystoles were detected. These polymorphic ventricular extrasystoles are usually caused by an elevation in calcium influx into myocardial cells. Recall that the mechanism of action of Digoxin involves the reversible inhibition of the Na-K ATPase enzyme, leading to various beneficial effects. The Na-K ATPase enzyme functions to maintain the intracellular environment by regulating the entry and exit of sodium, potassium, and calcium (indirectly). Na-K ATPase is also known as the sodium pump. The inhibition of the sodium pump by digoxin increases intracellular sodium and increases the calcium level in the myocardial cells, causing an increased contractile force of the heart. This improves the left ventricular ejection fraction (EF), an important measure of cardiac function. Therefore in order to stop these extrasystoles, Digoxin should be excluded from the therapy.
184. After a long drive with the window open a man developed a facial asymmetry; he cannot close his right eye, his right nasolabial fold is smoothed out, movements of expression are absent on the right, there is a disturbance of taste sensation in the tongue on the right. No other neurological abnormalities were detected. What disease can be provisionally diagnosed in this case?

Explanation

Facial (nerve) palsy is a neurological condition in which function of the facial nerve (cranial nerve VII) is partially or completely lost.  Key findings include; Sensory disturbances : Painful sensation around or behind the ear, Impairment of taste in the anterior tongue , Hyperacusis; dropping of the mouth, dry mouth, Bell\'s phenomenon: a physiologic, reflexive movement of the eye (upward and outward) that occurs when the eyelid is actively closed, Lagophthalmos: The patient cannot fully close the eyes (due to paralysis of the orbicularis oculi muscle). Decreased lacrimation.

In neuropathy of the trigeminal nerve ( trigeminal neuralgia), the individual will feel excruciating pain at the slightest pressure applied on the face e.g. while brushing the teeth, yawning etc.

In neuropathy of the oculomotor nerve, the individual will experience diplopia and strabismus.

185. A 73-year-old woman came to the family physician for one of her regular follow-up examinations. Three months ago she was found to have type 2 diabetes mellitus. She was keeping to her diet and exercise plan and taking phytopreparations. On examination her fasting glucose was within the range of 78-8.6 mmol/L, HbAlc - 7.9%. Height - 164 cm, weight - 83 kg. What blood glucose-controlling medicine should she be prescribed first in the course of her pharmacological therapy?

Explanation

 

This patient has been diagnosed with type 2 diabetes mellitus ( non-insulin dependent diabetes). The results from the current blood sugar level and glycated hemoglobin supports it ; Norm blood glucose 3.5-5.5 mmol/L; normal glycated hemoglobin -6 %. For the treatment of type 2 Diabetes, Metformin also known as Glucophage (a biguanide) is usually the first drug to be administered.

186. A patient, has gradually lost his consciousness. The skin is pale and dry. There is smell of ammonia from the mouth. Respirations are deep and noisy. Heart sounds are muffled, pericardial friction rub is present. Blood pressure is 180/130 mm Hg. Blood test: Hb- 80 g/L, leukocytes - 12 . 109/L, blood glucose - 6.4 mmol/L, urea -50 mmol/L, creatinine - 1200 mcmol/L, blood osmolarity - 350 mOsmol/kg H2O. No urinary excretion. Make the diagnosis:

Explanation

 

Recall that Ammonia is converted to urea ( via the ornithine/urea cycle). Urea is then excreted from the body through the urine. Notice that this patient has no urinary output, smell of ammonia from the mouth coupled with an increased urea level in the blood. Recall that ammonia and urea (at high blood concentration like in this case) can cross the blood brain barrier leading to encephalopathy and further loss of consciousness. N/B the blood glucose and osmolarity levels are also elevated but not enough to lead to their respective forms of coma.

187. A 54-year-old woman complains of a fogged vision in her right eye, rainbow circles in her vision, headache, and nausea. Within the last month she twice experienced a similar condition, but back then all the signs eventually disappeared and her sight was restored. Currently all the signs have beed persisting for over 2 days. Objectively the patient has eyelid edema, congestive injection of the eyeball, corneal opacity shallow anterior chamber of the eye, and dilated pupil that is unresponsive to the light. Her intraocular pressure is 48 mm Hg. Make the diagnosis:

Explanation

Observe that the intraocular pressure is 48mmHg ( norm is 10-21 mmHg). Glaucoma is a medical condition characterised by optic disc atrophy coupled with cupping, usually with an elevated intraocular pressure and progressive peripheral  visual field loss if untreated.

Iridocyclitis is an inflammation of the anterior uveal tract ( iris and ciliary body) ; symptoms include blurred vision, eyes pain, photosensitivity, red eye ( conjunctival injection) etc.

 
188. After eating shrimps, a 25-year-old man suddenly developed skin itching, some areas of his skin became hyperemic or erupted into vesicles. Make the diagnosis:

Explanation

Urticaria (hives) is a vascular reaction of the skin marked by the transient appearance of smooth, slightly elevated papules or plaques (wheals) that are erythematous and that are often attended by severe pruritus. Individual lesions resolve without scarring in several hours. Most cases of urticaria are self-limited and of short duration; the eruption rarely lasts more than several days, but may be recurrent over weeks. Chronic urticaria is defined as urticaria with recurrent episodes lasting longer than 6 weeks. Mostly occurs as a reaction towards food, drugs, contact etc.

Urticaria pigmentosa is composed of persistent brown or red marks, made of collections of mast cells that swell and itch transiently when rubbed, similar to a hive.

 
189. A patient is 45 years old. He was referred for a consultation with a psychiatrist due to complaints of abdominal pain and discomfort that occur in emotionally straining situations. Objectively, no changes of the gastrointestinal tract were detected. The complaints emerged over 10 years ago against the background of severe alcohol poisoning. The patient has been repeatedly visiting gastroenterologists, who were unable to find any significant changes in the patient. The prescribed therapy was ineffective. What is the likely conclusion?

Explanation

In Somatoform Autonomic dysfunction, the symptoms are presented by the patient as if they were due to a physical disorder of a system or organ that is largely or completely under autonomic innervation and control, i.e. the cardiovascular, gastrointestinal, respiratory, and  urogenital systems. Clinical and instrumental examination revealed no organic alterations in any system therefore indicating a somatoform autonomic dysfunction. 

Organic brain syndrome is defined as a state of diffuse cerebral dysfunction associated with a disturbance in consciousness, cognition, mood, affect, and behavior in the absence of drugs, infection, or a metabolic cause.

 
190. A 21-year-old man was hospitalized on the 2nd day of the disease. His general condition is severe, body temperature is 39°(7, On his skin there are numerous irregular-shaped hemorrhagic elements. The diagnosis of meningococcemia was made. The next day his body temperature suddenly decreased, blood pressure was 80/40 mm Hg, pulse was 120/min. Acrocyanosis was detected. What complication did the patient develop?

Explanation

191. A 45-year-old man with a history of myocardial infarction developed a brief attack of palpitations, accompanied by the sensations of lack of air, fear, and vertigo. His blood pressure is 90/60 mm Hg. ECG during the attack shows extended QRS complex (0.13 seconds) with heart rate of 160/min., discordant shift of ST segment and T wave, dissociation of atrial dtid ventricular rhythm. What disturbance of cardiac rhythm is it?

Explanation

Ventricular fibrillation (VF) and myocardial ischemia are inseparable. Ventricular fibrillation most commonly occurs during an acute myocardial infarction or shortly thereafter. When heart muscle does not get enough blood flow, it can become electrically unstable and cause dangerous heart rhythms.

ECG Findings

  • Fibrillation waves of varying amplitude and shape.

  • No identifiable P waves, widened or prolonged QRS complexes (norm: 0.12sec), or T waves

  • Heart rate anywhere between 150 to 500 per minute

 

Furthermore, Atrioventricular dissociation occurs when P waves, representing atrial depolarization, are seen at different rates than the QRS complexes. This is present in only a small percentage of Ventricular fibrillation ECG tracings, but it is diagnostic of VF. Frequently, it is difficult to identify the P waves due to the fast rate of the QRS complex.

192. A 23-year-old woman with type 1 diabetes mellitus during the 2nd week of community-acquired pneumonia developed nausea and vomiting. In the evening she has lost her consciousness and was hospitalized. Objectively the patient\'s skin is pale and dry Her respiration is loud, the tongue is dry with brown deposit. Her heart rate is 129/min., blood pressure is 85/50 mm Hg. Palpation of the patient\'s abdomen provokes no response. The liver is +3 cm. Acetone test is markedly positive, blood glucose is 26 mmol/L. Make the provisional diagnosis:

Explanation

Notice that this patient presents with a type 1 diabetes ( insulin dependent). In the absence of insulin, the body tends to release more glucose from the liver and free fatty acids from adipose tissue; these free fatty acids are then broken down into ketone bodies (acetoacetate and β-hydroxybutyrate) through the process of beta oxidation. Accumulation of these ketone bodies in the blood leads to metabolic acidosis due to the fact that ketone bodies reduce the blood pH level. The breath of a person with Diabetic ketoacidosis is usually associated with a specific smell ‘Fruity’ or “acetone”.

A diabetic hyperosmolar coma is caused by severe dehydration and very high blood glucose levels (hyperglycaemia). The kidneys respond to high levels of blood glucose by doing their best to remove it, along with a great deal of water. The person experiencing diabetic hyperosmolarity will be very thirsty coupled with a high level of osmolarity level from biochemical analysis. 

Lactic academic coma occurs as a result of a buildup of lactic acid in the blood.

   
193. On ultrasound of the thyroid gland, a 47-year-old woman presents with a hypoechoic node 1.6 cm in diameter with blurred margins and intranodular hypervascularization. The doctor suspects thyroid carcinoma. What method should be used to verify the diagnosis?

Explanation

A thyroid fine needle aspiration biopsy is a procedure that removes a small sample of tissue from your thyroid gland. Cells are removed through a small, hollow needle. The sample is sent to the lab for analysis. In some cases, hard nodules form inside the gland. Most times, the nodules are not dangerous. But in some cases they can be thyroid cancer. A thyroid fine needle aspiration biopsy can take a sample from the nodule to test for cancer.

Thyroid scan (thyroid scintigraphy) is a nuclear medicine examination used to evaluate thyroid tissue. Clinical indications include; functional status of a thyroid nodule, thyrotoxicosis: differential diagnosis and thyroid cancer. Note that an ultrasound procedure has already been carried out therefore, carrying out a thyroid scintigraphy will just be a repetitive test.

 
194. A 20-year-old man suffers from headache, general weakness, and face edema that appears in the morning. 18 days earlier he had a case of tonsillitis. Objectively his skin is pale, there are edema under his eyes. Heart rate is 60/min., blood pressure is 185/100 mm Hg, The sign of costovertebral angle tenderness (punch sign in the lumbar region) is negative. Urinalysis: color of “meat slops” protein - 4.5 g/L, altered erythrocytes - 40-45 in the vision field, leukocytes - 5-6 in the vision field. 24- hour diuresis is 400mL. What is the most likely diagnosis?

Explanation

 

Note that 18 days prior, this patient had a case of tonsillitis; this raises the idea of a group A beta hemolytic streptococci infection. This infection is most likely to cause glomerulonephritis (post streptococcal glomerulonephritis). Poststreptococcal (or post-infectious) glomerulonephritis (PSGN) refers to acute glomerular inflammation that results from a preceding infection with nephritogenic strains of streptococci. Deposition of immune complexes containing the streptococcal antigen within the glomerular basement membrane results in complement activation and subsequent damage to the glomeruli. PSGN typically presents as a nephritic syndrome with hematuria, mild proteinuria, edema, and hypertension. Elevated antistreptolysin O titers (ASO), low complement levels, and elevated creatinine support the diagnosis. 

195. An 18-year-old patient always obeys others and adapts his needs to the demands of the people on whom he depends. He excessively defers to their wishes and makes them responsible for his wellbeing, cannot defend his interests and needs support from other people. Such psychic profile has been formed in the childhood, remains unchanged, and hinders adaptation. What psychic disorder is observed, in this patient?

Explanation

People with dependent personality disorder may submit to the will of others in a misguided attempt to extract a promise of care and protection. They may think of themselves, or present themselves, as unable to cope with everyday life on their own. At the same time, they may fear that a show of confidence or competence will lead to rejection and abandonment. They demand advice and reassurance when making even minor decisions. They take no initiative and let others assume responsibility for their lives.

Anankastic personality disorder is also  known as Obsessive compulsive disorder (OCD),a disorder characterized by preoccupation with orderliness, perfectionism, and mental and interpersonal control, at the expense of flexibility, openness, and efficiency.

 
196. A 35-year-old woman complains of a pain in her right axillary region. She has been suffering from this condition for a week. Her body temperature is 38°C. In the right axillary region there are 2 formations, 2 cm in size each. The skin over the formations is dark red and thin. Palpation produces a yellow-white discharge from the fistular openings. What is the most likely diagnosis?

Explanation

Carbuncle: red, swollen, painful clusters of boil that are connected to each other under the skin. 

Furuncle: refers to a Boil, forms around the hair follicle and contains pus.

Hydradenitis: ‘acne inversa’ , inflamed and swollen lumps found in the axillary region ( armpit), under the breast, groin. These lumps most times break open forming a fistula or tunnels

 
197. During a regular examination, an 8- year-old girl with type I diabetes mellitus presents with a swelling on the anterior surface of her hip. The swelling is 3 cm in diameter; dense, painless on palpation. The skin over this formation has normal color and temperature. Localization of the swelling matches the place where the girl usually receives her insulin injections. What is the most likely cause of this clinical presentation?

Explanation

Lipohypertrophy Insulin Injection   Lipoatrophy Insulin Injection

The background information here is that - this young lady with Type I Diabetes Mellitus presents with a 3cm swelling on the anterior surface of the hip. 

Abnormal reaction in subcutaneous fat to insulin is called lipodystrophy which can be either hypertrophic or atrophic. Lipodystrophy is an exclusive complication of lean children and young Type 1 diabetic, although rarely can be seen in Type 2 diabetic.

Insulin lipohypertrophy denotes a benign tumor like swelling of fatty tissue at the injection site secondary to lipogenic effect of insulin, whereas lipoatrophy is considered an adverse immunological side effect of insulin therapy. Since the advent of recombinant human insulin and analogue, lipoatrophy, has virtually disappeared, whereas, lipohypertrophy still remains a serious local problem of insulin therapy.

With a swelling of 3cm, the best answer choice is development of hypertrophic lipodystrophy. Atrophic lipodystrophy will not have associated swelling, instead a depression should be observed.

 

Prevention, to avoid lipodystrophy, should take first place as there is little cure to it, and the best way is to educate the patient about rotating injection sites.

198. A 52-year-old woman has been suffering for 2 years from dull, occasionally exacerbating pain in her right subcostal area, occurring after eating high-fat foods, bitter taste in her mouth in the morning, constipations, and flatulence. Objectively she is obese, her body temperature is 36.9°C; there is a coating on the root of her tongue; the abdomen is moderately distended and painful in the area of gallbladder projection. What examination would be the most helpful for diagnosis making?

Explanation

 

Notice that this patient presents with signs in relation to gallbladder or liver pathology (bile in general). In order to get the correct diagnosis, an Ultrasound of the liver, gall bladder and surrounding abdominal organs should be carried out.

199. What should be prescribed as secondary prevention drugs for a patient with atrial fibrillation after an ischemic stroke caused by cardiac embolism?

Explanation

Dabigatran, rivaroxaban, apixaban, and edoxaban (direct acting oral anticoagulants) are approved for the lowering the risk of stroke and embolism in NVAF as well as deep vein thrombosis and pulmonary embolism treatment/prophylaxis. Unique indications include betrixaban for prophylaxis of venous thromboembolism (VTE) in hospitalized patients for an acute medical illness, and rivaroxaban in combination with aspirin to reduce major cardiovascular events in patients with chronic coronary artery disease (CAD) or peripheral artery disease.
200. A 55-year-old woman complains of thyroid gland enlargement that can be observed throughout the last 2 years and a discomfort during swallowing. Objectively she has signs of hypothyroidism. The thyroid gland on palpation is dense, non-fused with the surrounding tissues and mobile on swallowing. The regional lymph nodes are not enlarged. In the serum there are antithyroid antibodies detected. What is the most likely diagnosis?

Explanation

 

The presence of antithyroid antibodies helps us understand that there is an autoimmune attack on the thyroids- This refers to Hashimoto’s thyroiditis; the most common cause of hypothyroidism. It is characterised by lymphocyte infiltration of the stroma of the thyroid gland leading to its destruction and signs of hypothyroidism. Other antibodies that can be found include those against thyroid peroxidase and TSH receptors. For its treatment, a replacement therapy is carried out using L-thyroxine.

201. A 19-year-old young man complains of cough with expectoration of purulent sputum in amount of 100 mL per day hemoptysis, dyspnea, increased body temperature up to 378°C, general weakness, weight loss. The patient\'s condition lasts for 4 years. Exacerbations occur 2-3 times a year. The patient presents with malnutrition, pale skin, cyanosis of the lips, drumstick (clubbed) fingers. Tympanic percussion sound in the lungs, weakened respiration, and various numerous moist crackles in the lower pulmonary segments on the left can be observed in this patient. In blood: erythrocytes - 3.2 • 1012/L,leukocytes -8.4 • 109/L,ESR- 56 mm/hour. On X- ray: lung fields are emphysematous, the left pulmonary root is deformed and dilated. What is the most likely diagnosis?

Explanation

Bronchiectasis is a condition in which damage to the airways causes them to widen and become flabby and scarred (X-ray findings - left lung root is deformed and dilated). The airways are tubes that carry air in and out of your lungs.

Bronchiectasis usually is the result of an infection or other condition that injures the walls of your airways or prevents the airways from clearing mucus.

The initial airway damage that leads to bronchiectasis often begins in childhood. However, signs and symptoms may not appear until months or even years after you start having repeated lung infections.

The most common signs and symptoms of bronchiectasis are:

  • A daily cough that occurs over months or years.

  • Daily production of large amounts of sputum (spit). Sputum, which you cough up and spit out, may contain mucus (a slimy substance), trapped particles, and pus.

  • Shortness of breath and wheezing (a whistling sound when you breathe).

  • Chest pain.

  • Clubbing (the flesh under your fingernails and toenails gets thicker).

 

Over time, it may progress to more serious symptoms like coughing up blood (hemoptysis) or bloody mucus and feel very tired. Children may lose weight or not grow at a normal rate.

202. A 58-year old woman has type 2 diabetes mellitus that is compensated with diet and metformin. She prepares for cholecystectomy. Objectively, her height is 164 cm, weight is 90 kg, heart rate is 72/min., blood pressure is 130/80 mm Hg. Her abdomen is soft, painful in the right subcostal region. The liver is not enlarged. Fasting glucose - 6.2 mmol/L. Glycated hemoglobin - 6.5%. What further tactics of sugar-lowering therapy should be chosen in this case?

Explanation

Notice that this patient’s blood glucose and glycated hemoglobin levels are marginally high; she has also been on a diet regime coupled with metformin intake. On the basis that she is being prepared for a cholecystectomy ( a surgical procedure), she needs to be administered a short acting insulin  ( short acting just to last the period of the surgical process).
203. A 43-year-old man complains of facial edema, dyspnea, and difficult swallowing that appeared 3 hours after he was stung by an insect. Objectively the skin of his eyelids, cheeks, and lips is acutely hyperemic and edematous. What medicines should he be prescribed for emergency aid?

Explanation

 

This patient is having an Anaphylactic reaction after being stung by a bee. Notice that he is having inflammation of the airway, making it difficult to breath. For emergency care , this patient should be administered Intravenous epinephrine and glucocorticoids to reduce inflammation and help improve breathing. 

204. A 48-year-old woman was delivered into the surgical unit with wounds in her thigh. On examination the wound surface has a dirty-gray coating with unpleasant sweet smell. The wound content resembles a raspberry jelly. Skin tissues around the wound are glossy and turgid. Palpation reveals moderate crepitation in the tissues. What bacteria is the most likely to cause such inflammation?

Explanation

From the description; dirty gray coating with unpleasant sweet smell, resembling raspberry jelly with moderate criterion on palpation - this fits the description of a Gangrenous infection (gangrene); an infectious process caused by Clostridium Perfringens. Cl. Perfringens is a gram positive, rod shaped, spore forming anaerobic bacteria which is a major cause of food poisoning and gas gangrene.

Blue pus bacillus refers to pseudomonas aeruginosa.

 
205. A 26-year-old man complains of chills, rhinitis, dry cough, and fever up to 38°C. Examination shows him to be in a moderately severe condition; there are small pale pink non-merging spots on the skin of his back, abdomen, and extremities. Palpation reveals enlarged occipital and axillary lymph nodes. No information about vaccination history could be obtained. What is the likely etiology of this disease?

Explanation

Rubella ( accused by rubella virus) is an acute viral infection characterised by a short prodromal period, Exanthema ( rash) during three days and lymphadenopathy. The type of rash found is roseola and small macula-papula. Note that the rash elements do not merge ( non-merging spots) and are usually concentrated on the extensor surface of extremities, back, buttocks and outer surface of thigh.

Epstein-barr virus is  one of the causative agents of Infectious mononucleosis- a disease characterised by prolonged fever, systemic lymphadenopathy,acute tonsillitis, acute adenoiditis, hepatosplenomegaly, along with typical blood changes  such as lymphocytosis, monocytosis, presence of specific cells called atypical mononuclear cells or virocytes. 

Mumps infection is a systemic disease characterised by hyperthermic syndrome,symmetrical or unilateral swelling of parotid glands, epididymo-orchitis etc.

 
206. A 38-year-old patient has been delivered by an ambulance to a surgical department with complaints of general weakness, indisposition, black stool. On examination the patient is pale,there are dotted hemorrhages on the skin of his torso and extremities. On digital investigation there are black feces on the glove. Blood test: Hb-108 g/L, thrombocytopenia. Anamnesis states that similar condition was observed 1 year ago. Make the diagnosis:

Explanation

Immune thrombocytopenic purpura (ITP) is a clinical syndrome in which a decreased number of circulating platelets (thrombocytopenia) present as a bleeding tendency, easy bruising (purpura), or extravasation of blood from capillaries into skin and mucous membranes (petechiae). Recall that the patient presents with dotted hemorrhages on the skin (petechiae) and on analysis, thrombocytopenia. Caused by the binding of an autoantibody (specifically IgG) to platelets leading to platelet destruction.

In Hemophilia, patients will present with bleeding from joints (hemathroses), easy bruising or a case of bleeding after surgery, dental procedure. Has three forms; Hemophilia A ( Factor VIII deficient), Hemophilia B ( factor IX deficient), C ( factor XI def.).

 
207. A  27-year-old man complains of pain in his leg joints, purulent discharge from the eyes, and painful burning sensations during urination. The disease onset was acute. He has a history of influenza. The patient smokes and drinks alcohol in excess. In his line of work he is often away on business trips. What is the most likely etiological factor of this disease?

Explanation

 

This patient presents with Classical signs of Reactive arthritis ( He can’t See , Pee or Bend The Knee) ie, purulent discharge from the eyes, burning sensation while urinating and pain in his leg joints. Reactive arthritis is an autoimmune condition that develops in response to an infection in another part of the body. Coming into contact with bacteria and developing an infection can trigger reactive arthritis. Chlamydia trachomatis is the most common bacteria that triggers Reactive arthritis.

208. A 57-year-old man, a miner, complains of a pain in his chest, dyspnea on physical exertion, excessive sweating, constant subfebrile tempeature, and cough that produces blood-streaked sputum. He has been smoking for approximately 40 years (2 packs a day) and frequently has “pneumonias”. Survey chest X-ray shows a triangular shadow in the middle lobe of the right lung. One of the apices of the shadow points to the lung root. Cardiac and mediastinal shadows are displaced toward the affected area. Make the provisional diagnosis:

Explanation

209. A 31-year-old drug-addicted person complains of a cough with bloody expectorations, dyspnea, persistent fever, and leg edemas. The jugular veins are distended. There is a coarse pansystolic murmur detected above the base of the xiphoid process and in the second intercostal space on the left, close to the edge of the sternum. Heart sounds are clear; arrhythmia is detected, heart rate is 128/min., pulse - 82/min., blood pressure is 100/70 mm Hg. What is the most likely diagnosis?

Explanation

Infective endocarditis (IE) is an infectious inflammation of the endocardium that affects the heart valves. The condition is a result of bacteremia, which is most commonly caused by dental procedures, surgery, distant primary infections, and nonsterile injections. From anamnesis, we understand that this patient is a drug addict and most likely uses non sterile injections. Clinical features include constitutional symptoms (fatigue, fever/chills, malaise) in combination with signs of pathological cardiac changes (e.g., new or changed heart murmur, heart failure signs). Notice that this patient has a pansystolic (holosystolic) murmur (usually heard in tricuspid valve regurgitation which can be one of the cardiac changes in IE). 

Lutembacher syndrome is a very rare disorder that involves a congenital atrial septal defect coupled with an acquired mitral stenosis.

 
210. A 17-year-old girl has been suffering from hepatic cirrhosis for 3 years. Lately her periods of excitation have been intermittent with depression, she does not sleep enough. Objectively her condition is severe, the girl is sluggish, gives one-word responses, has tremor in her extremities, her skin is icteric, with single hemorrhagic rashes. Name the likely complication of her disease:

Explanation

Recall that one of the major functions of the liver is detoxification i.e., converting harmful metabolites in the body into less harmful products that can be easily excreted out eg Ammonia is converted in the liver to urea which is then excreted via the urine. In  a case of hepatic cirrhosis, this function  of the liver is affected leading to the accumulation of harmful metabolites in the body system. Ammonia itself can easily cross the blood brain barrier causing encephalopathy. Notice that this patient presents with signs that suggest a reduction in mental activities. 

Reye syndrome is a rare form of acute encephalopathy and fatty infiltration of the liver that tends to occur after some acute viral infections, particularly when salicylates (aspirin) are used. 

 
211. A 34-year-old woman after rapidly changing her position from horizontal to vertical suddenly paled, fell down, her skin became moist, her limbs are cold, her pupils are dilated. The pulse is rapid and thready blood pressure is 50/25 mmHg. What condition has likely developed in the patient?

Explanation

This patient just experienced an Orthostatic syncope or collapse. She was in the horizontal position (lying down) and when she rose to the vertical (upright position) she fell down. Orthostatic syncope refers to syncope resulting from a postural decrease in blood pressure. Classic orthostatic hypotension occurs when there is a persistent reduction in blood pressure of at least 20mmHg systolic or 10mmHg diastolic within 3 minutes of standing or being upright.

When the body assumes an upright position, there is an immediate gravitational pooling of about 500 to 1000ml of blood to the lower extremities, splanchnic and pulmonary circulations. The decrease in venous return to the heart reduces cardiac output and eventually causes a drop in blood pressure. Baroreceptors in the carotid and aortic arteries sense this decrease in blood pressure and activate the sympathetic nervous system which leads to increased heart rate, systemic vasoconstriction, and increased cardiac muscle contractility all of which eventually increase blood pressure.

 

In a healthy individual, this sympathetic activation causes a physiological increase in heart rate by 10 to 20 bpm, diastolic blood pressure by 5mmHg, but minimal to no change in systolic blood pressure. In patients with autonomic dysfunction, there is an inadequate engagement of the autonomic nervous system in response to a decrease in blood pressure leading to persistent hypotension and in serious conditions just as we have here, it can result in collapse.

212. A woman complains of a severe pain in her throat on the left,difficult swallowing and mouth opening, elevated body temperature, and general malaise. The onset of the disease was 4 days ago after a case of tonsillitis. Examination detects a trismus of the masticatory muscles, the left tonsil is displaced toward the midline, the anterior palatal arch is infiltrated and protrudiiig. The regional lymph nodes on the right are enlarged and painful on palpation. Make the diagnosis:

Explanation

 

A peritonsillar abscess is a bacterial infection that usually begins as a complication of untreated strep throat or tonsillitis (4 days ago, the patient had a case of tonsillitis). also known as quinsy, it leads to the  accumulation of pus due to an infection behind the tonsil. Key symptoms include; throat pain, difficulty swallowing , drooling , swelling of the face or neck, fever and chills. Notice that this patient presents with signs affecting the TONSILS; in a case of Pharyngitis, the posterior pharyngeal wall will be affected. In a case of infectious mononucleosis, Atypical lymphocytes will be present on analysis.

213. During winter epidemics of influenza caused predominantly by virus A/California/04/2009 (H1N1), on the 2nd day after the disease onset a 30-year-old hospitalized man presented with high fever, dry cough, myalgia, headache, and general weakness. What should be prescribed as an etiotropic treatment in this case?

Explanation

An etiotropic treatment is one aimed at the causative agent. In this case a Virus - Influenza A. Oseltamivir is an antiviral drug aimed at the treatment of influenza A and B. It acts by inhibiting influenza neuraminidase ( an enzyme needed for influenza replication). Zanamavir also has this function.

Acyclovir is also an antiviral drug used mainly in treatment of the herpes Simplex virus and Varicella zoster virus.

 
214. A 30-year-old woman complains of itching skin, predominantly in the evening and at night. The condition lasts for 2 weeks already On the skin of the interdigital folds, mammary glands, abdomen, buttocks, and thighs there are numerous fine papular and papulovesicular rashes located in pairs, excoriations. There is no rash on the face and neck. Similar rash is observed in the husband of the patient. What is the most likely diagnosis?

Explanation

Scabies is a parasitic skin infestation caused by the Sarcoptes scabiei. It is characterised by intense pruritus (itching) at night; this characteristic and the location of the rashes helps us to diagnose scabies. The primary lessios found are usually papules, vesicles or burrows ( accompanied by excoriations and pustules) which are commonly seen in interdigital folds, flexor surfaces of the wrist, axillary folds, buttocks (closed areas of the body). Additionally in children, elderly persons, and immunosuppressed patients: scalp, face, neck, under the nail, palms of hands, and soles of feet.

Neurodermatitis is also referred to as Lichen simplex chronicus; it is chracterised by chronic itching and scaling localised on the wrist, neck, forearm, legs and anal region. 

Epidermophytosis is referred to as ‘Athlete’s foot’.
215. A 20-year-old patient complains of a severe headache, double vision, weakness, fever; irritability Objectively: body temperature is 38.1°C, the patient is reluctant to contact, sensitive to stimuli. There are ptosis of the left eyelid, exotropia, anisocoria S > D, and pronounced meningeal syndrome. On lumbar puncture the cerebrospinal fluid flowed out under a pressure of 300 mmHg, the fluid is clear, slightly opalescent. 24 hours later there appeared a fibrin film. Protein - 1.4 g/L, lymphocytes - 600 per mm3, glucose - 0.3 mmol/L. What is the provisional diagnosis?

Explanation

Meningitis is an acute infectious disease with involvement of the arachnoid and pia of the brain and spinal cord by pathogenic microorganisms. Etiologically, the various forms include; bacterial, viral. Fungal and tuberculous meningitis. The tuberculous form occurs as a secondary infection; The CSF is under increased pressure, is clear or slightly cloudy, and contains a predominance of mononuclear cells (usually >400/mm3), increased protein (100 to 400 mg/dL), and a decreased glucose content.  Recall that a bacterial meningitis will have a predominance of neutrophils while a viral form will be predominated by lymphocytes in CSF count.
216. A 75-year-old man in a severe condition suffers from dyspnea at rest, marked weakness, and arrhythmia. Abdominal aortic pulsation is observed, further on there is a systolic murmur detected. Palpation reveals a volumetric formation in the mesogastrium. Blood pressure is 70/40 mm Hg. There is no pulsation over the femoral arteries. Oliguria is detected. Which diagnosis is the correct one?

Explanation

An aortic aneurysm is a localized pathologic dilation of the aorta; may cause abdominal and/or back pain which is a sign of  leaking, dissection or imminent rupture. This patient is most likely having an aneurysm of the abdominal aorta which usually presents as a palpable abdominal pulsatile mass - Notice that in this patient, abdominal pulsation is observed coupled with a volumetric formation in the mesogastrium.

Acute cardiac aneurysm or a Cardiosclerotic aneurysm are common complications of a recent Myocardial Infarction. It refers to ventricular aneurysms which leads to decreased contractility of the heart and reduction of the flow of oxygen-rich blood to the body, causing heart failure and death. A cardiac aneurysm will rather show retrosternal symptoms instead of a mesogastric one as seen in this case.

 
217. A dweller of the northern Dnieper area, a fisherman, for the last several days has been complaining of a discomfort in his right subcostal region, periodical episodes of diarrhea, intermittent with constipations, frequent skin rashes. Abdominal ultrasound shows enlarged liver and pancreatic head. Make the provisional diagnosis:

Explanation

From anamnesis, the patient is a fisherman by profession- with this we can assume that he consumes his primary product ( fish).  Opisthorchiasis is a trematode (fluke) infection caused by infection with one of the species of the liver fluke Opisthorchis, which is acquired by eating raw or undercooked freshwater fish containing infectious metacercariae. The three species are: O. sinensis (still widely known as Clonorchis sinensis), O. felineus/tenuicollis and O. viverrini. Mostly asymptomatic but can present with dyspepsia, abdominal pain, diarrhoea or constipation. In the long term, we begin to observe hepatomegaly and malnutrition. 

Trichinellosis is gotten after consumption of meat ( especially pork) infected by Trichinella spp. In this disease, the larva  enters into the bloodstream and encysts in striated muscles leading to myositis ( a key finding).

The adult form of Taenia solium and T. saginata cause taeniasis, T. solium larvae cause cysticercosis. They are differentiated by the number of proglottids/ uterine branches 7-12 for T. solium and 17-35 for saginata.

 
218. An 18-year-old patient complains of skin rash. The patient has been suffering from this condition for 5 years. The first instance of this disease occurred after a car accident. Objectively: the patient presents with a papular rash covered in silvery scales, “thimble” sign (small pits on the nails), affected joints. What is the most likely diagnosis?

Explanation

Psoriasis is a common skin disorder characterized by the presence of papules and plaques with silvery scaling especially on the knees and elbows. Other characteristics include acanthosis, parakeratosis and pinpoint bleeding (Auspitz sign). The patient presents with a thimble symptom which translates into a pitting or a psoriatic nail.

A Panaritium or Whitlow is an acute purulent inflammation of the tissues of the finger or toes. Onychomycosis is a fungal infection of the nails; signs are white or yellow nail discoloration, thickening of the nail, and separation of the nail from the nail bed; it is also known as  tinea unguium.

 
219. During regular medical examination a lyceum student presents with signs of cheilitis that manifests as epithelial maceration in the area of lip seal. The lips are bright-red, with single vertical cracks covered with brown-red scabs. These clinical signs are most likely caused by insufficient content of the following in the diet:

Explanation

The signs and symptoms of riboflavin (vitamin B2) deficiency typically include sore throat with redness and swelling of the mouth and throat mucosa, cheilosis and angular stomatitis (cracking of the lips and corners of the mouth), glossitis, seborrheic dermatitis or pseudo-syphilis, and a decreased red blood cell count with normal cell size and hemoglobin content (normochromic normocytic anemia).

Thiamine (vit B1) deficiency is known as Beri-Beri, deficiency of ascorbic acid will lead to scurvy, easy bruising and bleeding are seen in such patients.

 
220. A 72-year-old man with pneumonia complains of marked dyspnea, chest pain, severe cough with expectoration, t° is 39.5-40°C, no urination for the last 24 hours. Objectively the patient is conscious. Respiratory rate is 36/min. Percussion sound is dull over the right lower pulmonary lobe; on auscultation, there is bronchial respiration and numerous moist crackles. Blood pressure is 80/60 mm Hg, Heart rate is 120/min. Heart sounds are muffled, there is tachycardia. What tactics should the family doctor choose for the management of this patient?

Explanation

 

Observe that this patient presents pneumonia, high grade fever, low blood pressure and has been unable to urinate for the past 24 hours (which could be due to renal failure). This patient should be admitted to the intensive care unit; a urinary catheter should be inserted into the urethra in order to drain out the accumulated urine. After rectifying the issue of urinary flow, he can then be transferred to the pulmonology unit.

221. A 48-year-old man came to a doctor with complaints of vomiting that brings no relief and a burning pain in his left subcostal region that irradiates to the left lumbar region. These signs appeared after a meal. The Ortner’s and Mayo-Robson’s signs are positive. In the blood: leukocytosis and increased ESR. In the urine: elevated diastase levels. Make the diagnosis:

Explanation

Ortner’s sign- tenderness on light percussion on right costal margin by the edge of the palm

Mayo-Robson’s signs- pain while pressing at the top of the angle lateral to the erector spinae muscles and below the left 12th rib left costovertebral angle.

These signs are present in a patient with pancreatitis and can be confirmed by the elevation of Diastase level. Urine diastase is useful in diagnosing uncertain abdominal cases (especially when pancreatitis is suspected).

 
222. A 32-year-old woman complains of general weakness, low-grade fever persisting for 4 months, lumbar pain, and dysuria. Anamnesis includes frequent acute respiratory diseases, overexposure to cold, low-calorie diet, a case of pulmonary tuberculosis in childhood. Clinical urine analysis: pH- 4.8, leukocyturia, hematuria. Complete blood count: leukocytosis, lymphocytosis, raised ESR. Urography concludes: dilatation of renal pelvis and calyceal system of both kidneys, foci of calcification in the projection of right kidney parenchyma. What is the most likely diagnosis?

Explanation

Notice that from anamnesis, this patient already had a case of tuberculosis in childhood. Nephrotuberculosis is a long term complication of an improperly treated primary pulmonary Tuberculosis which usually occurs years after the initial TB infection. Nephrotuberculosis (renal tuberculosis) refers to a disease of the kidney due to Mycobacterium tuberculosis, usually from bacillemia (bacilli in circulation) in cases of pulmonary tuberculosis. Pathological changes include granulomatous inflammation and caseous necrosis of kidney tissue.
223. An unconscious patient was delivered to a hospital by an ambulance. Objectively his body temperature is 39°C, he presents with convulsions and red dry skin. It is known that the patient works as a stoker in the boiler room. What is the likely diagnosis?

Explanation

From anamnesis, we observe that this patient works in a very hot/ steaming environment; the signs shown by this patient i.e., a red dry skin and a temperature of 39℃ indicates the effect of heat on the patient. Heat stroke is a form of hyperthermia in which the body temperature is elevated dramatically. The cause of heat stroke is an elevation in body temperature, often accompanied by dehydration.

The most common symptoms of CO (carbon monoxide) poisoning are headache, dizziness, weakness, upset stomach, vomiting, chest pain, and confusion on the background of inhalation of fumes from cars, generators etc.

 
224. 2 hours after eating unknown mushrooms, a 28-year-old man sensed a decrease in his mobility and deterioration of his ability to focus. This condition was then followed by a state of agitation and agression. On examiantion he is disoriented and his speech is illegible. 4 hours later he developed fetor hepaticus and lost his consciousness. What syndrome can be observed in this patient?

Explanation

Fetor hepaticus is a strong musty smell observed from the breath of a patient whose liver is failing to detoxify a toxic substance. An acute liver failure is observed when the liver loses its function within days or weeks. Note that this patient started experiencing the expressed symptoms a few hours after consuming mushrooms; it can also be caused by infection, alcohol or drug induced. 

Cytolytic syndrome is a type of liver disease that leads in cell destruction; the major cause of this disorder is paracetamol overdose.

Hepatoliniel syndrome involves the reduction of kidney function accompanied by liver cirrhosis or liver failure.

 
225. A 19-year-old girl complains of moderate itching and hair loss on her head. Objectively on the skin of her occipital region there is a single round erythematous focus 3 cm in diameter with clear margins. Asbestos-like scales can be observed on the surface of the lesion. The hair in the focus of the lesion is broken off at the length of 6-8 mm. What is the most likely diagnosis?

Explanation

The description - a single round erythematous focus 3cm in diameter, asbestos like scales accompanied by itching and hair loss on the affected area is typical for Ringworm (tinea capitis in this case) also known as Microsporai; an infection caused by microsporum fungi.

Scabies is a parasitic skin infestation caused by the Sarcoptes scabiei. It is characterised by intense pruritus ( itching) at night; this characteristic and the location of the rashes helps us to diagnose scabies. The primary lessios found are usually papules, vesicles or burrows ( accompanied by excoriations and pustules) which are commonly seen in interdigital folds, flexor surfaces of the wrist, axillary folds, buttocks ( closed areas of the body).
226. A woman complains of frequent watery stool (up to 9-10 times per day) with mucus and blood admixtures, dull pain in the hypogastrium, weight loss of 4 kg within the last year. Objectively: malnutrition, dry skin, low turgor, aphthous stomatitis. The abdomen is soft, the sigmoid colon is spastic and painful on palpation. Occult blood test is positive. Fibrocolonoscopy: edema, hyperemia, mucosal granulation, pseudopolyp, small ulcers with irregular edges. Make the diagnosis:

Explanation

Differences between Ulcerative Colitis and Crohn Disease

Ulcerative Colitis

Crohn Disease

Only colon involved

Panintestinal

Continuous inflammation extending proximally

 from rectum

Skip-lesions with intervening normal mucosa

Inflammation in mucosa and submucosa only

Transmural inflammation

No granulomas

Noncaseating granulomas

Perinuclear ANCA (pANCA) positive

ASCA positive

Bleeding (common)

Bleeding (uncommon)

Fistulae (rare)

Fistulae (common)

Cathartic colon

Occult blood in stool is indicative of a hemorrhagic process from the colon, while the results from the colonoscopy indicates the pathology is located in the colon.

 

Irritable bowel syndrome is an inflammatory bowel disease characterised by recurrent abdominal cramps (pain), change in form (consistency) and frequency of stool, constipation etc. . This condition is common in middle aged women and is associated with underlying conditions such as stress, anxiety, depression or a previous case of intestinal infection.

227. A patient was delivered to a surgical department after a road accident with a closed trauma of chest and right-sided rib fracture. The patient was diagnosed with right sided pneumothorax; it is indicated to perform drainage of pleural cavity. Pleural puncture should be made:

Explanation

228. An 18-year-old patient was admitted to a hospital with complaints of headache, weakness, high fever, sore throat. Objectively: enlargement of all groups of lymph nodes was revealed. The liver is enlarged by 3 cm, spleen- by 1 cm. In blood: leukocytosis, atypical lymphocytes - 15%. What is the most probable diagnosis?

Explanation

 

Note that Atypical lymphocytes are mostly associated with Infectious Mononucleosis, an infection obtained from the epstein barr virus (EBV). These lymphocytes are termed atypical because they have a larger cytoplasm and nucleoli in their nuclei. All other symptoms are generalised and can indicate every other disease.

229. A 45-year-old patient complains of pain in the epigastric region, left subcostal area, abdominal distension, diarrhea, loss of weight. He has been suffering from this condition for 5 years. Objectively: tongue is moist with white coating near the root; deep palpation of abdomen reveals slight pain in the epigastric region and Мауо-Robson’s point. Liver is painless and protrudes 1 cm from the costal arch. Spleen cannot be palpated. What disease can be primarily suspected?

Explanation

 

It is stated that the patient has had this condition for 5 years; this rules out the possibility of an Acute form of the disease. The presence of an epigastric pain also confirms the answer isn't a Chronic enteritis. Pain and tenderness in the Mayo-Robson’s point indicates a pancreatic disorder. N/B Mayo-Robson's point – a point on the border of inner 2/3 with the external 1/3 of the line that represents the bisection of the left upper abdominal quadrant. This is where the tail of the pancreas is located. 

230. A 25-year-old patient was delivered to an infectious diseases unit on the 3rd day of illness with complaints of headache, pain in lumbar spine and gastrocnemius muscles, high fever, chill. Objectively: condition of moderate severity. Scleras are icteric. Pharynx is hyperemic. Tongue is dry with dry brown coating. Abdomen is distended. Liver is enlarged by 2 cm. Spleen is not enlarged. Palpation of muscles, especially gastrocnemius muscles, is painful. Urine is dark in colour. Stool is normal in colour. The most likely diagnosis is:

Explanation

Leptospirosis is an acute generalized infectious disease, characterized by extensive vasculitis, caused by spirochetes of the genus Leptospira. It presents with flu-like symptoms, myalgias ( majorly calf muscles eg gastrocnemius ), jaundice. Photophobia, icteric scleras etc. The icterohemorrhagic form of leptospirosis presents with a severe form of jaundice and azotemia from the liver and kidney dysfunction coupled with anemia.

N/B Virus hepatitis has a gradual onset, without chills, the temperature rises at the pre-icteric period. Muscle pains, scleritis, conjunctivitis are not characteristic of it. There are no meningeal and renal syndromes.

 
231. A 47-year-old patient came to see a doctor on the 7th day of disease. The disease is acute: after the chill body temperature rose to 40*C and lasted up to 7 hours, then dropped abruptly, which caused profuse sweat. There were three such attacks occurring once every other day. Two days ago the patient arrived from Africa. Objectively: pale skin, subicteric sclera, significantly enlarged liver and spleen. What is the cause of fever attacks in this disease?

Explanation

From anamnesis, we understand that the patient has just returned from Africa , this should prompt the idea of a malarial infection. Malaria is caused by a plasmodium infection (P.vivax, P.falciparum, P.malariae, P.ovale. In its life cycle, various stages are involved ( see image).

The initial symptoms of malaria infection are nonspecific and can include headache, nausea, vomiting, photophobia and muscle aches. A malarial paroxysm is marked by onset of a sudden shaking chill which may last from 10 to 15 minutes or perhaps longer. Elevated temperature accompanies the paroxysm and may be sustained for typically 10 hours or more. This cycle repeats itself every 36 to 72 hours depending on which species the human host has been infected with. The phase of erythrocytic schizogony takes place within  day 6-15 of the disease ( patient arrives on Day 7); it is characterized by the production of merozoites which are released into the bloodstream.

 
232. A 24-year-old patient complains of general weakness, dizziness, body temperature rise up to 37,5*C, sore throat, neck edema, enlargement of submaxillary lymph nodes. Objectively: mucous membrane of oropharynx is edematic and cyanotic, tonsils are enlarged and covered with films that spread beyond the tonsils and cannot be easily removed. What is the leading mechanism of this illness development?

Explanation

 

Note that the toxins are covered with films that ‘cannot be easily removed’ in another term, the examiner might say ‘bleeds when removed’; these phrases are characteristic of a corynebacterium diphtheria infection. This bacteria produces a Diptheria toxin  ( an Exotoxin).

233. A patient’s X-ray image (anteroposterior projection) shows deformation of lung pattern, pneumofibrosis, reticular (honeycomb) lung pattern of lower pulmonary segments, cylindric and fusiform lumps. The most likely diagnosis is:

Explanation

234. A 58-year-old patient complains of general weakness, weight loss up to 10 kg within the last 1,5 months, progressive pain in the small of the back, raise of blood pressure to 220/160mm Hg, subfebrile body temperature. Objectively: tuberous slightly movable lump can be palpated in the right subcostal area; veins of spermatic cord and scrotum are dilated. Blood test: Hb - 86 g/l, ESR - 44 mm/h. Urine test: specific gravity - 1020, proteine - 0,99g/l, erythrocytes - all field of vision, leukocytes- 4-6 in the field of vision. The provisional diagnosis is:

Explanation

The fact that the patient lost a lot of weight in a short period of time should signal a carcinogenic process. Key diagnostic features of this pathology include; blood in urine, presence of flank mass and pain accompanied with weight loss and high blood pressure; the dilated veins of the spermatic cord and scrotum (varices) is due to disrupted blood flow  caused by increased pressure from the tumour.

A patient with urolithiasis will also experience flank pain, change in quantity of urine and can be differentiated by results from ultrasound. Glomerulonephritis will present with a nephrotic syndrome ( increased proteinuria accompanied by presence of edema).

 
235. A 37-year-old patient complains of pain in the lumbar spine, which increases during walking, limited mobility, edema of the right side of abdomen. Focal tuberculosis is recorded in the anamnesis. X-ray shows destruction of adjacent surfaces of the bodies of the 1-2 lumbar vertebrae, vertebral bodies height is decreased, intervertebral fissure cannot be detected. Abdominal US reveals a growth 15x20 cm in size in the retroperitoneal space and echo signs of liquid. What diagnosis can be suspected?

Explanation

 

Also known as Pott’s disease, lumbar tuberculous spondylitis is a medical condition caused by the hematogenous spread of tuberculosis mostly from the lungs; these spread contents are latter deposited in the spine- mostly the lower thoracic and upper lumbar vertebrae; they can also affect the intervertebral disc  and generally cause symptoms of arthritis. Observe that from anamnesis, this patient has a history of tuberculosis and the various results from tests carried out shows a disruption in the normal form/functioning of the spine. A fracture will occur in case of trauma while osteochondrosis occurs mostly in developing individuals eg children and adolescents and is characterized by an interrupted blood supply to the bone especially the epiphysis of the bone.

236. A 53-year-old woman complains of weight loss up to 10 kg within the last 2 years, liquid foul smelling stool two times a day that poorly washes off the toilet, periodic bouts of nausea, girdle pain in the upper abdomen. Objectively: pain in Gubergrits zone (on the right from navel) and Mayo Robson’s point. Biochemical blood analysis: glucose - 3,2 mmol/l, bilirubin -16,5 micromole/l, crude protein - 56,4 g/l. Urine diastase - 426 g/h/l. D-xylose test (oral administration of 25 g of dxylose) after 5 hours reveals 3 g of xylose in urine. The most likely diagnosis is:

Explanation

Notice that the patient’s diastase level is high (norm: 25-125 U/L), diastase refers to groups of enzymes that catalyze the the breakdown of starch to maltose; these enzymes include α,β and γ amylase (especially α). The diastase level is useful in the diagnosing of diseases such as pancreatitis, jaundice, gall stones etc.

D-xylose test is a medical test performed to diagnose conditions that present with malabsorption (defect in GIT mucosa). In an individual with intact GIT mucosa, a 25 g oral dose of D-xylose will be absorbed and excreted in the urine at approximately 4.5 g in 5 hours ( it is 3.5g in this patient indicating a malabsorption syndrome).

 
237. A 23-year-old patient after intake of brake fluid has developed anuria that has been lasting for 5 days already. Creatinine level increased up to 0,769 mmol/l. What treatment tactics should be chosen in the given case?

Explanation

 

Hemodialysis is a procedure used for removal of waste materials from the blood through filtration. It is carried out on people with renal failure. This patient experiences anuria ( production of only about 50-100ml of urine daily); creatinine levels are also rising; This procedure should be carried out to remove such waste products since the amount of urine being produced is inadequate.

238. A patient complains of weight gain, chill, edemas, xeroderma, somnolence, difficulties with focusing. Objectively: height is 165 cm; weight is 90 kg; body proportions are of female type, to - 35,8*C, heart rate - 58/min, BP -105/60 mm Hg. Heart sounds are weakened, bradycardia is observed. Other internal organs have no changes. Thyroid gland cannot be palpated. Milk secretion from mammary glands is observed. Hormone investigation revealed increased levels of thyroid stimulating hormone (TSH) and prolactin, and decreased level of thyroxine (T4). Which one is the cause for obesity?

Explanation

Hypothyroidism is an endocrine disorder in which the thyroid gland does not produce enough thyroid hormones. It is classified under congenital and acquired. The acquired form is further divided into Primary, secondary and tertiary forms.

Primary Hypothyroidism: Occurs due to thyroid gland disturbances. This can be as a result of environmental factors, autoimmune causes e.g. Hashimoto, radiation etc. Characterized by low levels of T3 triiodothyronine and T4 thyroxine.

Secondary hypothyroidism: occur due to pituitary gland disorders. In this form, thyroid stimulating hormone (TSH) is low.

Tertiary Hypothyroidism: Occurs due to hypothalamic related disorders. Characterized by a low level of thyroid releasing hormone.

Symptoms include intolerance to cold, hair loss, fatigue, brittle nails, weight gain, bradycardia etc. sometimes referred to as Myxedema.

 
239. A 54-year-old patient complains of weakness, jaundice, itching skin. Disease onset was 1,5 months ago: fever up to 39*C appeared at first, with progressive jaundice developed 2 weeks later. On hospitalisation jaundice was severely progressed. Liver cannot be palpated. Gallbladder is enlarged and painless. Blood bilirubin is 190 micromole/l (accounting for direct bilirubin). Stool is acholic. What is the most likely jaundice genesis in this patient?

Explanation

Prehepatic/ hemolytic - occurs due to increased breakdown of RBC eg, hemolytic disease of newborn. There is an increased level of unconjugated bilirubin. Stool and urine colour are normal.

Hepatic/ parenchymatous  - Occurs in diseases affecting the liver parenchyma eg, cirrhosis, hepatitis etc. There is an increase in both unconjugated and conjugated bilirubin. Urine appears dark and faeces pale.

Post hepatic/ obstructive/ mechanical jaundice - pathology lies after conjugation of bilirubin and is caused by obstruction of biliary path. Conjugated bilirubin is accumulated, Urine is dark and faeces pale or acholic. Seen in disease such as cholelithiasis.

 
240. A 40-year-old patient complains of constant moderate pain in the lumbar spine and significantly reduced mobility. The patient has been suffering from this condition for the last 7 years since pain appeared first in the sacrum area. X-ray: ankylosis of sacroiliac articulation, significant narrowing of intervertebral joint fissures of lumbar vertebrae and calcification of spinal ligaments. What pathology is most likely to cause such X-ray image?

Explanation

 

Ankylosing spondylitis (AS) is a chronic, multisystem inflammatory disorder primarily involving the sacroiliac (SI) joints and the axial skeleton. Overtime, it results in the fusion of  the small bones in your spine (vertebrae) making the spine less flexible and can result in a hunched-forward posture. The term “swayback” is an inappropriate curve of the upper lumbar spine; And Bamboo spine is a radiographic feature seen in ankylosing spondylitis that occurs as a result of vertebral body fusion. Notice on X-ray ‘ankylosis of sacroiliac articulation, significant narrowing of intervertebral joint fissures of lumbar vertebrae and calcification of spinal ligaments’.

241. After overexposure to cold a 45-year- old woman developed acute pain in her suprapubic and lumbar areas during urination, sharp pains at the end of urination, false urges to urinate. Urine is turbid with blood streaks. The doctor suspects urinary tract infection. What results of laboratory analysis would be the most indicative of such infection?

Explanation

Urinary tract infections are commonly associated with dysuria, urinary frequency, urinary urgency and lumboabdominal pains.

When you have a UTI, the lining of the bladder and urethra become inflammed and irritated just as your throat does when you have phrayngitis, common cold - upper respiratory tract infections. The irritation can cause pain in your lower abdomen - pelvic area and even lower back, and will usually make you feel like urinating more often (false urges). Burning or pain when urinating is the most common symptom. You may even feel a strong urge or need to urinate but only get a few drops. This is because the bladder is so irritated that it makes you feel like you have to urinate, even when you don't have much urine in your bladder. UTIs can be found by analyzing a urine sample for Urine Microscopy/Culture/Sensitivity (Urine M/C/S). The urine is examined under a microscope for bacteria or white blood cells (leukocyturia), which are signs of infection. Blood can also be found in the urine sample (hematuria); this can be due to an infective or mechanical cause.

 

The other options given are more specific for kidney pathologies - proteinuria, gross hematuria, creatinine and urea. Rarely are they elevated in pathologies relating to Urinary Tract Infections except in cases where UTI’s become complicated and the kidney is affected after a prolonged and untreated UTI.

242. A 34-year-old man complains of pale edema of the face, feet, shins, and lumbar area, elevated blood pressure up to 160/100 mm Hg, and general weakness. He has a clinical history of nonspecific ulcerative colitis. Objectively: pulse - 84/min., rhythmic, blood pressure - 165/100 mm Hg; edemas all over the body; the skin is pale and dry, with low turgor. The kidneys cannot be palpated, on an attempt to palpate them they are painless. Blood test: erythrocytes - 3.0∙1012/L, Hb- 100 g/L, erythrocyte sedimentation rate - 50 mm/hour. Urinalysis: proteins - 3.5 g/L, erythrocytes -7-10 in the vision field, leukocytes - 5-6 in the vision field. Daily proteinuria 6 grams. What analysis should be conducted additionally to verify the diagnosis?

Explanation

243. A 40-year-old man with Bekhterev disease (ankylosing spondylitis) complains of elevated body temperature up to 37.8°C, back pain and stiffness, especially observed during the second half of the night. This condition has been lasting for 2 years. Objectively: reduced spinal mobility, painful sacroiliac joint, erythrocyte sedimentation rate - 45 mm/hour. X-ray shows narrowing of the intervertebral disc space and of the sacroiliac joint. What eye pathology is often associated with this type of disease progression?

Explanation

Ankylosing Spondylitis(AS) is a type of arthritis. It causes pain and stiffness, mainly in your spine. But it can also cause eye inflammation called uveitis. Left untreated, uveitis can harm your vision and, in some cases, lead to blindness.

Uveitis - An inflammation of the middle layer of the eye (uvea).

The most common type is an inflammation of the iris called iritis (anterior uveitis). If the ciliary body is also involved, it is called iridocyclitis.

About half of people with ankylosing spondylitis have uveitis at least once. It’s one of the most common complications of that form of arthritis.

 

Your eye doctor could actually be the first to figure out you have AS. That’s because the same inflammation that makes your back hurt can also cause inflammation in your eyes and other parts of your body.

244. A 27-year-old man complains of pain in his leg joints, purulent discharge from the eyes, and painful burning sensations during urination. Disease onset was acute. He has a history of influenza. The patient smokes and drinks alcohol in excess. In his line of work he is often away on business trips. What is the most likely etiological factor of this disease?

Explanation

Reactive arthritis is an inflammatory arthritis which manifests after several days to weeks after a gastrointestinal or genitourinary infection. It is also described as a classic triad of arthritis, urethritis and conjunctivitis. However, a majority of patients do not present with the classic triad. It was previously called "Reiter syndrome."

Classic triad:

Conjunctivitis: purulent discharge from the eyes

Urethritis: painful burning sensations during urination

Arthritis: pain in leg joints

 

Reactive arthritis is known to be triggered by a bacterial infection, particularly of the genitourinary (Chlamydia trachomatis, Neisseria gonorrhoeae, Mycoplasma hominis, and Ureaplasma urealyticum) or gastrointestinal (GI) tract (Salmonella enteritidis, Shigella flexneri, and dysenteriae, Yersinia enterocolitica, Campylobacter jejuni, Clostridium difficile). 

The classic triad can be remembered using this:

 

Conjunctivitis; Urethritis; Arthritis - Can’t See; can’t pee, can’t bend my knee

 
245. A 51-year-old man complains of vomiting with blood. He has been drinking alcohol excessively. Health disorder has been observed since he was 40, when he first developed jaundice. On examination the skin and visible mucosa are icteric, with a stellate vascular pattern. The patient is malnourished and presents with abdominal distension, umbilical hernia, and ascites. The edge of the liver is tapered and painless, +3 cm, the spleen is +2 cm. Blood test: Hb - 80 g/L, leukocytes -3∙109/L, platelets - 85∙109/L. What is the cause of portal hypertension in this patient?

Explanation

Cirrhosis is a complication of liver disease that involves loss of liver cells and irreversible scarring of the liver. The scarring is accompanied by loss of functioning liver cells. Cirrhosis is usually a late-stage disease of the liver that is not reversible in severe cases.

Hepatitis B, C, and chronic alcohol abuse are frequent causes (Question stem - patient takes alcohol excessively). Initially patients may experience fatigue, weakness and weight loss. During later stages, patients may develop jaundice (yellowing of the skin), gastrointestinal bleeding, abdominal swelling and confusion.

The most common cause of portal hypertension is cirrhosis of the liver. Portal hypertension is an increase in the blood pressure within a system of veins called the portal venous system. Veins coming from the stomach, intestine, spleen, and pancreas merge into the portal vein, which then branches into smaller vessels and travels through the liver. If the vessels in the liver are blocked due to liver damage, blood cannot flow properly through the liver. As a result, high pressure in the portal system develops.

 

Budd-Chiari syndrome is a condition in which the hepatic veins (veins that drain the liver) are blocked or narrowed by a clot (mass of blood cells). It may lead to portal hypertension, ascites, cirrhosis etc. Budd-Chiari syndrome can be caused by conditions and situations that cause your blood to clot (form a blockage) like sickle cell disease, pregnancy, myeloproliferative disorders. It is not caused by excessive alcohol intake or Hepatitis.

 

Hemochromatosis, Constrictive Pericarditis and Thrombosis of the splenic vein will not directly lead to portal hypertension.

 
246. A 53-year-old man complains of general weakness, loss of appetite, and painful vesicles appearing on his skin. The disease onset occurred suddenly, after hyperinsolation one week ago. Examination detects isolated vesicles with wrinkled opercula and occasional painful erosions on the skin of the patient’s torso and limbs. Nikolsky sign is positive. What is the most likely diagnosis?

Explanation

Nikolsky's sign has been a very useful diagnostic tool in cases of skin disorders like pemphigus, toxic epidermal necrolysis, etc., The sign is demonstrated when lateral pressure is applied on the border of an intact blister, which results in the dislodgment of the normal epidermis and extension of the blister. For the Nikolsky sign in patients with active blistering, firm sliding pressure with a finger separates normal-appearing epidermis, producing an erosion. This sign is made possible due to the loss of cell-to-cell adhesion in the epidermis.

Acantholysis is the loss of intercellular connections, such as desmosomes, resulting in loss of cohesion between keratinocytes, seen in diseases such as pemphigus vulgaris. It is absent in bullous pemphigoid, making it useful for differential diagnosis. Hence, Nikolsky’s sign will be negative in non-acantholytic pemphigus.

Nikolsky's sign is pathognomonic for pemphigus, toxic epidermal necrolysis, and staphylococcal scalded skin syndrome (SSSS). This sign basically differentiates intraepidermal blisters from subepidermal blisters.

 

Toxicoderma are usually caused by drugs and this patient has no history of taking any medication that would have resulted in toxicoderma. Dermatitis herpetiformis is an autoimmune blistering disorder associated with a gluten-sensitive enteropathy (patient wasn’t diagnosed of celiac disease). Herpes usually presents as grouped vesicles, meanwhile, on examination of the patient isolated vesicles were detected.

247. A 48-year-old woman developed insomnia, depressive mood, anxiety, fears and suicidal thoughts after the death of her husband that occurred one month ago. During her stay in the hospital she speaks in a low voice, is depressed, anxious, avoids sleeping, refuses to eat. What medications should be prescribed in this case?

Explanation

The question specifies that the described lady is Depressed therefore, antidepressant is the best choice among the listed groups. Antipsychotic drugs/neuroleptics are used in cases of Schizophrenia or bipolar disorders. Nootropics are memory enhancing drugs.

Examples of antidepressants:

 
  • Tricyclic antidepressants: Amitriptyline, Imipramine

  • Monoamine oxidase inhibitors: Selegiline

  • Selective Serotonin reuptake inhibitors: Fluoxetine, Paroxetine, Sertraline

  • Serotonin-Norepinephrine reuptake Inhibitors: Venlafaxine, Duloxetine

248. A woman has been provisionally diagnosed with pheochromocytoma. At the stage of intermission her BP is within norm; there is a tendency towards tachycardia. No urine pathologies. The decision has been made to perform a provocative test with histamine. What drug should be kept close at hand for emergency aid in case of positive test result?

Explanation

 

It is important to note that certain drugs can be used for the diagnosis of Pheochromocytoma (a tumor of the adrenal medulla). The Histamine test was the first test introduced and later on drugs such as  piperoxan, tetraethylammonium bromide, methacholine chloride  and phentolamine hydrochloride (Regitine) were added. Histamine, tetraethylammonium bromide, and methacholine chloride (Mecholyl) stimulate the discharge of the pressor substances from the tumor. Thus, tests with these drugs are useful in cases of paroxysmal hypertension to produce attacks similar to those that occur spontaneously. Phentolamine (Regitine) and piperoxan (Benzodioxane), on the other hand, lower blood pressure by blocking the pressor effect of epinephrine and norepinephrine in the blood if a pheochromocytoma is present.

249. A 52-year-old man for the last 3 years has been suffering from difficult swallowing of solid food, burning retrosternal pain that aggravated during eating, loss of body mass, and occasional vomiting with undigested food. Esophageal X-ray shows S-shaped deformation of the esophagus and its dilation; at the cardiac orifice the esophagus is constricted; esophageal mucosa is smooth, without signs of peristalsis. Make the provisional diagnosis:

Explanation

250. A 72-year-old man diagnosed with ischemic heart disease presents with diffuse cardiosclerosis, permanent tachysystolic atrial fibrillation, heart failure IIa, FC III. Objective examination of vital signs: blood pressure is 135/80 mm Hg, heart rate is 160/min., pulse is 125/min. Left ventricular ejection fraction is 32%. What drug is indicated in this case and should be prescribed to the patient?

Explanation

Basically, in this patient we are looking for a medication that will take care of all the conditions - atrial fibrillation and heart failure.

Digoxin is a cardiac glycoside used to treat atrial fibrillation, mild to moderate heart failure in adults, and heart failure in children. Digoxin belongs to a class of drugs called antiarrhythmics. It works by slowing your heart rate down and improving the way your ventricles are filled with blood. In patients with heart failure, it increases contractility and in Atrial fibrillation, it decreases conduction at AV node and depression of SA node.

Verapamil: Class IV antiarrhythmic drug - calcium channel blocker. It is contraindicated in patients with heart failure.

Ivabradine: mainly used to treat heart failure conditions.

Procainamide: Class I antiarrhythmic drug - sodium channel blocker. It can be used for both atrial and ventricular arrhythmias but not heart failure. 

Isadrine (Isoprenaline): is a non-selective β adrenoceptor agonist, a bronchodilator that works by relaxing muscles in the airways to improve breathing.
251. A 32-year-old woman complains of tumor-like formation on the anterior surface of her neck that appeared 2 years ago. Within the last 3 months the tumor has been rapidly growing. It hinders swallowing and impairs speech; the tumor causes a sensation of pressure. Objectively the skin moisture is normal, pulse is 80/min., rhythmic, blood pressure is 130/80 mm Hg. In the right lobe of the thyroid gland there is a dense lumpy node 3.0x3.5 cm that moves during swallowing. Scanning image shows a ”cold nodule” in the thyroid gland. Make the provisional diagnosis:

Explanation

Your doctor may recommend a thyroid scan to help evaluate thyroid nodules. During this test, an isotope of radioactive iodine is injected into a vein in your arm. You then lie on a table while a special camera produces an image of your thyroid on a computer screen.

Nodules that produce excess thyroid hormone — called hot nodules — show up on the scan because they take up more of the isotope than normal thyroid tissue does. Hot nodules are almost always non-cancerous.

In some cases, nodules that take up less of the isotope — called cold nodules — are cancerous.

Therefore, the best choice here is Thyroid Cancer, as the scanning image shows a “cold nodule.”

252. A 32-year-old woman complains of body weight loss despite her increased appetite, nervousness, and tremor of the extremities. Objectively: the skin is moist; the thyroid gland is diffusely enlarged, painless, soft, and mobile. Blood test: increased level of T3, T4, and thyroid-stimulating hormone (TSH). What is the most likely diagnosis?

Explanation

Diffuse toxic goiter or grave’s disease is an autoimmune disease characterised by an increased amount of thyroid hormones ( T3- triiodothyronine and T4- thyroxine). Symptoms include; loss of weight, with increased appetite, increased sweating, cardiac findings (tachycardia, increased risk for atrial fibrillation), tremors of extremities, heat intolerance, diarrhea, anxiety etc.

It is the most common cause of hyperthyroidism and more common in females than males. There is production of IgG antibodies against the TSH receptor (stimulating type of antibody - Type II hypersensitivity reaction)

Hashimoto’s thyroiditis is a destructive autoimmune thyroiditis leading to hypothyroidism.

In Diffuse non-toxic goitre, thyroid function is normal and patients are often asymptomatic. It manifests without hyperthyroidism, hypothyroidism, or inflammation.
253. A 45-year-old man, a farmer, presents with acute onset of a disease. He complains of headache, high temperature, pain in the gastrocnemius muscles, icteric face, and dark urine. Objectively: body temperature - 38°C, blood pressure - 100/70 mm Hg, conjunctival hemorrhages, hepatosplenomegaly, and oliguria. What is the most likely provisional diagnosis?

Explanation

Leptospirosis is an acute generalized infectious disease, characterized by extensive vasculitis, caused by spirochetes of the genus Leptospira. It presents with flu-like symptoms, myalgias (majorly calf muscles eg gastrocnemius), jaundice (icteric face), Photophobia etc. The icterohemorrhagic form of leptospirosis presents with a severe form of jaundice and azotemia from the liver and kidney dysfunction coupled with anemia. Leptospira is commonly found in water contaminated with animal urine.

N/B Virus hepatitis has a gradual onset, without chills, the temperature rises at the pre-icteric period. Muscle pains, scleritis, conjunctivitis are not characteristic of it.

 

Brucellosis is caused by gram negative bacteria, transmitted via ingestion of contaminated animal products (eg, unpasteurized milk). Typically presents with fever, night sweats, and arthralgia but jaundice is usually absent.

254. A 38-year-old patient has been brought by an ambulance to the surgical department with complaints of general weakness, indisposition, black stool. On examination the patient is pale, there are dotted hemorrhages on the skin of his torso and extremities. On digital investigation there are black feces on the glove. Blood test: Hb - 108 g/L, thrombocytopenia. Anamnesis states that a similar condition was observed 1 year ago. Make the diagnosis:

Explanation

Immune thrombocytopenic purpura (ITP) is a clinical syndrome in which a decreased number of circulating platelets (thrombocytopenia) present as a bleeding tendency, easy bruising (purpura), or extravasation of blood from capillaries into skin and mucous membranes (petechiae). Recall that the patient presents with dotted hemorrhages on the skin (petechiae) and on analysis, thrombocytopenia. Caused by the binding of an autoantibody (specifically IgG) to platelets leading to platelet destruction.

 

In Hemophilia, patients will present with bleeding from joints (hemathroses), easy bruising or a case of bleeding after surgery, dental procedure. Has three forms; Hemophilia A ( Factor VIII deficient), Hemophilia B ( factor IX deficient), C ( factor XI def.).

255. A 42-year-old man, a worker at the meat processing factory, developed an itching spot on his lower jaw, which gradually transformed into a slightly painful carbuncle 3 cm in diameter, surrounded by a painless swelling that reaches the clavicle. Temperature is subfebrile, under 37.8°C. The doctor suspects anthrax. What drug should this man be prescribed for treatment?

Explanation

Bacillus anthracis is an aerobic, non-motile, spore forming, large non-hemolytic Gram-positive rod that grows well on blood agar. The clinical manifestations of human anthrax - Cutaneous and Inhalation. 

Cutaneous anthrax, the most common form of naturally occurring disease, begins as a small, painless, pruritic papule that within 2 days enlarges, develops vesicles, and ulcerates to form an eschar. Inhalational anthrax follows inhalation of infectious doses of anthrax spores.

Four antibiotics are approved for use for post exposure prophylaxis following exposure to aerosolized spores of B. anthracis: doxycycline, ciprofloxacin, levofloxacin, and parenteral procaine penicillin G. Of all these four antibiotics, only Penicillin is listed as a choice for this question.
256. A patient is being treated in the tuberculosis clinic. Throughout the last 3 weeks he has been suffering from headaches of increasing intensity. Neurological examination detects nuchal rigidity without focal signs. Make the provisional diagnosis:

Explanation

Meningitis is an acute infectious disease with involvement of the arachnoid and pia mater of the brain and spinal cord by pathogenic microorganisms. Etiologically, the various forms include; bacterial, viral. Fungal and tuberculous meningitis. The tuberculous form occurs as a secondary infection (from anamnesis, the patient already suffers from pulmonary tuberculosis).

 

The classic triad of fever, headache, and neck stiffness (nuchal rigidity) is usually present.

257. A 37-year-old man suddenly developed acute headache accompanied by nausea, vomiting, and impaired consciousness. Objectively blood pressure is 190/120 mm Hg, the face is hyperemic. Patient’s consciousness is clouded, his answers to the questions are short, monosyllabic. Movement and sensory disturbances are absent. Meningeal signs are positive. Cerebrospinal fluid contains blood. What provisional diagnosis can be made?

Explanation

The term subarachnoid hemorrhage (SAH) refers to extravasation of blood into the subarachnoid space i.e. the space between the pia and arachnoid membranes.

The subarachnoid space is the space where the cerebrospinal fluid (CSF) circulates, therefore, hemorrhage into this space will result in bloody CSF as mentioned in the question. The CSF is responsible for protecting your brain from injury by serving as a cushion. A hemorrhage in this space can cause a coma, paralysis, and even death. It is often characterized by a severe headache (often referred to as ‘worst headache of my life’).

Severe headaches and bloody CSF are the hallmarks for making Subarachnoid hemorrhage diagnosis especially in patients with Severe Hypertension.

 

The most common cause of primary Subarachnoid Hemorrhage is a berry aneurysm. These aneurysms swell up and weaken the walls of the arteries over time. Brain aneurysms are more common in smokers, and in those with elevated blood pressure. In some cases, trauma to the brain during an injury can cause aneurysms and result in a subarachnoid hemorrhage.

258. A 20-year-old student after failing an exam developed complaints of a sensation of a round foreign body in her throat, difficult swallowing. She fixates on her condition, limits her diet, often cries, seeks attention, exhibits demonstrative attitude. She is highly susceptible to psychotherapeutic suggestion. What psychiatric diagnosis can be made in this case?

Explanation

259. A 39-year-old man suffers from chronic rheumatic heart disease. He complains of dyspnea during physical exertion, cough with expectoration, and palpitations. Ausculation detects intensified I heart sound and diastolic murmur; the sound of opening mitral valve can be auscultated at the cardiac apex. The II heart sound is accentuated over the pulmonary artery. The patient is cyanotic. X-ray shows dilated pulmonary root and enlargement of the right ventricle and left atrium. What is the most likely diagnosis?

Explanation

 

Valvular diseases

Mitral stenosis

Aortic stenosis

Aortic regurgitation

Mitral regurgitation

Tricuspid regurgitation

Main causes and risk factors

Almost always caused by rheumatic heart disease

-Calcification of tricuspid aortic valve with age (>50%)

-Calcification of bicuspid aortic valve (30-40%)

-Rheumatic fever (<10%)

Hypertension, diabetes mellitus, hyperlipoproteinemia and uremia may speed up the process.

Acute

-Infective endocarditis

-Trauma

Chronic

-Primary valvular: rheumatic fever, bicuspid aortic valve, Marfan's syndrome, Ehlers–Danlos syndrome, ankylosing spondylitis, systemic lupus erythematosus

-Disease of the aortic root: syphilitic aortitis, osteogenesis imperfecta, aortic dissection, Behçet's disease, reactive arthritis, systemic hypertension

Acute

-Endocarditis, mainly S. aureus

-Papillary muscle rupture or dysfunction, including mitral valve prolapse

Chronic

-Rheumatic fever

-Marfan's syndrome

-Cardiomyopathy

Usually secondary to right ventricular dilation

Other causes: Tricuspid endocarditis, rheumatic fever, Ebstein's anomaly, carcinoid syndrome and myxomatous degeneration

Symptoms

Heart failure symptoms, such as dyspnea on exertion, orthopnea and paroxysmal nocturnal dyspnea

-Palpitations

-Chest pain

-Hemoptysis

-Thromboembolism

-Ascites and edema (if right-sided heart failure develops)

Symptoms increase with exercise and pregnancy

-Heart failure symptoms, such as dyspnea on exertion (most frequent symptom), orthopnea and paroxysmal nocturnal dyspnea

-Angina pectoris

-Syncope, usually exertional

-Heart failure symptoms, such as dyspnea on exertion, orthopnea and paroxysmal nocturnal dyspnea

Palpitations,

-Angina pectoris

-In acute cases: cyanosis and circulatory shock

-Heart failure symptoms, such as dyspnea on exertion, orthopnea and paroxysmal nocturnal dyspnea

-Palpitations

-Pulmonary edema

Symptoms of right-sided heart failure, such as ascites, hepatomegaly, edema and jugular venous distension

Medical signs

-Opening snap followed by a low-pitched diastolic rumble with presystolic accentuation.

The opening snap follows closer to the S2 heart tone with worsening stenosis.

The murmur is heard best with the bell of the stethoscope lying on the left side and its duration increases with worsening disease.

-Loud S1 - may be the most prominent sign

-Advanced disease may present with signs of right-sided heart failure such as parasternal heave, jugular venous distension, hepatomegaly, ascites and/or pulmonary hypertension (presenting with a loud P2.

Signs increase with exercise and pregnancy

Systolic murmur of a harsh crescendo-decrescendo type, heard in 2nd right intercostal space, radiating to the carotid arteries

-Pulsus parvus et tardus, that is, diminished and delayed carotid pulse

-Fourth heart sound

-Decreased A2 sound

-Sustained apex beat

Precordial thrill

-Increased pulse pressure by increased systolic and decreased diastolic blood pressure, but may not be significant if acute

-Diastolic decrescendo murmur best heard at left sternal border

Water hammer pulse

-Austin Flint murmur

-Apex beat displaced down and to the left

-Third heart sound may be present

Holosystolic murmur at the apex, radiating to the back or clavicular area

Commonly atrial fibrillation

Third heart sound

Laterally displaced apex beat, often with heave

Loud, palpable P2, heard best when lying on the left side

-Pulsatile liver

-Prominent V waves and rapid  descents in jugular venous pressure

-Inspiratory third heart sound at left lower sternal border (LLSB)

-Blowing holosystolic murmur at LLSB, intensifying with inspiration, and decreasing with expiration and -Valsalva maneuver

-Parasternal heave along LLSB

-Atrial fibrillation is usually present